Tài liệu hạn chế xem trước, để xem đầy đủ mời bạn chọn Tải xuống
1
/ 237 trang
THÔNG TIN TÀI LIỆU
Thông tin cơ bản
Định dạng
Số trang
237
Dung lượng
9,65 MB
Nội dung
Tailieumontoan.com Điện thoại (Zalo) 039.373.2038 PHÁT TRIỂN KỸ NĂNG GIẢI TỐN HÌNH HỌC CHO BẬC THCS (Liệu hệ tài liệu word mơn tốn SĐT (zalo) : 039.373.2038) Tài liệu sưu tầm, ngày 15 tháng năm 2023 Website: tailieumontoan.com I TAM GIÁC – TỨ GIÁC Bài Cho tam giác ABC, vuông A ∠ABC = 60° Hãy chia tam giác ABC thành bốn tam giác vuông LỜI GIẢI Cách 1: Bốn tam giác AFE, BFD, EDF, DEC tam giác vuông Theo giả thiết tam giác ABC vuông A ∠ABC = 60° , ta có ∠ABC= 30° ⇒ BC= AB Gọi D, E, F trung điểm cạnh BC, CA, AB ⇒ EF = BD = DC Và AF = FB = ED, EF / / BC , ED / / AB, FD / / AC Suy ∠EDF = ∠DEC = ∠BFD = 90° Cách 2: AD = BD = DC ⇒ bốn tam giác AFD, DEA, BDF, DEC tam giác vng Cách 3: Ta có: AD = AB Hạ AH vng góc với BC Khi tam giác ABH ADH Suy bốn tam giác ABH, ADH, ADE, CDE Cách 4: Tam giác ABD tam giác đều, K trung điểm AD, suy bốn tam giác vuông BAK, BDK, ADE, CED Bài Có tồn hay khơng tam giác có hai đường trung tuyến nhỏ nửa cạnh đối diện? LỜI GIẢI Giả sử BD CE hai đường trung tuyến tam giác ABC thỏa mãn BD < 1 AC CE < AB 2 Ta có: BD < AD BD < DC Liên hệ tài liệu word toán SĐT (zalo): 039.373.2038 Website: tailieumontoan.com Xét tam giác ABD có BD < AD , theo tính chất quan hệ góc cạnh đối diện tam giác, ta có ∠BAD < ∠ABD (1) Tương tự BD < CD ⇒ ∠BCD < ∠DBC (2) Cộng hai vế đẳng thức (1) (2) ta được: Theo tính chất tổng ba góc tronng tam giác ⇒ 180° − ∠ABC < ∠ABC hay ∠ABC > 90° Tương tự ∠ACB > 90° Như tam giác ABC có hai góc ∠ABC > 90° ∠ACB > 90° Điều không xảy Vậy không tồn tam giác có hai đường trung tuyến nhỏ nửa cạnh cạnh đối diện Bài Cho tam giác ABC, trung tuyến AD Chứng minh 1 AB − AC < AD < ( AB + AC ) 2 LỜI GIẢI Lấy điểm E tia đối tia DA cho DE = AD Giả thiết DB = DC ⇒ ∆ADB = ∆EDC (c.g c) ⇒ CE = AB Áp dụng bất đẳng thức tam giác ACE: AC − CE < AE < ( AC + CE ) ⇒ AC − AC < AD < ( AC + AB ) ⇒ 1 AC − AC < AD < ( AC + AB ) 2 Bài Cho tam giác nhọn ABC, đường cao BD CE Gọi P, Q hình chiếu B C DE Chứng minh PE = DQ LỜI GIẢI Gọi M trung điểm cạnh BC Theo giả thiết, BD CE đường cao tam giác ABC ⇒ tam giác BDC CEB tam giác vuông nhận BC cạnh huyền ⇒= MD 1 BC ,= ME BC 2 ⇒ MD = ME ⇒ tam giác MDE cân M Gọi N hình chiếu M DE Khi ND = NE MN song song với BP CQ Giả thiết MB =MC ⇒ S MBN =S MCN ⇒ S MPN =S MQN ⇒ NP = NQ ⇒ PE = NP − NE = NQ − ND = DQ Liên hệ tài liệu word toán SĐT (zalo): 039.373.2038 Website: tailieumontoan.com Bài Cho hình vng ABCD Trên cạnh AB AD lấy điểm P Q AP + AQ = AB Chứng minh DP vng góc với CQ LỜI GIẢI Theo giả thiết AP + AQ = AB , suy AP = AB − AQ = AD − AQ = DQ Tam giác ADP DCQ hai tam giác vuông có = AB CD = , AP DQ ⇒ hai tam giác ADP DCQ ⇒ ∠ADP = ∠DCQ ABCD hình vng nên: ∠PDC= 90° − ∠ADP ⇒ ∠PDC + ∠QCD= 90° − ∠ADP + ∠ADP= 90° ⇒ DP vng góc với CQ Bài Cho hình vuông ABCD Đường thẳng m thay đổi qua đỉnh B Gọi H K hình chiếu A C m AK CH cắt E Chứng minh DE vng góc với m LỜI GIẢI Nối H với D nối K với D Vì H, B, K nằm đường thẳng m ⇒ ∠HBA + ∠KBC= 90° Mà ∠HBA + ∠HAB = 90° ⇒ ∠HBA = ∠KCB Tam giác HAB tam giác KCB hai tam giác vng có AB = BC ∠HBA = ∠KCB ⇒ hai tam giác HAB KCB ⇒ HB = CK HA = BK Ta có ∠HAD = ∠HAB + ∠BAD = ∠KBC + 90° = ∠ABK ⇒ tam giác HAD tam giác KBA (c.g.c) ⇒ ∠ADH = ∠BAK Mặt khác ∠BAK + ∠DAK= 90° ⇒ ∠ADH + ∠DAK= 90° ⇒ DH vng góc với AK Tương tự CH vng góc với DK Tam giác HDK có AK CH hai đường cao cắt E ⇒ E trực tâm tam giác ⇒ BE vuông góc với HK ⇒ DE vng góc đường thẳng m Bài Cho hình vng ABCD Lấy điểm M BC điểm N CD Biết BM + DN = MN Chứng minh ∠MAN =° 45 Liên hệ tài liệu word toán SĐT (zalo): 039.373.2038 Website: tailieumontoan.com Điều ngược lại có khơng? LỜI GIẢI Từ A kẻ đường thẳng vng góc với MA cắt CD kéo dài E ⇒ ∠BAM = 90° − ∠MAD = ∠MAE − ∠MAD = ∠DAE Tam giác ABM AB =∠ AD, BAM = ∠DAE ADE nên hai chúng tam giác vng có (g.c.g) ⇒ AM = AE , BM = DE Từ giả thiết: MN = BM + DN = DE + DN = NE Xét hai tam giác AMN AEN có AM = AE , MN = NE cạnh AN chung ⇒ hai tam giác AMN AEN (c.c.c) ⇒ ∠MAN = ∠EAN Mà ∠MAE= 90° ⇒ ∠MAN= 45° Ngược lại, ∠MAN =° 45 , AE vuông góc với AM 45° ⇒ AM = AE , BM = DE ∠EAN = ⇒ tam giác MAN EAN (c.g.c) ⇒ MN = EN hay BM + DN = MN Bài Cho hình vng ABCD Góc ∠xAy =45° quay quanh đỉnh A cắt cạnh BC, CD M N Chứng minh rằng: a) Chu vi tam giác CMN không phụ thuộc vào vị trí chuyển động góc ∠xAy b) Khoảng cách từ A đến MN không đổi LỜI GIẢI a) Qua A kẻ đường thẳng vng góc với AM cắt CD E ⇒ ∠BAM = ∠DAE ⇒ tam giác ABM tam giác ADE (c.g.c) ⇒ BM = DE , AM = AD Theo giả thiết góc ∠MAN =° 45 ⇒ ∠NAE= 45° ⇒ tam giác NAM tam giác NAE (c.g.c) ⇒ MN = EN ⇒ chu vi tam giác CMN = MN + NC + CM = EN + NC + CM = ED + DN + NC + CM = BM + DN + NC + CM = DC + CB Vậy chu vi tam giác CMN nửa chu vi hình vng ABCD b) Hạ AH vng góc với MN Tam giác AMN tam giác AEN Liên hệ tài liệu word toán SĐT (zalo): 039.373.2038 Website: tailieumontoan.com ⇒ hai đường cao tương ứng AH = AD ⇒ khoảng cách từ A tới MN không đổi Bài Cho hình vng ABCD Góc có số đo 45° quay quanh đỉnh A cắt cạnh BC, CD M N Đường chéo BD cắt AM, AN P, Q Chứng minh QP = QD + BP LỜI GIẢI Cách Từ A kẻ đường thẳng vng góc với AM, từ D kẻ đường thẳng vng góc với BD, hai đường cắt E ⇒ ∠BAM = ∠DAE ABCD hình vng ⇒ ∠ADB= 45° ⇒ ∠ADE = ∠BDE − ∠ADB = 45° ⇒ tam giác ABP tam giác ADE (g.c.g) có AD = AB ⇒ AP = AE , BP = DE Theo giả thiết ∠MAN= 45° ⇒ ∠NAE= 45° ⇒ tam giác QAP tam giác QAE (c.g.c) ⇒ QP = QE Tam giác DEQ tam giác vuông D ⇒ EQ = QD + DE ⇒ QP = QD + BP Cách Lấy H đối xứng với B qua AM H, D đối xứng qua AN, = HP BP = , HQ QD ∠AHP = ∠ABP = 45°, ∠AHQ = ∠ADQ = 45° ⇒ ∠PHQ = ∠PHA + ∠AHQ = 90° ⇒ tam giác PHQ tam giác vuông ⇒ PQ = HP + HQ = PB + QD Bài 10 Cho đa giác ABCDE thỏa mãn AB 90° BD CE cắt ∠AED = = AE , BC = CD = DE ∠ABC = F Chứng minh AB = AF LỜI GIẢI Theo giả thiết: AB = AE , BC = ED, ∠ABC = ∠AED = 90° ⇒ tam giác ABC tam giác AED ⇒ ∠BCA = ∠EDA AC = AD ⇒ tam giác ACD cân A ⇒ ∠ACD = ∠ADC Cũng theo giả thiết: BC = CD = DE ⇒ tam giác BCD CDE tam giác cân, ∠BCD = ∠BCA + ∠ACD = ∠EDA + ∠ADC = ∠EDC Liên hệ tài liệu word toán SĐT (zalo): 039.373.2038 Website: tailieumontoan.com ⇒ tam giác BCD tam giác CDE (c.g.c) ⇒ ∠CBD = ∠CDB = ∠DCE = ∠DEC ⇒ tam giác BCF tam giác EDF (g.c.g) ⇒ BF = EF ⇒ tam giác ABF tam giác AEF (c.c.c) ⇒ ∠AFB = ∠AFC ⇒ ∠AFB= 1 ∠BFE= ∠CFD= (180° − 2∠CBD ) 2 = 90° − ∠CBD = ∠ABF ⇒ tam giác ABF cân A ⇒ AB = AF Bài 11 Cho tam giác ABC Trên cạnh BC, CA, AB lấy điểm D, E, F Qua D dựng đường thẳng d1 vng góc với BC, qua E dựng đường thẳng d2 vng góc với AC, qua F dựng đường thẳng d3 vng góc với AB Chứng minh d1, d2, d3 đồng quy khi: DB + EC + FA2 = DC + EA2 + FB (Định lý Carnot) LỜI GIẢI Thuận: Giả sử d1, d2, d3 đồng quy I Theo định lí Pythagoras ⇒ DB =IB − ID , EC =IC − IE , FA = IA2 − IF Cộng đẳng thức ⇒ DB + EC + FA2 = IB − ID + IC − IE + IA2 − IF = ( IC − ID ) + ( IA2 − IE ) + ( IB − IF ) = DC + EA2 + FB ⇒ DB + EC + FA2 = DC + EA2 + FB Đảo: Giả sử D, E, F thỏa mãn DB + EC + FA2 = DC + EA2 + FB Gọi J giao điểm d1, d2 H hình chiếu J AB ⇒ DB + EC + HA2 = DC + EA2 + HB ⇒ FA2 − FB = HA2 − HB ⇒ H ≡ F Vậy d1, d2, d3 đồng quy Bài 12 Cho hình vng ABCD P điểm hình vng thỏa mãn PA : PB : PC = 1: : Tính góc ∠APB LỜI GIẢI PA PB PC Giả thiết viết dạng = = Trên đường thẳng vng góc với BP B, lấy điểm E cho BE = BP E, C phía AB ⇒ tam giác PBE vng cân B Liên hệ tài liệu word tốn SĐT (zalo): 039.373.2038 Website: tailieumontoan.com PA PB PC Đặt = = = k ⇒ PA = k , PB = 2k , PC = 3k ⇒ PE = BP + BE = 8k Tam giác ABP tam giác CBE có: AB = BC , BP =∠ BE , ABP = ∠CBE ⇒ hai tam giác (c.g.c) ⇒ ∠APB = ∠CEP, PA = EC ⇒ PE + EC = 9k = PC Theo định lí đảo Pythagoras ⇒ ∠PEC = 90° ⇒ ∠APB = ∠CEP = ∠PEC + ∠PEB = 135° Bài 13 Cho tam giác vuông ABC vuông A hai đường phân giác BD, CE Gọi M N hình chiếu A BD CE Tính góc ∠MAN LỜI GIẢI BD CE phân giác ⇒ ∠ABD = ∠DBC = ∠ABC , ∠ACE = ∠ECB = ∠ACB, AM vng góc với BD, AN vng góc CE, AB vng góc BC ⇒ ∠NAC= 90° − ∠ACB, ⇒ ∠MAD= 90° − ∠ADB= ∠ABD ⇒ ∠MAN = ∠NAC − ∠MAD = 90° − ∠ABC − ∠ABC = 90° − ( ∠ABC + ∠ACB )= 45° Bài 14 Cho tam giác ABC vuông A, góc ∠ABC = 40° Trên cạnh AB lấy hai điểm D E cho ∠ACD = ∠BCE = 10° Chứng minh BE = AD LỜI GIẢI Trên tia đối tia AB lấy điểm F cho AF = AD Tam giác ABC vuông A ⇒ tam giác DCF cân C ⇒ ∠ACF = ∠ACD = 10° Theo tính chất góc ngồi tam giác, ta có: CEF = ∠ABC + ∠BCE = 40° + 10° = 50° ⇒ ∠ACE = ∠CAE − ∠AEC = 90° − 50° = 40° ⇒ ∠ECF = ∠ECA + ∠ACF = 50° Liên hệ tài liệu word toán SĐT (zalo): 039.373.2038 Website: tailieumontoan.com ⇒ tam giác FEC cân F ⇒ FE = FC = CD ∠ABC= 40° ⇒ ∠ACB= 50° ⇒ ∠DCB= ∠ACB − ∠ACD= 40° ⇒ tam giác BCD cân D ⇒ DB = DC = EF ⇒ BE = BF − EF = BF − DB = DF = AD Bài 15 Cho tam giác ABC thỏa mãn ∠ABC= 50°, ∠ACB= 30° Điểm D AC cho AD = AB Chứng minh BD = AC LỜI GIẢI Từ giả thiết suy ∠BAC = 180° − ∠ABC − ∠ACB = 100° Gọi E điểm đối xứng A qua cạnh BC ⇒ AE vng góc với BC ∠BAE= 90° − ∠ABC= 40°, CA= CE Mà ∠ACE = 2∠ACB =° 60 ⇒ tam giác ACE ⇒ AE = AC Theo giả thiết: AD = AB ⇒ AB = AD = BE ⇒ tam giác BAD ABE (c.g.c) ⇒ BD = AE ⇒ BD = AC AC ) , ∠BAC = 20° Lấy điểm D AB cho AD = BC Tính Bài 16 Cho tam giác cân ABC ( AB = góc tam giác BDC LỜI GIẢI Dựng tam giác ABE cạnh AB cho E, C phía AB Vì ∠BAC = 20° nên ∠ABC = 80° ∠ACB = ⇒ ∠EBC = ∠ABC − ∠ABE = 80° − 60° = 20° Tam giác ADC tam giác BCE ta có: 20° BC = AD, AD =∠ BE , CAD = ∠EBC = ⇒ hai tam giác Mà AE = AC ⇒ tam giác ACE cân A Hơn nữa, ∠CAE = ∠BAE − ∠BAC = 60° − 20°= 40° ⇒ ∠ACE= 180° − 40° = 70° ⇒ ∠BCE = ∠BCA − ∠ACE = 80° + 70° = 150° ⇒ ∠ADC= 150° ⇒ ∠BDC= 30°, ∠DCB = ∠ACB − ∠DCA = 70°, ∠DBC = 80° Bài 17 Gọi I giao điểm ba đường phân giác tam giác ABC Chứng minh 2∠ACB AB + BI = AC ∠ABC = Liên hệ tài liệu word toán SĐT (zalo): 039.373.2038 Website: tailieumontoan.com LỜI GIẢI Theo giả thiết I giao điểm ba đường phân giác tam giác ABC ⇒ ∠IBC = 1 ∠ABC ∠ICB = ∠ACB 2 Trên tia đối tia BA lấy điểm D cho BI = BD 1 ∠ABI = ∠ABC ⇒ ∠ADI = • Nếu AB + BI = AC ⇒ AB + BD = AD ⇒ tam giác ADC cân A Mặt khác I giao điểm ba đường phân giác ⇒ ∠BAI = ∠IAC ⇒ AI trục đối xứng tam giác ADC ⇒ ∠ADI = ∠ACI ⇒ 1 ∠ABC =∠ACB ⇒ ∠ABC = 2∠ACB Ngược lại ∠ABC = 2∠ACB ⇒ 1 ∠ABC =∠ACB ⇒ ∠ADI = ∠ACI Hai tam giác ADI ACI có AI chung ∠ADI = ∠ACI nên hai tam giác (g.c.g) ⇒ AD = AC Mặt khác BI = BD ⇒ AC = AD = AB + BD = AB + BI Bài 18 Cho tam giác ABC Gọi M, N trung điểm BC, AC BH đường cao Gọi K điểm AC cho MK vng góc với ME phân giác góc ∠HMN Biết = HK ( AB + BC ) ∠HMN =° 45 Chứng minh tam giác ABC cân LỜI GIẢI Theo giả thiết: BH vuông góc với AC ⇒ tam giác BHC tam giác vuông M, N trung điểm BC AC ⇒ HM = 1 = BC , MN AB 2 Ta có HK = ( AB + BC ) = HM + MN Đặt ∠MHN = α Xét vị trí H, N, K 1) N nằm H K Trên tia đối tia MH lấy điểm D cho MD = MN ⇒ HD = MK ⇒ tam giác HDK cân H ⇒ ∠HDK= 90° − α MK vng góc với tia phân giác ∠HMN Liên hệ tài liệu word toán SĐT (zalo): 039.373.2038 Website: tailieumontoan.com ⇒ MK phân giác góc ∠NMD ⇒ ∠NMK = ∠KMD ⇒ tam giác KMD tam giác KMN (c.g.c) ⇒ ∠MNK= 90° − α Mặt khác ∠MNK= 45° + α ⇒ 90° − α= 45° + α ⇒ α= 30° ⇒ ∠HNM= 105° ⇒ ∠MHB= 60° Mà tam giác MHB cân ⇒ ∠HMB= 60° ⇒ ∠BMN= 60° + 45°= 105° ⇒ tam giác CMN cân C ⇒ tam giác ABC cân C ⇒ AC = BC 2) Trường hợp H nằm N K Trên tia đối tia MH lấy điểm D cho MD = MN ⇒ HD = HK ⇒ ∠HKD = ∠HDK = α ⇒ MK trục đối xứng ND ⇒ KD = KN ⇒ tam giác NMK tam giác DMK ⇒ ∠KNM = 1 α ⇒ α + α + 45°= 180° ⇒ α = 90° 2 ⇒ H trùng với C ⇒ tam giác ABC vuông cân C Bài 19 Cho tam giác ABC Gọi D điểm nằm tam giác ABC thỏa mãn DB = DC = AB, ∠ABD = 40° , ∠DBC =° 10 Tính góc ∠DAC LỜI GIẢI Theo giả thiết: BA = BD , ta có tam giác BAD cân B Mà ∠ABD = 40° ⇒ ∠BAD= (180° − 40° )= 70° DB= DC , ∠DBC= 10° ⇒ ∠DCB= 10° Dựng tam giác ABE (E, C phía AB) ⇒ AB= BE ; ∠ABD= 40°, ∠DBC = 10° ⇒ ∠CBE = ∠ABE − ∠ABD − ∠DBC = 60° − 40° − 10° = 10° Suy ∠CBE < ∠CBD Mà BE = AB = BD nên E đối xứng với D qua BC Mà DB = DC nên BDCE hình thoi ⇒ EC = EB = EA, ∠BEC = 180° − ∠DBE = 160° ⇒ ∠AEC = ∠BEC − ∠BEA = 100° ⇒ ∠EAC = 40° Mà ∠EAD = ∠BAD − ∠BAE = 10° ⇒ ∠DAC = 30° Bài 20 Cho tam giác cân ABC ( AB =AC , ∠BAC =110° ) P điểm nằm tam giác thỏa mãn ∠PBC= 25°, ∠PCB= 30° Tính góc ∠APC LỜI GIẢI Liên hệ tài liệu word toán SĐT (zalo): 039.373.2038 10 Website: tailieumontoan.com Tam gác ABC có AB = AC , ∠BAC =110° 180° − ∠BAC = 35° ⇒ ∠ABC = ∠ACB = Dựng tam giác DBC (D, A phía BC) ⇒ DA trục đối xứng BC ⇒ ∠BDA = 30° ⇒ ∠DBA = ∠DBC − ∠ABC = 60° − 35° = 25° Tam giác cân DBA CBP có: DB =∠ BC , DBA = ∠CBP = 25°, ∠BDA = ∠BCP = 30° ⇒ hai tam giác (c.g.c) ⇒ BA = BP ⇒ tam giác ABP cân B Mà ∠ABP = ∠ABC − ∠PBC = 10° ⇒ ∠BAP = ∠BPA = 180° − ∠ABP = 85° ⇒ ∠PAC = ∠BAC − ∠BAP = 110° − 85° = 25° Mà ∠ACP = 35° − 30° = 5° ∠ACB − ∠PCB = ⇒ ∠APC = 180° − 25° − 5= ° 150° Bài 21 Cho tam giác ABC ( AB = AC , ∠BAC = 20° ) Trên cạnh AC lấy điểm D, cạnh AB lấy điểm E cho ∠DBA = 30° ∠ECA = 20° Tính góc ∠DEC LỜI GIẢI Tam giác ABC có AB = 20° AC , ∠BAC = ⇒ ∠ABC = ∠ACB = 180° − ∠BAC = 80° Dựng tam giác BCI phía tam giác ABC ⇒ ∠IBC = ∠ICB = 60° ⇒ ∠ICA = ∠ACB − ICB = 80° − 60° = 20° Theo giả thiết ∠ECA = 20° ⇒ C, I, E thẳng hàng BI cắt AC F ⇒ ∠ECB = ∠FBC = 60° ⇒ EF song song với BC ⇒ tam giác IEF Theo giả thiết ∠DBA = 30° ⇒ ∠DBC = ∠ABC − ∠DBA = 80° − 30° = 50° ⇒ ∠CDB= 180° − ∠DBC − ∠DCB= 50° ⇒ tam giác CBD cân C ⇒ CB = CD ⇒ CD = CI ⇒ tam giác CDI cân C ⇒ ∠CID = ∠CDI = (180° − ∠DCI ) = 80° Liên hệ tài liệu word toán SĐT (zalo): 039.373.2038 11 Website: tailieumontoan.com ⇒ ∠DIF = 180° − ∠BID = 180° − 60° − 80= ° 40° Mặt khác ∠BFC = ∠CDI − ∠DIF = 80° − 40° = 40° ⇒ tam giác DIF cân D ⇒ DF =DI ⇒ tam giác DEF tam giác DEI (c.c.c) ⇒ ∠DEC = ∠DEI = ∠DEF = ∠FEC = 30° Bài 22 Cho tam giác ABC có ∠ABC = 60° Gọi M N trung điểm AB, AC đồng thời thỏa mãn ∠NBC = 45° Tính góc ∠MCB LỜI GIẢI Kéo dài MN hai phía lấy K E cho MK = MN = NE Theo giả thiết CM: trung tuyến ứng với cạnh AB ⇒ MA = MB ⇒ tứ giác AKBN hình bình hành ⇒ ∠KAB = ∠ABN = ∠ABC − ∠NBC = 15° AK song song BN, KN song song BC ⇒ ∠AKN= 45° Trung trực AK cắt AB P ⇒ tam giác PAK cân P ⇒ ∠AKP= 15° ⇒ ∠KPM = 30° Mà ∠PKN = ∠AKN − ∠AKP = 30° ⇒ tam giác MKP cân M ⇒ MK= MN= MP ⇒ ∠KPN= 90° ⇒ ∠PNK= 60° ⇒ tam giác MPN ⇒ PM = MN = NP ⇒ ∠MPE= 90°, PE= PK= PA ⇒ tam giác APE vuông cân P ⇒ ∠AEP = 45° ⇒ ∠AEM = ∠AEP + ∠PEK = 75° Tứ giác AECM hình bình hành ⇒ AE song song với MC ⇒ ∠MCB = ∠CME = ∠AEM = 75° Bài 23 Cho tam giác ABC M điểm nằm tam giác cho tam giác MAC tam giác cân có góc ∠AMC = 120° D điểm bên tam giác cho tam giác BCD tam giác I trung điểm AD Tính góc tam giác BMI LỜI GIẢI Trên tia đối tia MA lấy điểm N cho AM = MN Liên hệ tài liệu word toán SĐT (zalo): 039.373.2038 12 Website: tailieumontoan.com Theo giả thiết ∠AMC = 120°, AM =MC ⇒ ∠CMN= 60° ⇒ tam giác MCN ⇒ ∠ACN= ∠ACM + ∠MCN= 30° + 60°= 90° Ta có: ∠MCB = ∠MCN − ∠BCN = ∠BCD − ∠BCN = ∠NCB BCD tam giác ⇒ BC = CD ⇒ tam giác MCB NCD (c.g.c) ⇒ MB = ND Mặt khác, M trung điểm AN, I trung điểm AD ⇒ MI đường trung bình tam giác AND ⇒ MI= 1 ND= BM 2 AM cắt CD K ⇒ ∠IMK = ∠DNK (đồng vị), 120° = ∠KNC = ∠DNC − ∠DNK = ∠BMC − ∠IMK = ∠BMI + ∠KMC = ∠BMI + 60° ⇒ ∠BMI= 60°, MI= BM ⇒ ∠MBI= 30° ⇒ ∠BIM= 90° Bài 24 Cho tam giác ABC có ∠BAC = 40° 40°, ∠ABC = 60° Điểm D cạnh AC cho ∠DBC = Đường cao AH cắt BD E, CE cắt AB I Tính góc ∠BIC LỜI GIẢI Theo giả thiết ∠BAC = 40°, ∠ABC = 60° ⇒ ∠ACB= 180° − ∠BAC − ∠ABC= 80° Dựng tam giác ABK cho K, C phía AB Giả thiết AH vng góc với BC ⇒ HB = HK , ∠CAK = ∠BAK − ∠BAC = 60° − 40° = 20° BD cắt AK F HB = HK ⇒ EB = EK ⇒ ∠EBK = ∠EKB = 40° ⇒ ∠AKE =∠AKB − ∠EKB =20°, ∠BEK =180° − 2∠EBK =100° ⇒ ∠FEK= 180° − ∠BEK= 80° ⇒ ∠KFE= 80°= ∠KEF ⇒ tam giác KEF cân K ⇒ KF = KE (*) Tam giác ABC tam giác KBF có: AB =∠ BK , ABC = 60°, ∠BAC = 40° ∠BKF = ∠KBF = ⇒ hai tam giác (g.c.g) ⇒ BC = KF Kết hợp (*) ⇒ BC = BE 180° − 40° ⇒ tam giác BCE cân B ⇒ ∠BEC = 70° ∠BCE = = ⇒ ∠BIC = 180° − ∠IBC − ∠ICB = 180° − 60° − 70= ° 50° Bài 25 Cho tam giác ABC Điểm M nằm tam giác thỏa mãn ∠MBC= 30°, ∠MCB= 40°, Liên hệ tài liệu word toán SĐT (zalo): 039.373.2038 13 Website: tailieumontoan.com 10 Tính góc tamm giác ABC 20°, ∠MCA =° ∠MAC = LỜI GIẢI Cách 1: Gọi D điểm đối xứng C qua AM AM cắt CD H ⇒ ∠CMH = ∠MAC + ∠MCA = 30° ⇒ ∠CMD= 60° ⇒ tam giác MCD ⇒ ∠MCD= 60° Mà ∠MBC =° 40° 30 , ∠MCB = ⇒ ∠BMC = 180° − ∠MBA − ∠MCB = 110° ⇒ ∠BMD = ∠BMC − ∠DMC = 110° − 60° = 50° D C đối xứng qua AM ⇒ ∠ADM = 10° ⇒ ∠AEM = ∠BMD + ∠ADM = 60° ⇒ ∠AEM = ∠MCD = 60° ⇒ tứ giác MCDE nội tiếp ⇒ ∠DEC = ∠DMC = 60° ⇒ ∠BED = ∠CED = 60° ⇒ ∠ECB = 180° − ∠BEC − ∠EBC = 30° = ∠EBC ⇒ AD vng góc với BC ⇒ AD trục đối xứng cạnh BC ⇒ ∠ABC = ∠ACB = ∠ACM + ∠MCB = 50° ⇒ ∠BAC = 80° Cách Dựng tam giác cạnh MCD (D, B phía MC) ⇒ ∠DMC= 60° MD = MC Từ giả thiết ∠MAC = 20°, ∠MCA= 10° ⇒ ∠AMC = 180° − ∠MAC − ∠MCA = 150° ⇒ ∠AMD = 360° − ∠AMC − ∠CMD = 150° ⇒ tam giác AMC tam giác AMD (c.g.c) ⇒ AD = AC Từ chứng minh AD trục đối xứng BC ⇒ ∠ABC = ∠ACB = 50°, ∠BAC = 80° Bài 26 Cho tam giác ABC có phân giác AD BE thỏa mãn ∠EDC ∠DEC Chứng minh tam giác = ∠ADE ∠BED ABC cân LỜI GIẢI ∠EDC ∠DEC Đặt = = k , với k số thực dương ∠ADE ∠BED ⇒ ∠EDC = k ∠ADE , ∠DEC = k ∠BED Liên hệ tài liệu word toán SĐT (zalo): 039.373.2038 14 Website: tailieumontoan.com Gọi I giao điểm AD BE ⇒ CI phân giác ∠ACB CI cắt cạnh AM M Theo tính chất góc ngồi tam giác ⇒ ∠EIC = ∠IBC + ∠ICB, ∠DIC = ∠IAC + ∠ICA ⇒ ∠EID = ∠EIC + ∠DIC = ∠ACB + ⇒ ∠ACB ( ∠ABC + ∠CAB ) = 90° + 2 ∠ACB = ∠EID − 90° = 90° − ∠IED − ∠IDE (1) Mặt khác ∠ACB = 180° − ∠EDC − ∠DEC = 180° − k ∠ADE − k ∠BED (2) Từ (1) (2) ⇒ 180° − 2∠IED − 2∠IDE = 180° − k ∠ADE − k ∠BED ⇒ ( ∠ADE + ∠BED )( − k ) = ⇒ k = Gọi EJ phân giác góc ∠DEC ⇒ DJ phân giác góc ∠EDC Từ k = ⇒ ∠EDC = 2∠ADE , ∠DEC = 2∠BED ⇒ ∠BED = ∠DEJ , ∠ADE = ∠EDC ⇒ tam giác IED tam giác JED (g.c.g) ⇒ ED vng góc với IJ ⇒ tam giác CDE cân C ⇒ CD = CE ⇒ ∠CED = ∠CDE ⇒ ∠CEI = ∠CDI ⇒ ∠CIA = ∠CIB ⇒ tam giác ICA tam giác ICB (g.c.g) ⇒ CA = CB ⇒ tam giác ABC cân C Bài 27 Cho tứ giác ABCD, có ∠A =∠B =90° BC < AD , đồng thời thỏa mãn AB = AD, CD = BC + AD Gọi E trung điểm AD Chứng minh ∠ADC = 2∠ABE LỜI GIẢI Gọi M trung điểm cạnh AB Kéo dài CM cắt AD N ⇒ tam giác MBC MAN hai tam giác (g.c.g) ⇒ AN = BC MN = MC Mặt khác theo giả thiết CD = BC + AD ⇒ DN = DA + AN = DA + BC = DC ⇒ tam giác DCN cân D ⇒ DM vng góc với CN ∠CDM = ∠MDN E trung điểm AD ⇒ EA = ED Kết hợp AB = AD ⇒ AM = AE Tam giác BAE DAM hai tam giác (c.g.c) ⇒ ∠ABE = ∠ADM ⇒ ∠ADC = 2∠ABE Liên hệ tài liệu word toán SĐT (zalo): 039.373.2038 15 Website: tailieumontoan.com Bài 28 Cho tam giác ABC Dựng phía ngồi tam giác ABC tam giác ABD ACE vuông cân đỉnh D E Gọi M trung điểm BC Chứng minnh MDE tam giác vuông cân LỜI GIẢI Gọi I, J trung điểm AB, AC Theo giả thiết tam giác ADB tam giác AEC vuông cân ⇒ DI vng góc AB EJ vng góc AC ⇒ DI = IA = IB, EJ = JA = JC Theo tính chất đường trung bình, ta có MI song song AC, MJ song song AB= DI MJ = , MI EJ Do tính chất song song ⇒ ∠BIM = ∠BAC = ∠MJC ⇒ ∠DIM = ∠DIB + ∠BIM = 90° + ∠BJC = ∠MJE ⇒ tam giác DIM tam giác MJE (c.g.c) ME , ∠IDM = ⇒ MD = ∠JME , ∠IMD = ∠JEM Ta có ∠IDM + ∠IMD + ∠DIM= 180° ⇒ ∠JME + ∠IMD + ( ∠DIB + ∠BIM= ) 180° ⇒ ∠JME + ∠IMD + ∠IMJ= 90° ⇒ ∠DME= 90° ⇒ tam giác MDE vuông cân M Bài 29 Cho tứ giác ABCD Dựng phía ngồi tứ giác tam giác vuông cân ABP, BCM, CDQ, DAN tương ứng đỉnh P, M, Q, N Chứng minh PQ MN PQ vng góc với MN LỜI GIẢI Gọi I trung điểm AC Theo kết toán ⇒ IP =IM , ∠PIM =90° IQ = IN , ∠QIN = 90° Xét hai tam giác PQI tam giác MIN, ta có: IP = IM , IQ = IN , ∠PIQ = ∠PIN + ∠NIQ = ∠PIN + ∠PIM = ∠NIM ⇒ hai tam giác (c.g.c) ⇒ PQ = MN ∠PQI = ∠MNI ⇒ ∠PQI + ∠MNQ= 45° − ∠PQI + 45° + ∠MNI= 90° ⇒ PQ vng góc với MN Bài 30 Cho tam giác ABC P điểm tùy ý cạnh BC Qua P kẻ đường thẳng song song với AB AC cắt AC D AB E Từ B kẻ đường thẳng song song với DE cắt PD N Chứng minh AN qua điểm cố định P thay đổi cạnh BC LỜI GIẢI Liên hệ tài liệu word toán SĐT (zalo): 039.373.2038 16 Website: tailieumontoan.com Gọi giao điểm AN với BC M Từ C kẻ đường thẳng song song DE cắt AN Q PE song song với AC ⇒ tam giác DQC tam giác QDE (g.c.g) ⇒ CQ = DE Mặt khác, hai tam giác ENB NED (g.c.g) ⇒ BN = DE ⇒ CQ = BN ⇒ tam giác BNM tam giác CQM (g.c.g) ⇒ MB = MC ⇒ M trung điểm BC ⇒ AN qua trung điểm BC ⇒ AN qua điểm cố định P thay đổi cạnh BC Bài 31 Cho tam giác ABC ( AB > AC ) P điểm tia đối tia AM thỏa mãn PA + PC = AB M trung điểm BC Từ C kẻ đường thẳng vng góc với AM, cắt AM Q Chứng minh BQ = AP LỜI GIẢI Trên tia đối tia BA lấy điểm D cho AD = AB Theo giả thiết PA + PC = AB ⇒ PA + PC = AD ⇒ PC = AD − PA = PD ⇒ tam giác PCD cân P Gọi N trung điểm CD ⇒ PN vng góc với CD MN đường trung bình tam giác BCD ⇒ MN song song với CD Theo giả thiết AM vng góc với CQ, PN vng góc với CD ⇒ PN song song với CQ ⇒ PN đường trung bình tam giác CDQ ⇒ PD = PQ ⇒ BQ = AB − AQ = AD − ( PQ − AP ) = AD − PQ + AP = AD − PD + AP = AP Bài 32 Cho tam giác nhọn ABC Xác định điểm M, N, P cạnh tam giác ABC để chu vi tam giác MNP nhỏ LỜI GIẢI Giả sử M điểm cạnh BC để chu vi tam giác MNP có hai đỉnh hai cạnh tam giác ABC có chu vi nhỏ Gọi D E điểm đối xứng M qua cạnh AB AC Theo tính chất đối xứng MD vng góc AB AD = AM Từ suy PD = PM Liên hệ tài liệu word toán SĐT (zalo): 039.373.2038 17 Website: tailieumontoan.com Tương= tự AE AM , NE NM = ⇒ AD = AE Ta có P = MN + NP + PM ≤ DE ⇒ chu vi tam giác MNP không lớn đường thẳng DE Tam giác ADE tam giác cân, cạnh bên AM, theo tính chất đối xứng, ta có ∠DAB = ∠BAM , ∠EAC = ∠CAM ⇒ ∠DAE = ∠DAM + ∠MAE = 2∠BAC DE cạnh đáy tam giác ADE, ∠DAE = 2∠BAC : không đổi Vậy DE nhỏ cạnh bên nhỏ nhất, nghĩa AM nhỏ M hình chiếu A cạnh BC Tam giác MNP có chu vi nhỏ M, N, P chân đường cao kẻ từ đỉnh ABC xuống cạnh đối diện Tam giác cịn có tên tam giác trực tâm Nhận xét: Đây phương pháp Descartes xây dựng để giải toán Liên hệ tài liệu word toán SĐT (zalo): 039.373.2038 18 Website: tailieumontoan.com II ĐỊNH LÝ THALES VÀ TAM GIÁC ĐỒNG DẠNG Định lý thuận: Nhiều đường thẳng song song cắt hai cát tuyến (đường thẳng) d, d’ tạo d, d’ đoạn thẳng tương ứng tỷ lệ: a A, d ∩ = b B, d ∩ = c C d ∩ = a A′, d ′ ∩= b B ′, d ′ ∩= c C′ d ′ ∩= a || b || c ⇒ AB BC CA = = A′B ′ B ′C ′ C ′A′ Hệ thức viết dạng hệ thức: AB A′B ′ AB A′B ′ BC B ′C ′ = = = ; ; BC B ′C ′ CA C ′A′ CA C ′A′ * Chứng mính: Từ A kẻ đường thẳng song song với d’ cắt b c B” C’’ => tứ giác AA’B’B”, AA’C’C” hình ′B ′, AC ′′ A′C ′ bình hành= => AB ′′ A= Theo tính chất tỷ số diện tích hai tam giác, ta có S ABB′′ AB S ABB′′ AB ′′ = = , S ACB′′ AC S ABC ′′ AC ′′ Mặt khác BB’ // CC’ ⇒ S BCB′′ = S BC ′′B′′ ⇒ S ACB′′ = S ABC ′′ ⇒ AB AB ′′ AB A′B ′ = ⇒ = AC AC ′′ AC A′C ′ Cách chứng minh đời vào thời Euclid Định lí Thales đảo: Ba đường thẳng a, b, c a song song với b, cắt hai đường thẳng d, d’, định d, d’ đoạn thẳng tương ứng tỉ lệ c song song với a AB BC CA Nghĩa là: a // b, = = a // c A′B ′ B ′C ′ C ′A′ * Chứng mình: Liên hệ tài liệu word toán SĐT (zalo): 039.373.2038 Website: tailieumontoan.com Giả sử c không song song với a Từ C kẻ đường thẳng c’ song song với a cắt d’ C” Theo định lí Thales thuận => AB BC = A′B ′ B ′C ′′ Giả thiết AB BC , từ ta suy = A′B ′ B ′C ′ BC BC = ⇒ B ′C ′ = B ′C ′′ ⇒ C ′ ≡ C ′′ ⇒ c // a B ′C ′ B ′C ′′ Định lí đường thẳng đồng quy cắt hai đường thẳng song song Nếu ba đường thẳng a, b, c đồng quy O cắt hai đường thẳng d, d’ song song với khơng qua O chúng định d, d’ ba đoạn thẳng tỉ lệ: Nếu a, b, c đồng quy O d // d’ OA OB OC AB BC CA = = = = OA′ OB ′ OC ′ A′B ′ B ′C ′ C ′A′ (Bạn đọc áp dụng định lí Thales để tự chứng minh) Bài Cho tam giác ABC, O điểm nằm tam giác AO, BO, CO cắt cạnh BC, CA, AB D, E, F Qua O kẻ đường thẳng song song với BC cắt DE, DF N M Chứng minh OM = ON LỜI GIẢI Qua A kẻ đường thẳng song song với BC cắt BO, CO, DE, DF P, Q, I, H Theo định lí Thales ta có: Liên hệ tài liệu word toán SĐT (zalo): 039.373.2038 Website: tailieumontoan.com AQ AH BC DC = = , BC BD AP AI Nhân hai đẳng thức ta thu được: AQ AH DC = AP BD AI Cũng theo định lí Thales ta có: AQ DC AQ DC AH DC AH = ⇒ = = ⇒ =1 AP DB AP DB BD AI AI ⇒ AH = AI Theo định lí Thales OM DO DN = = ⇒ OM = ON AH DA AI Bài Cho tam giác nhọn ABC, đường cao BD CE, P hình chiếu D AB, Q hình chiếu P BC, PQ cắt BD K Chứng minh EK song song AC LỜI GIẢI Gọi H trực tâm tam giác ABC Vì AH vng góc với BC PQ vng góc với BC => AH song song với PQ Vì DP vng góc với AB => DP song song CE ⇒ BK BP BH BE =và = BH BA BD BP BK BK BH BP BE BE Từ ta có đẳng thức: = = = BD BH BD BA BP BA Theo định lí Thales đảo ta có EK song song với AC Bài Cho tam giác ABC, đường cao AH trung tuyến CM, N trung điểm AM, D trung điểm BC thỏa mãn ND vng góc với BC Chứng minh ba trung tuyến tam giác AMC độ dài ba cạnh tam giác BND LỜI GIẢI Liên hệ tài liệu word toán SĐT (zalo): 039.373.2038 Website: tailieumontoan.com Gọi K hình chiếu M cạnh BC AH CM giao E AH vng góc với BC => AH, ND, MK song song với CM trung tuyến => MA = MB ⇒ KB = KH N trung điểm AM ⇒ DK = DH Gọi I giao điểm ND CM ⇒ IM =IE ⇒ tam giác IMN tam giác IED (c.g.c) ⇒ ∠IMN = ∠IED => DE song song AB ⇒ EM = EC => AE trung tuyến tam giác AMB, DM song song AN, AE song song ND => AEDN hình bình hành => AE = ND Gọi J trung điểm AC => MJ trung tuyến tam giác AMC => MJ = BD , tam giác BNC tam giác cân ⇒ CN = BN , AE = ND => trung tuyến tam giác AMC độ dài cạnh tam giác BND Bài Cho tứ giác ABCD thỏa mãn ∠ABC = ∠ACD ∠BAC = ∠CAD Hình chiếu A BC P CD Q Chứng minh trực tâm tam giác APQ nằm BD LỜI GIẢI Theo giả thiết ∠ABC = ∠ACD ∠BAC = ∠CAD => tam giác ABC tam giác ACD đồng dạng (g.g) Mà AP AQ đường cao tương ứng, suy BP CQ = PC QD (1) Từ P kẻ đường thẳng song song với CD cắt BD H ⇒ Từ (1) (2) ⇒ BP BH = (2) PC HD BH CQ = HD QD Theo định lí đảo Thales ta thu HQ song song BC Kết hợp giả thiết AP vng góc BC => HQ vng góc với AP Mà HP vng góc với AQ (vì HP song song với CD CD vng góc với AQ) => H trực tâm tam giác APQ Bài Cho tam giác ABC, O điểm nằm tam giác AO, BO, CO cắt cạnh BC, CA, AB D, E, F Chứng minh DB EC FA = (Định lý Ceva) DC EA FB LỜI GIẢI Liên hệ tài liệu word toán SĐT (zalo): 039.373.2038 Website: tailieumontoan.com Từ A kẻ đường thẳng song song với BC cắt BO, CO kéo dài P Q Theo định lí Thales ta có DB AP EC BC FA AQ = = , = , DC AQ EA AP FB BC Nhân ba đẳng thức với ta được: DB EC FA AP BC AQ = = DC EA FB AQ AP BC * Định lí Ceva tổng quát: Ba đường thẳng qua đỉnh A, B, C tam giác ABC đồng quy cắt cạnh BC, CA, AB hay đường kéo dài D, E, F Chứng minh DB EC FA = DC EA FB Bài Trên đường trung tuyến AD tam giác ABC lấy điểm E Đường thẳng BE cắt AC M đường thẳng CE cắt AB N Chứng minh MN song song với BC LỜI GIẢI Từ E (hoặc từ A) dựng đường thẳng song song với BC cắt AB, AC P Q Theo định lí Thales: PE BD = = ⇒ PE = EQ EQ DC Mà NP PE EQ MQ NP MQ = = ⇒ = NB BD DC MC NB MC => MN song song BC * Nhận xét: Trong hình thang, trung điểm hai cạnh đáy, giao điểm hai đường chéo, giao điểm hai đường thẳng chứa hai cạnh bên nằm đường thẳng Bài Cho tam giác ABC, trung tuyến AD, phân giác AE (D, E thuộc cạnh BC) Qua C kẻ đường thẳng vng góc với AE cắt AE, AD P Q Chứng minh DP song song với AB QE song song với AC Gọi O giao điểm DP QE, từ suy OE = OQ = PO LỜI GIẢI Liên hệ tài liệu word toán SĐT (zalo): 039.373.2038 Website: tailieumontoan.com Kéo dài DP cắt AC M CP cắt AB N Theo giả thiết CN vng góc với AE ∠BAE = ∠CAE => tam giác CAN cân A PC = PN Theo định lí Thales ta có DM song song với AB PD = AQ AN AN , NB, = = QD DP NB CE DC − DE DC BD BE − DE 1 = = −= −= −1 DE DE DE DE DE = ( BN + NA ) AN BE AB CE AQ − 2= − 2= − 2= ⇒ = DE DP BN NB DE QD => QE song song với AC Bài Cho tam giác ABC = , CA b= , AB c ) Trung tuyến AD, đường cao BH phân giác CE ( BC a= đồng quy O Chứng minh đẳng thức: ( a + b ) ( a + b − c ) = 2ab LỜI GIẢI * Cách 1: Theo tốn ta có HE song song với BC Áp dụng định lí Thales tính chất đường phân giác tam giác, ta có: HA EA AC AH AC = = ⇒ = HC EB BC HC + AH BC + AC AC ⇒ AH = BC + AC (1) Áp dụng định lý Pythagoras ta được: AH = AB − BH = AB − ( BC − CH ) = AB − BC + CH = AB − BC + ( AC − AH ) = AB − BC + AC + AH − AC AH AB − BC + AC (2) ⇒ AH = AC Thay (2) vào (1) ⇒ 2AC = ( BC + AC ) ( AB + AC − BC ) ⇒ 2b3 = ( a + b ) ( b + c − a ) ⇒ 2b3 = ( a + b ) b + ( a + b ) ( c − a ) ⇒ b3 + 2ab= 2ab + ( a + b ) ( c − a ) Liên hệ tài liệu word toán SĐT (zalo): 039.373.2038 Website: tailieumontoan.com ⇒ ( a + b ) ( a + b2 − c2 ) = 2ab * Cách 2: Xét tam giác vuông BHC: CH = BC − BH = BC − ( AB − AH ) = BC − AB + AH = BC − AB + ( CA − CH ) ⇒ BC + CA2 − AB = 2CA.CH ⇒ CH = Tương tự: AH = BC + CA2 − AB 2CA CA2 + AB − BC CH BC + CA2 − AB = ⇒ AH CA2 + AB − BC 2CA CO đường phân giác ΔADC ⇒ OD CD BC = = OA CA 2CA (1) (2) HC Từ D kẻ DK ⊥ AC ( k ∈ AC ) ⇒ BH // DK ⇒ HK = ⇒ OD HK CH = = OA HA HA Từ (1), (2) (3) ⇒ (3) BC + CA2 − AB BC = CA2 + AB − BC CA ⇒ BC CA + CA3 − AB CA = CA2 BC + AB BC − BC ⇒ ( BC + CA3 ) + BC CA + CA2 BC − AB CA − AB BC = BC.CA2 ⇒ ( BC + CA ) ( BC + CA2 − AB ) = BC.CA2 ⇒ ( a + b ) ( a + b2 − c2 ) = 2ab Bài Cho tam giác ABC nhọn, đường trung tuyến AD, đường cao BH đường phân giác CE cắt O Chứng minh HE song song với BC sin B = tan C cos A LỜI GIẢI Theo tốn 6, ta có HE song song BC Theo định lí Thales tính chất phân giác ⇒ CH BE BE BC CH BC , = = ⇒ = AH AE AE AC AH AC Mà CH BC = = cos C , AH AB cos A ⇒ BC cos C BC cos C AB = ⇒ = AB cos A AC cos A AC Liên hệ tài liệu word toán SĐT (zalo): 039.373.2038 Website: tailieumontoan.com Mặt khác AB sin C cos C sinC sin B = ⇒ = ⇒ = tan C AC sin B cos A sin B cos A Bài 10 Cho tam giác ABC vuông A, D điểm AB, H hình chiếu D BC điểm E nằm AC cho DE = DH Gọi I trung điểm HE Chứng minh ∠BEH = ∠HCI LỜI GIẢI Theo giả thiết DE = DH => tam giác DHE cân D Cũng từ giả thiết IH = IE => DI vng góc với HE Gọi K hình chiếu E BC Ta có ∠BDH = 90° − ∠ABC = ∠ECK => tam giác DHB tam giác CKE đồng dạng (g.g) ⇒ BH DH DH EK = ⇒ BH = EK CK CK (1) Ta có ∠DHE = ∠HEK (so le trong) => tam giác IHD tam giác KEH đồng dạng (g.g) ⇒ HI DH DH EK = ⇒ HE = EK HE HI (2) Chia hai đẳng thức (1) (2) ta ⇒ BH DH EK HI HI IK = = = HE CK DH EK CK KC (3) Mà ∠IHK = ∠IKC ⇒ BHE = ∠IKC Kết hợp (3) suy tam giác BHE IKC đồng dạng (c.g.c) ⇒ ∠BEH = ∠HCI Bài 11 Cho tam giác ABC,= BC 8,= CA 7,= AB Trên cạnh AC lấy điểm D cho CD = AB , tia BA lấy điểm E cho BE = CA Đường thẳng ED cắt BC F Tính độ dài BF LỜI GIẢI Qua A kẻ đường thẳng song song với EF cắt BC I BI AB IF AD Theo định lí Thales ta có = = , = = IF AE FC DC ⇒ BI + IF 5+2 = BI + IF + FC + + ⇒ BF = BC 12 ⇒ BF = 14 = 12 Liên hệ tài liệu word toán SĐT (zalo): 039.373.2038 Website: tailieumontoan.com Bài 12 Cho tam giác ABC, M điểm cạnh BC Qua B, C dựng đường thẳng song song với AM cắt AC, AB P, Q Chứng minh 1 = + AM PB QC LỜI GIẢI Giả thiết AM, PB, QC song song với Theo định lý Thales, ta có AM CM AM BM = = PB CB QC QC Cộng hai vế ta có: AM AM CM BM BM + MC BC + = + = = =1 PB QC CB BC BC BC ⇒ 1 = + AM PB QC Bài 13 Cho tam giác ABC ( AB = AC ) Trên tia đối tia BC lấy điểm M Đường thẳng d qua M cắt cạnh AB, AC P Q Chứng minh BM CM không phụ thuộc vào vị trí M đường thẳng d − BP CQ LỜI GIẢI Qua A dựng đường thẳng song song với d cắt BC N Theo định lí Thales ta có Trừ hai vế ta có Từ suy BM BN CM CQ = = BP BA CN CA BM CM BN CQ BC − = − = BP CN BA CA AB BM CM khơng phụ thuộc vị trí M − BP CQ đường thẳng d Bài 14 Cho tứ giác ABCD, O giao điểm hai đường chéo AC, BD Gọi M, N trung điểm BD AC, H điểm đối xứng O qua MN Đường thẳng qua H song song với MN cắt AD, BC, BD, AC P, Q, E, F Chứng minh PE = QF LỜI GIẢI Liên hệ tài liệu word toán SĐT (zalo): 039.373.2038 Website: tailieumontoan.com Theo giả thiết H đối xứng với O qua MN ME Mà MB = MD ⇒ OM = ⇒ OB = ED Tương tự FA = OA Qua O kẻ IJ // MN Theo định lí Thales ta có: PE DE OB OB.OI = = ⇒ PE = OI DO DO DO Tương tự QF = ⇒ OA.OJ MN cắt AD BC thứ tự K G CO KM DM BD KN AN AC = = , = = IO DO DO IO AO AO MN AC BD Trừ hai đẳng thức ta được= − IO AO DO MN BD AC Tương tự:= − JO BO CO Ta có PE = QF ⇔ OB.OI OA.OJ OB.MN OA.MN = ⇔ = DO CO DO.OJ CO.OI ⇔ OB BD AC OA AC BD BD OB AC AC OA.BD − − − = − = ⇔ DO BO CO CO AO DO DO DO.CO CO CO.DO ⇔ BD OA.BD AC OB AC + = + DO CO.DO CO DO.CO Hai vế cho kết AC.BD DO.CO Bài 15 Cho tam giác ABC Qua đỉnh A bờ AB kẻ tia Ax tia Ay thỏa mãn Ax // BC tia Ax nằm góc ∠CAy Từ C kẻ đường thẳng d cắt Ax tai D Ay E Đường thẳng BD cắt AC F Chứng minh đường thẳng EF qua điểm cố định không phụ thuộc đường thẳng d LỜI GIẢI Kéo dài tia Ay cắt BC P EF cắt AD M BC N Theo giả thiết Ax // BC, theo định lí Thaless ta có MA NP MA NC = = MD NC MD NB ⇒ NP NC NP NC = ⇒ = NC NB NP + NC NC + NB ⇒ NP NC CP − CN NC = ⇒ = CP BC CP BC Liên hệ tài liệu word toán SĐT (zalo): 039.373.2038 10 Website: tailieumontoan.com ⇒ NC NC + = + ⇒ NC = BC CP BC CP BC.CP ⇒ CN = BC + CP Tia Ay cố định => P cố định => CP khơng đổi => biểu thức BC.CP có giá trị không đổi => CN không đổi BC + CP Vậy đường thẳng d qua N cố định Bài 16 Cho tam giác ABC, M điểm cạnh BC Chứng minh MA.BC < MB.CA + MC AB LỜI GIẢI Từ M kẻ đường thẳng song song với AB, AC cắt cạnh AC, AB D E => tứ giác AEMD hình bình hành ⇒ MD = AE Theo định lí Thales ta có Tương tự MC = MB ME BC.ME = ⇒ MB = BC CA CA BC.MD AB Thay MB MC vào biểu thức MB.CA + MC AB ta suy ra: MB.CA + MC AB = BC.ME BC.MD CA + AB CA AB = BC ( ME + MD= ) BC ( ME + AE ) Mà ME + AE > MA nên MB.CA + MC AB > BC.MA Bài 17 Cho tam giác ABC, ba điểm M, N, P nằm đường thẳng chứa cạnh BC, CA, AB Chứng minh M, N, P thẳng hàng MB NC PA = (Định lí Menelaus) MC NA PB LỜI GIẢI Giả sử M, N, P thẳng hàng Từ A kẻ đường thẳng song song với BC cắt MN D Theo định lí Thales ta có NA AD PB BM ⇒ = ,= NC MC PA AD Nhân hai đẳng thức với ta NA PB AD BM MB NC PA = ⇒ = NC PA MC AD MC NA PB Ngược lại lấy điểm E BC cho M, N, E thẳng hàng Liên hệ tài liệu word toán SĐT (zalo): 039.373.2038 11 Website: tailieumontoan.com ⇒ MB NC EA MB NC PA PA EA =1 ⇒ = ⇒P≡E = Kết hợp với MC NB PB PB EB MC NB EB => M, N, P thẳng hàng Bài 18 Cho tứ giác ABCD I, J trung điểm AD BC Gọi G, E trọng tâm tam giác ABC ABD Chứng minh DG, CE, IJ đồng quy Từ suy GE song song với CD LỜI GIẢI G trọng tâm ΔABC ⇒ AG = 2GJ Áp dụng định lí Menelaus với ΔAIJ , đường thẳng DG cắt IJ M ⇒ DA MI GJ = DI MJ GA Mà IA = ID ⇒ DA = DI ⇒ MI MJ = ⇒ MI = MJ Tương tự, CE qua trung điểm IJ => ba đường thẳng DG, CE, IJ đồng quy Áp dụng định lý Menelaus cho ΔADG , với ba điểm M, I, J thẳng hàng ta IA MD JG MD MD 1⇒ = 1⇒ = = ID MG JA MG MG Tương tự MC MD MC 3⇒ = = ME MG ME Theo định lí Thales ta thu EG song song với CD Bài 19 Cho tam giác ABC, gọi I trung điểm BC Qua I kẻ đường thẳng d1 cắt CA, AB M, N đường thẳng d2 cắt cạnh CA, AB P, Q Đường thẳng PM cắt cạnh BC E đường thẳng QM cắt cạnh BC F Chứng minh IE = IF LỜI GIẢI Liên hệ tài liệu word toán SĐT (zalo): 039.373.2038 12 Website: tailieumontoan.com Sử dụng định lí Menelaus: M, N, I thẳng hàng ΔABC ⇒ IB MC NA = IC MA NB ⇒ MC NA = MA NB P, Q, I thẳng hàng ⇒ IC QB PA QB PA 1⇒ = = IB QA PC QA PC ⇒ MC NA QB PA PC NA QB MA (*) = ⇒ = MA NB QA PC PA NB QA MC Tương tự N, E, P Q, M, F EB PC NA FC QB MA = =1 EC PA NB FB QA MC ⇒ EB PC NA FC QB MA = EC PA NB FB QA MC Từ (*) suy ⇒ EB FC EB FC =⇒ = EC FB EB + EC FB + FC EB FC = ⇒ EB = FC ⇒ IE = IF BC BC Bài 20 Cho tam giác ABC, đường phân giác BD, CE cắt tai I, đường thẳng DE cắt BC F Chứng AF vng góc với AI LỜI GIẢI Gọi giao điểm AI với cạnh BC J Theo định lí Menelaus với tam giác ABC cát tuyến FED, ta có FB DC EA = FC DA EB Áp dụng định lí Ceva cho tam giác ABC với ba đường BD, CE, AJ đồng quy I: JC EB DA =1 JB EA DC Nhân hai đẳng thức cho ta có FB DC EA JC EB DA FB JC 1⇒ = = FC DA EB JB EA DC FC JB ⇒ FB JB FB JB AB = ⇒ = = FC JC FC JC AC => AF phân giác góc ∠BAC Liên hệ tài liệu word tốn SĐT (zalo): 039.373.2038 13 Website: tailieumontoan.com => AF vng góc với AI Bài 21 Cho tứ giác ABCD Đường thẳng d cắt AB, BC, CA, AD M, N, P, Q Chứng minh MA NB PC QD =1 MB NC PD QA LỜI GIẢI Từ A, B kẻ đường thẳng song song với CD cắt MN E F Theo định lí Thales ta có MA AE NB BI QD PD = = , = , MB IB NC CP QA AE ⇒ MA NB QD AE BI PD PD = = MB NC QA IB CP AE CP ⇒ MA NB PC QD = MB NC PD QA Bài 22 Cho tam giác ABC ( AC > AB ) , đường phân giác góc A đường trung trực BC cắt D Gọi H, K hình chiếu D AC AB, I trung điểm BC Chứng minh I, H, K thẳng hàng LỜI GIẢI Từ giả thiết suy DK = DH DB = DC => tam giác DHC, tam giác DKB hai tam giác vuông => CH = BK Xét đẳng thức IB HC KA IC HA KB Theo chứng minh IB HC KA IC ⇒ = = CH BK = , AH AK IB = IC HA KB Theo định lí đảo Menelaus ta suy I, H, K thẳng hàng Bài 23 Cho tam giác vuông cân ABC, AB = AC = a M N AB, AC thỏa mãn AM = AN = b BN CM cắt D Tính diện tích tam giác BDC theo a, b LỜI GIẢI Liên hệ tài liệu word toán SĐT (zalo): 039.373.2038 14 Website: tailieumontoan.com * Cách Kẻ DE vng góc với AB 1 AB.DE a.DE = 2 ⇒ S ABD = Giả thiết = AB AC = , AM AN => MN song song với BC AD qua trung điểm MN => AD phân giác ∠BAC ⇒ DE = EA Theo định lí Thales ta có DE BE DE a − DE = ⇒ = AN BA b a ab ⇒ DE = a+b a2b a ( a − b ) a2b = ⇒ S BDC = S ABC − S ADB = a − = a + b (a + b) a+b 2 ⇒ S ABD * Cách Áp dụng định lí Menelaus ba điểm M, D, C ta S MB CA DN DN b b a 1⇒ = = ⇒ CDN = ⇒ S DBC = S BNC MA CN DB DB a S DBC a a+b Mặt khác S= BNC ⇒= S DBC ( a − ab ) 2 a (a − b) a (a = − ab ) a+b a+b Bài 24 Cho tam giác ABC, (I) đường tròn nội tiếp tam giác ABC Gọi H, K trực tâm tam giác IAC IAB Chứng minh HK qua tiếp điểm đường tròn (I) với cạnh BC LỜI GIẢI Theo giả thiết H K trực tâm tam giác IAC IAB => BK, CH vng góc với AI => KB song song CH => tứ giác BHCK hình thang Gọi M giao điểm HI với AC => IM vng góc với AC Tương tự IN vng góc với AB Gọi D hình chiếu I cạnh BC Khi đó: M, N, D tiếp điểm đường tròn (I) với cạnh CA, AB, BC ⇒ CM = CD BN = BD Liên hệ tài liệu word toán SĐT (zalo): 039.373.2038 15 Website: tailieumontoan.com Vì ∠HCM = ∠KBN = 90° − ∠BAC => tam giác HCM tam giác KBN hai tam giác vuông đồng dạng ⇒ CH CM CD => H, D, K thẳng hàng = = BK BN BD Bài 25 Gọi M điểm nằm tam giác ABC Đường thẳng AM, BM, CM cắt cạnh đối diện D, E, F Chứng minh AM AE AF (Định lí Van Oben) = + MD EC FB LỜI GIẢI Qua A kẻ đường thẳng song song với BC cắt đường thẳng BM CM P Q ⇒ AM AQ AP = = MD DC BD AM AQ + AP AQ + AP ⇒ = = MD CD + BD BC = AQ AP AF AE AM AE AF + = + ⇒ = + BC BC FB EC MD EC FB Chú ý: Trong trường hợp đặc biệt, M trọng tâm tam giác ⇒ M tâm đường tròn nội tiếp tam giác ⇒ AM 2; = MD AM c b c + b Ta suy nhiều giá trị khác tùy = + = MD a a a theo vị trí M Bài 26 Cho tam giác ABC, M điểm nằm tam giác AM, BM, CM cắt cạnh đối diện D, E, F Chứng minh tỉ số AM BM CM có tỉ số khơng lớn tỉ số , , MD ME MF không nhỏ LỜI GIẢI Ba trung tuyến AI, BK, CP tam giác ABC chia tam giác ABC thành tam giác BGI, CGI, CGK, AGK, AGP, BGP => điểm M nằm tam giác đó, kể cạnh Giả sử M thuộc ΔAPK Theo định lí Van Oben ta có Mặt khác AM AF AE AF AE = + ≤ + ≤ MD FB EC PB KC BM BF BD BF BD = + ≥ + ≥2 ME FA DC PA ID Liên hệ tài liệu word toán SĐT (zalo): 039.373.2038 16 Website: tailieumontoan.com Vậy với điểm P ra, ta có AM BM ≤ ≥ Dấu xảy M trùng với G MD MD Bài 27 Cho tam giác ABC, M điểm tam giác AM, BM, CM cắt cạnh đối diện A1, B1, C1 Đường thẳng BC cắt B1C1 A2 Gọi A0 trung điểm A1A2.Tương tự ta có B0, C0 Chứng minh A0, B0, C0 thẳng hàng LỜI GIẢI Theo định lí Ceva ta có: AC1 BA1 CB1 = C1 B A1C B1 A Áp dụng định lí Menelaus với ΔABC cát tuyến A2 B1C1 ⇒ A2 C B1 A C1 B = A2 B B1C C1 A Nhân hai đẳng thức trên, ta được: BA1 A2 C BA AB ⇒ =2 = A1C A2 B A1C A2 C ⇒ A0 B A1 B A2 B + A1 B A0 B = = = A1C A2 C + A1C A2 A1 A0 A1 A B A2 B − A1 B A0 A1 = = A1C A2 C − A1C A0 C Nhân vế theo vế, ta được: A0 B A1 B = A0 C A1C B0 C B1C C0 A C1 A2 Tương tự: = = , B0 A B1 A2 C0 B C1 B 2 A B B C C A BC C A AB ⇒ = = A0 C B0 A C0 B B1 A C1 B A1C ⇒ A0 , B0 , C0 thẳng hàng Bài 28 Cho tam giác ABC nhọn, H hình chiếu A BC, M điểm tùy ý AH BM CM cắt AC, AB tương ứng D E Chứng minh ∠AHD = ∠AHE LỜI GIẢI * Cách 1: Gọi K, G hình chiếu E D cạnh BC EK cắt BD I, DG cắt CE J => EK, AH DG song song với Liên hệ tài liệu word toán SĐT (zalo): 039.373.2038 17 Website: tailieumontoan.com Theo định lí Thales, ta có: EI AM DJ EI EK hay = = = EK AH DG DJ DG (1) HK ME EI = = HG MJ DJ (2) Từ (1) (2) ⇒ HK EK = HG DG => hai tam giác vuông KHE tam giác GHD đồng dạng (c.g.c) ⇒ ∠KEH = ∠GDH ⇒ ∠AHD = ∠AHE * Cách Qua M kẻ đường thẳng song song với BC cắt cạnh AC AB thứ tự P Q, cắt HD HE thứ tự I J Vì PQ // BC nên theo định lí Thales, ta có: MI BH MJ HC , = = MP BC MQ BC Chia hai vế ta MI MQ BH (1) = MP MJ HC Mặt khác áp dụng Thalé, ta có: Thay vào (1) ⇒ MQ HB = MP HC MI = MJ ⇒ MI = MJ => tam giác MIJ cân M Mà MH vng góc IJ ⇒ ∠MHI = ∠MHJ ⇒ ∠AHD = ∠AHE Bài 29 Cho tam giác ABC, M N BC Chứng minh ∠BAM = ∠CAN BM BN AB = CN CM AC LỜI GIẢI Ta có ∠BAM = ∠CAN ⇔ S ABM AB AM = S ACN AC AN Theo tính chất tỉ số ta ln có Tương tự S ABM BM BM AB AM = ⇔ = S ACN CN CN AC AN BN AB AN BM BN AB = ⇔ = CM AC AM CN CM AC Liên hệ tài liệu word toán SĐT (zalo): 039.373.2038 18 Website: tailieumontoan.com Chú ý: AM, AN gọi đường đẳng giác góc ∠BAC (hay AM AN đối xứng với qua đường phân giác góc ∠BAC ) Một hai đường AM, AN trùng với trung tuyến thuộc cạnh BC gọi đối trung Giả AN trùng với đường trung tuyến ta có tốn: Tam giác ABC, AM đường đối trung BM AB (Định lí Steiner) = CN AC Bài 30 Cho tam giác ABC, AM, AN hai đường đẳng giác góc ∠BAC BD, BE hai đường đẳng giác góc ∠ABC Các đường cắt P, Q Chứng minh PQ, CQ đường đẳng giác góc ∠ACB LỜI GIẢI Theo giả thiết: AM, AN hai đường đẳng giác góc ∠BAC ⇒ BM BN AB =2 CN CM AC BD, BE hai đường đẳng giác góc ∠ABC ⇒ AD AE AB BM BN CE CD AB BC BC =2 ⇒ = = CE CD BC CN CM AD AE AC AB AC CQ cắt AB H, CP cắt AB K Theo định lí Ceva ta có AN, BE, CH đồng qui ⇒ HA NB EC AM, BD, CK đồng qui = HB NC EA ⇒ KA MB DC = KB MC DA ⇒ HA NB EC KA MB DC = HB NC EA KB MC DA ⇒ BM BN CE.CD HB.KB HA.KA CA2 = ⇒ = CN CM AD AE HA.KA KB.KB CB => CD, CK đường đẳng giác góc ∠ACB Chú ý: Hai điểm P Q gọi điểm đẳng giác Trong tam giác, đường đối trung đồng qui điểm Liên hệ tài liệu word toán SĐT (zalo): 039.373.2038 19 Website: tailieumontoan.com Bài 31 Cho tam giác nhọn ABC khơng cân Dựng ta phía ngồi tam giác ABC tam giác ABD ACE vuông cân B C Gọi M trung điểm DE Chứng minh MBC tam giác vuông cân LỜI GIẢI Gọi I J thứ tự trung điểm AD AE Theo giả thiết M trung điểm DE => MI song song AE MJ song song AD Tam giác ABD, ACE tam giác vuông cân ∠CJM ⇒ MJ = AI =IB, MI = AJ =JC Mà ∠MIB = => tam giác MIB tam giác CJM (c.g.c) ⇒ MB = CM Hai tam giác tam giác ABD ΔACE vuông cân nên chúng đồng dạng ⇒ AB BI BI = = AC CJ MI Mặt khác ∠DAE = ∠DIM Từ ta có: ∠BAC = ∠DEA − ∠DAB − ∠CAE = ∠DAE − 90° = ∠DIM − 90° = ∠BIM => tam giác BAC tam giác BIM đồng dạng (c.g.c) => ∠ABC = ∠IBM ⇒ ∠MBC = ∠IBA = 45° ⇒ tam giác MBC vuông cân Bài mở rộng Cho tứ giác ABCD Dựng ta phía ngồi tứ giác tam giác vuông cân ABM, CBQ, CDP, AND có đỉnh B D Gọi I J trung điểm MN PQ Chứng minh tứ giác BIDJ hình vng LỜI GIẢI Áp dụng toán => tam giác IBD JBD tam giác vuông cân I J ⇒ ∠IBJ = ∠IDJ = 90° => tứ giác BIDJ hình chữ nhật => tứ giác BIDJ hình vng Liên hệ tài liệu word toán SĐT (zalo): 039.373.2038 20 Website: tailieumontoan.com Bài 32 Cho tam giác ABC Dựng phía ngồi tam giác vng cân ABD ACE đỉnh D E Gọi M trung điểm cạnh BC Chứng minh MDE tam giác vuông cân LỜI GIẢI Trên tia đối tia DB CE lấy điểm P Q thỏa mãn = EC ⇒ DM, EM đường trung bình tam DP = DB EQ giác BPC tam giác CQB ⇒ = DM 1 PC , = EM QB (*) 2 Tam giác ADB vuông cân => AD vng góc PB DB= DP= DA ⇒ ∠PAB= 90°, AP= AB Tương tự ∠QAC = 900 , AQ = AC Xét hai tam giác PAC BAQ có ∠PAC = ∠PAB + ∠A = 90° + ∠A = ∠QAC + ∠A = ∠BAQ , AP = AB, AQ = AC ⇒ hai tam giác PAC BAQ (c.g.c) ⇒ PC = QB, ∠APC = ∠ABQ Kết hợp (*) ⇒ MD = ME Gọi H giao điểm PC BQ Từ AP vuông góc với AB ∠APC = ∠ABQ suy PC vng góc QB => DM vng góc ME hay ∠DME =° 90 => MDE tam giác vuông cân M Bài 33 Cho tứ giác ABCD Dựng phía ngồi tam giác vng cân ABP, BCM, CQD, DAN (đỉnh điểm P, M, Q, N) Chứng minh PQ, MN vng góc với LỜI GIẢI Gọi I trung điểm AC Sử dụng kết toán tam giác ABC tam giác ACD IN , ∠QIN = 90° ⇒ IP =IM , ∠PIM =90° IQ = Hai tam giác PIQ tam giác MIN= có IP IM = , IQ IN , ∠PIQ = ∠PIN + 90° = ∠MIN => hai tam giác (c.g.c) ⇒ PQ = MN , ∠IPQ = ∠IMN Mà IP vng góc IM => PQ vng góc với MN Liên hệ tài liệu word toán SĐT (zalo): 039.373.2038 21 Website: tailieumontoan.com Bài 34 Cho hình thang ABCD (AB song song CD) Dựng ta phía ngồi tam giác AED CFB đồng dạng ( ∠EAD = ∠FCB, ∠EDA = ∠FBC ) Gọi H K hình chiếu E, F cạnh AD BC Qua H, K kẻ đường thẳng song song với BD, tương ứng cắt AB M, cắt CD N Đường thẳng qua M, N vng góc với AB cắt EF Q P Chứng minh EP = FQ LỜI GIẢI Theo giả thiết ΔEAD ΔFCB đồng dạng, EH ⊥ AD FK ⊥ BC ⇒ AH CK = HD KB (1) HM song song với BD ⇒ AH AM = HD MB (2) NK song song với BD ⇒ CN CK = ND KB Từ (1), (2) (3) ⇒ (3) AM CK = ⇒ MK song song với AC; MB KB AH CN = ⇒ HN song song với AC HD ND => tứ giác MKNH hình bình hành Từ E, H hạ đường thẳng vng góc với CD L, U Từ F, K hạ đường vng góc với AB T V Theo định lí Thales ta có: HU DH BM BK KV (h khoảng cách AB, CD) = = = = h DA BA BC h ⇒ HU = KV ⇒ tứ giác HVKU hình bình hành => HK, UV cắt trung điểm đường Tứ giác EHDL tứ giác BTFK có góc đối vuông nên tứ giác nội tiếp ⇒ ∠EDH = ∠ELH = ∠LHD, ∠KBF = ∠KTF = ∠TKV Mặt khác theo giả thiết, hai tam giác EAD tam giác FCB đồng dạng => Tam giác LHU tam giác TKV => KT = LH LK song song KT => LHTK hình bình hành => đường chéo HK LT cắt trung điểm đường Liên hệ tài liệu word toán SĐT (zalo): 039.373.2038 22 Website: tailieumontoan.com => HK, UV LT cắt trung điểm I Gọi J trung điểm EF => IJ, LE, FT, NP, MQ song song với Mà I trung điểm MN => J trung điểm PQ => EP = PQ Bài 35 Cho tức giác ABCD Chứng minh AB.CD + BC AD ≥ AC.BD Dâu tứ giác ABCD nội tiếp (bất đẳng thức Ptolemy) LỜI GIẢI Lấy điểm M tứ giác cho ∠MDC = ∠ADB ∠MCD = ∠ABD => tam giác DBA CDM đồng dạng (g.g) ⇒ AB BD = ⇒ AB.CD = BD.MC (1) MC CD Theo dựng ta có ∠ADM = ∠BDC , BD AD = CD MD => tam giác ADM BDC đồng dạng (c.g.c) ⇒ AD AM = ⇒ AD.BC = AM BD (2) BD BC Cộng (1) (2) ta AB.CD + AD.BC = BD ( MC + MA ) ≥ BD AC ⇒ AB.CD + BC AD ≥ AC.BD Dâu xảy M cạnh AC => A, M, C thẳng hàng => tứ giác ABCD nội tiếp Bài 36 Cho hình vng ABCD, đường thẳng m qua đỉnh B Gọi H K hình chiếu A C m AK CH cắt E Chứng minh DE vng góc với HK LỜI GIẢI * Cách Nối H với D K với D H, B, K đường thẳng m ⇒ ∠HBA + KBC= 90° Mà ∠HBA + ∠HAB = 90° ⇒ ∠HBA = ∠KCB Tam giác HAB tam giác KCB hai tam giác vng có AB = BC ∠HBA = ∠KCB => hai tam giác ⇒ HB = CK HA = BK Liên hệ tài liệu word toán SĐT (zalo): 039.373.2038 23 Website: tailieumontoan.com ∠HAD = ∠HAB + ∠BAD = ∠KBA + 90° = ∠ABK => Tam giác AHD tam giác BKA (c.g.c) ⇒ ADH = ∠BAK Mặt khác ∠BAK + ∠DAK= 90° ⇒ ∠ADH + ∠DAK= 90° => DH vng góc với AK Tương tự CH vng góc với DK Tam giác HDK có AK CH hai đường cao cắt E => E trực tâm tam giác => DE vng góc với HK => DE vng góc đường thẳng m * Cách Qua D dựng đường thẳng song song với đường thẳng m cắt KC HA P Q => tứ giác HKPQ hình chữ nhật Tam giác ABH ADQ hai tam giác vuông nhau, tam giác ABH BCK ⇒ AQ = HB AH = BK ⇒ HK = HQ ⇒ HKPQ hình vng AH, CK vng góc với m => AH song song với CE Theo định lí Thales ta có: AE AH QD = = EK CK DP => DE song song với CK => DE vng góc với HK => DE vng góc đường thẳng m * Cách Theo giả thiết AH, CK vng góc với đường thẳng m => AH song song CK ⇒ AE AH = EK CK Mặt khác tam giác ABH tam giác BCK hai tam giác vuông ⇒ HB = CK ⇒ AE AH = EK HB (1) Kéo dài AH, BK cắt CD kéo dài I J => tam giác vuông AID tam giác JCK đồng dạng (g.g) ⇒ DI CK DI CK (2) = ⇒ = AD KJ DC KJ Tam giác CKJ AHB đồng dạng (g.g) ⇒ CK AH = KJ HB (3) Liên hệ tài liệu word toán SĐT (zalo): 039.373.2038 24 Website: tailieumontoan.com Từ (1), (2) (3) ⇒ ID AE = ⇒ DE song song với CK DC EK => DE vng góc với đường thẳng m 1 Bài 37 Cho tam giác ABC thỏa mãn ∠A = 2∠B = 4∠C Chứng minh = + c b a LỜI GIẢI * Cách Từ toán: Trong tam giác ABC, chứng minh ∠A = 2∠B a= b + bc (Bạn đọc tự chứng minh) Từ giả thiết ∠A = 2∠B = 4∠C ⇒ ∠A = 2∠B ∠B = 2∠C 2 Tứ ta có: a= b + bc (1) b= c + ca (2) Tam giác ABC không cân ⇒ b − c ≠ Nhân hai vế (1) với b − c ⇒ a ( b − c ) = (b + bc ) ( b − c ) = b ( b − c ) (3) Từ (2) ⇒ b − c = ca Thay vào (3) ⇒ a ( b − c ) = bca ⇒ a ( b − c ) = bc Chia hai vế cho abc ⇒ 1 = + c b a * Cách Gọi H trung điểm BC Qua H dựng đường thẳng vng góc với BC cắt AB kéo dài D => ΔDBC tam giác cân D ⇒ ∠B = ∠BCD Theo giả thiết ∠A = 2∠B = 4∠C ⇒ 7∠C = 180° Đặt α= 180° ⇒ ∠C= α , ∠B= 2α , ∠A= 4α ⇒ ∠DAC = ∠B + ∠C = 3α ⇒ ∠ACD= α ⇒ ∠BDC= 3α ⇒ CA = CD = BD ⇒ AB AB = AC BD CA phân giác góc ∠BCD ⇒ (1) AB AD = (2) BC CD Cộng (1) với (2) ⇒ AB AB AB AD AB + AD + = + = =1 AC BC BD CD BD ⇒ 1 1 1 + = ⇒ + = AC BC AB a b c Liên hệ tài liệu word toán SĐT (zalo): 039.373.2038 25 Website: tailieumontoan.com Bài 38 Tỉ lệ ba góc tam giác : : Chứng minh chân ba đường phân giác tam giác tam giác cân LỜI GIẢI * Cách ∠A : ∠B : ∠C= : :1 ⇒ ∠A ∠B ∠C = = ⇒ ∠A = 2∠B = 4∠C b + bc ⇒ ab − ac = bc a= ⇒ a + ac = b + ab AE AB c Theo tính chất đường phân giác = = EC BC a AE c bc = ⇒ AE = AE + EC a + c a+c ⇒ Tương tự BF = ac a+b c ( ab + b ) − ( ac + a ) bc ac − = = BF ⇒ AE = Suy AE − BF ⇒ a+c a+b ( a + b )( a + c ) Mặt khác AD phân giác góc ∠A nên ∠BAD = ∠ABD ⇒ΔABD cân D ⇒ AD = BD => hai tam giác FBD EAD (c.g.c) ⇒ DF = DE => tam giác DFE cân D * Cách Giả sử ∠A = 2∠B = 4∠C ⇒ ∠A ∠B ∠C 180° = = = Đặt 180° = 2α ⇒ ∠= A 8α , ∠B = 4α , ∠C = 2α , 180= ° 14α Liên hệ tài liệu word toán SĐT (zalo): 039.373.2038 26 Website: tailieumontoan.com Gọi I giao điểm ba đường phân giác Từ B kẻ đường thẳng vng góc với AD cắt cạnh AC M => tam giác ABM cân A hay B M đối xứng qua AD ⇒ ∠ABM =° 90 − 4α = 7α − 4α = 3α , ∠CBM = ∠ABM − ∠ABM = α BE phân giác ∠ABC ⇒ ∠MBE =α AD trục đối xứng ⇒ ∠IMB =α => IM song song BC ⇒ ∠EIM = ∠EMI = 2α ⇒ EM = EI ∠AEB = ∠EBC + ∠ECB = 4α ⇒ ∠IAE = ∠IEA ⇒ IA = IE ∠AFC = ∠FBC + ∠FCB = 5α ∠AIF = ∠CID = ∠CAI + ∠ICA = 4α + α = 5α => tam giác AIF cân A ⇒ AI = AF ⇒ AF = EM ⇒ BF = AB − AF = AM − EM = AE AD phân giác ∠A= 8α , ∠B= 4α ⇒ => tam giác ABD cân D ⇒ AD = BD Suy hai tam giác DBF tam giác DAE (c.g.c) ⇒ DF = DE ⇒ tam giác DEF tam giác cân D Từ hai toán ta đến với toán sau: Bài 39 Cho đa giác ABCDEFG Chứng minh đẳng thức: 1 1 1 , =+ =+ AB BD DA BC EG AD LỜI GIẢI Đa giác ABCDEFG => AB= BC= = GA Các đường chéo tạo hai cạnh liên tiếp AC = BD Các đường chéo tạo ba cạnh liên tiếp AD = AE π 2π 4π ∠ADB = , ∠BAD = , ∠ABD = 7 Theo tốn ta có đẳng thức 1 1 1 =+ , =+ AB BD DA BC EG AD Từ ta lập nhiều đẳng thức Bài 40 Cho tam giác cân ABC ( AB = AC ) , ∠BAC = 20° Xác định đẳng thức liên hệ cạnh tam giác Chứng minh a + b3 = 3ab LỜI GIẢI Liên hệ tài liệu word toán SĐT (zalo): 039.373.2038 27 Website: tailieumontoan.com Từ giả thiết ∠ABC = ∠ACB = (180° − 20° ) = 80° Dựng điểm D cạnh AB cho góc ∠BCD = 20° Suy ∠BDC= 80° ⇒ tam giác CBD cân C => hai tam giác ABC CBD đồng dạng ⇒ AB BC = ⇒ BC = AB.BD = AB ( AB − AD ) BC BD ⇒ BC = AB − AB AD ∠ACD = ∠ACB − ∠DBC = 80° − 20° = 60° Áp dụng cạnh đối diên góc 60° ta được: AD = AC + CD − AC.CD = AB + BC − AB.BC Từ đẳng thức BC AB − AB AD = AB − BC ⇒ AD AB = ⇒ AB AD = ( AB − BC ) ⇒ AB ( AB + BC − AB.BC ) = ( AB − BC ) 3BC AB ⇒ AB + BC = Bài 41 Cho tam giác cân ABC ( AB = AC = b, BC = a ) BD phân giác góc ∠ABC cắt AC D thỏa mãn BD + AD = BC Chứng minh a + b3 = 3ab LỜI GIẢI * Cách Trên BC lấy điểm E cho BE = BD Thay vào đẳng thức cho ta được: BE + AD = BE + CE ⇒ AD = CE Theo tính chất đường phân giác ⇒ AD AB CE AC = ⇒ = DC BC DC BC => tam giác ACB tam giác ECD đồng dạng ⇒ ∠ECD = ∠CDE = ∠ABC => tứ giác ABED nội tiếp Mặt khác ∠DEB = 2∠DCE = 2∠ABC Mà ∠BDE + ∠DEB + ∠DBE = 180° 180° ⇒ ∠ABC = 40° ⇒ 4∠ABC + ∠ABC = Kẻ phân giác góc ∠BDC cắt BC F Liên hệ tài liệu word toán SĐT (zalo): 039.373.2038 28 Website: tailieumontoan.com ∠BDC = 180° − ∠DBC − ∠DCB = 120° ⇒ ∠BDF= ∠FDC= 60°, ∠ADB= 60° => tam giác DBA tam giác DBF (c.g.c) ⇒ AB = BF AD AB bc b2 ab = ⇒ AD = = , DC = DC BC a+c a+b a+b Thay vào đẳng thức BD + AD = BC ⇒ BD + b2 a + ab − b = a ⇒ BD = a+b a+b DF phân giác góc ∠BDC ⇒ ⇒ BF DB = CF DC b a + ab − b = ⇒ ab =a + a b − ab − a b − ab + b3 a−b ab ⇒ a + b3 = 3ab * Cách Từ kết ∠ABC = 40° , trung trự AC cắt BC M ⇒ AM = MC ⇒ tam giác ABC tam giác MAC hai tam giác cân đồng dạng ⇒ AB MA a − b2 = ⇒ b = a − a.BM ⇒ BM = BC AC a Mặt khác ∠BAM =° 60 ⇒ BM = AB + AM − AB AM = b + ( a − BM ) − b ( a − BM ) = BM a + b − ab a − b a + b − ab ⇒= ⇒= a + b3 3ab 2a − b a 2a − b Bài 42 Chotam giác ABC điểm M nằm tam giác thỏa mãn: ∠AMC − ∠ABC = ∠AMB − ∠ACB Đường phân giác góc ∠ABM ∠ACM cắt I Chứng minh A, I, M thẳng hàng LỜI GIẢI Trên cạnh AC tam giác ABC, dựng phía ngồi góc: ∠DAC = ∠BAM ∠ACD = ∠ABM => tam giác ACD tam giác ABM đồng dạng (g.g) ⇒ ∠ADC = ∠AMB AB AM = AC AD Mặt khác ∠BAC = ∠MAD => tam giác ABC tam giác AMD đồng dạng (c.g.c) ⇒ ACB = ∠ADM , ∠ABC = ∠AMD Liên hệ tài liệu word toán SĐT (zalo): 039.373.2038 29 Website: tailieumontoan.com ⇒ ∠MDC = ∠ADC − ∠ADM = ∠AMB − ∠ACB ; ∠DMC = ∠AMC − ∠AMD = ∠AMC − ∠ABC Theo giả thiết ⇒ MDC = ∠DCM ⇒ CM = CD Tam giác ACD tam giác ABM đồng dạng ⇒ CD BM CM BM = ⇒ = CA BA CA BA => I giao điểm phân giác góc ∠ACM vứi AM Bài 43 Cho tam giác nhọn ABC, H trực tâm Gọi I trung điểm cạnh BC Đường thẳng qua H vuông góc với HI cắt AC M AB N Chứng minh H trung điểm MN LỜI GIẢI * Cách Kéo dài BH lấy điểm K cho HK = HB I trung điểm BC => HI đường trung bình tam giác BCK => HI song song với CK Mà HI vuông góc với MN => HM vng góc với CK Giả thiết H trực tâm tam giác ABC => BH vng góc AC hay CM vng góc HK => M trực tâm tam giác HKC => KM vng góc với HC Mà CH vng góc AC nên KM song song với AB ⇒ ∠HKM = ∠HBN => tam giác HKM tam giác HBN (g.c.g) ⇒ HM = HN * Cách Theo giả thiết H trực tâm tam giác ABC => AH vng góc BC CH vng góc AB ⇒ BAH = ∠BCH Cũng theo giả thiết HI vng góc MN ⇒ ∠IHC = ∠HNA => tam giác ANH tam giác CHI đồng dạng (g.g) ⇒ AH HN = CI IH Tương tự (1) AH HM = BI IH Từ (1) (2) IB =IC ⇒ (2) HN HM = ⇒ HM =HN HI HI Liên hệ tài liệu word toán SĐT (zalo): 039.373.2038 30 Website: tailieumontoan.com * Cách Đường kính qua A cắt đường trịn ngoại tiếp tam giác ABC K ⇒ ∠ABK = ∠ACK = 90° => KB song song với CH KC song song với BH => tứ giác BHCK hình bình hành => H, I, K thẳng hàng KH vng góc với MN => tứ giác BNHK nội tiếp ⇒ ∠NKH = ∠NBH Tương tự ∠HKM = ∠HCM Mặt khác BH, CH đường cao ⇒ NBH = ∠HCM ⇒ ∠NKH = ∠MKH => tam giác KMN cân K ⇒ HM = HN * Cách Dựng đường trịn tâm I đường kính BC cắt cạnh AC D AB E => BD vng góc với AC CE vng góc với AB => BD CE đường cao tam giác ABC => BD CE giao H trực tâm tam giác Theo giả thiết đường thẳng qua H vuông góc với IH cắt đường trịn tâm I P Q ⇒ HP = HQ Cát tuyến CD EB cắt PQ M N Theo toán “Con bướm” ta có HM = HN Bài 44 Cho tam giác ABC, AB = AC , D điểm cạnh AC cho CD = AD P điểm BD cho ∠APC = 90° Chứng minh ∠ABP = ∠PCB LỜI GIẢI Liên hệ tài liệu word toán SĐT (zalo): 039.373.2038 31 Website: tailieumontoan.com Kéo dài BA lấy điểm E cho AE = AB BD cắt CE F => A trung điểm BE ⇒ AB= AE= CA ⇒ ∠BCE= 90° Theo giả thiết CD = AD => D trọng tâm tam giác FC ⇒ FE = => AF song song với BC ⇒ ∠AFC= 90° ∠AFB = ∠FBC , ∠APC = 90° => tức giác AFCP tứ giác nội tiếp ⇒ ∠AFP = ∠ACP ⇒ ∠ACP = ∠FBC ⇒ ∠ABP = ∠ABC − ∠FBC = ∠ACB − ∠ACP = ∠BCP Bài 45 Cho tam giác ABC thỏa mãn BC = AB Đường phân giác góc ∠ABC cắt cạnh AC D Đường vng góc với BD kẻ từ B cắt đường thẳng song song với AB kẻ từ C E Chứng minh DE qua trung điểm cạnh BC LỜI GIẢI Gọi F giao điểm AB DE M giao điểm DE với cạnh BC Theo giả thiết AB song song với CE ⇒ ∠ABC = ∠BCE BD phân giác góc ∠ABC ⇒ ∠ABD = ∠DBC BD vng góc với BE ⇒ ∠BCE = 2∠DBC ⇒ ∠CBE= 90° − ∠DBC ⇒ ∠BEC = 180° − ∠CBE − ∠BCE = 180° − ( 90° − ∠DBC ) − 2∠DBC = 90° − ∠DBC = ∠CBE => tam giác BCD cân C ⇒ CB = CE Theo tính chất đường phân giác ⇒ CD BC = =2 DA BA AB song song với CE ⇒ CE CD = = ⇒ CE = AF ⇒ AF = AB AF DA ⇒ CE = BF Liên hệ tài liệu word toán SĐT (zalo): 039.373.2038 32 Website: tailieumontoan.com Theo định lí Thales ta có BM BF = = CM ⇒ BM = MC CE Bài 46 Cho tam giác ABC D hình chiếu A BC, E hình chiếu D AC, M điểm DE cho DM cot C Chứng minh AM vng góc với BE = ME cot B LỜI GIẢI * Cách BK vng góc với AC => BK song song với DE ⇒ DC EC = DB EK Theo định nghĩa cot B = ⇒ DB DC , cot C = AD AD cot C DC = cot B DB Theo giả thiết DM cot C EC DM EM DM = ⇒ = ⇒ = ME cot B EK ME EK EC EM DM + EM DE (1) ⇒= = EK EC + EK CK AD, BK đường cao tam giác ABC ⇒ ∠DAE = ∠CBK = 90° − ∠ACB => hai tam giác vuông AED BKC đồng dạng (g.g) ⇒ Kết hợp (1) ⇒ AE ED = BK KC EM AE = EK BK => tam giác MEA tam giác EKB đồng dạng (c.g.c) ⇒ ∠EAM = ∠KBE ⇒ ∠MAD = ∠EBC Gọi N giao điểm BE AM ⇒ ∠NAD = ∠NBD => tứ giác ABDN nội tiếp ⇒ ∠ANB = ∠ADB = 90° hay AM vng góc BE Nhận xét: Cách ngồi kiến thức hình học địi hỏi phải nắm vững tính chất phép biến đổi tỉ lệ thức * Cách Bài toán mấu chốt chứng minh ∠EAM = ∠EBK Biến đổi FE EK = ED KC EK ED EM ED EK KC EK EK tan ∠EAM = = KC = = = = tan ∠KBE EA EA EA KC KB KC KB Liên hệ tài liệu word toán SĐT (zalo): 039.373.2038 33 Website: tailieumontoan.com ⇒ ∠EAM = ∠KBE ⇒ ∠MAD = ∠EBC ⇒ AM vng góc BE Bài 47 Cho tam giác ABC nhọn, trực tâm H Gọi I trung điểm AC Qua I dựng đường l thẳng song song với phân giác góc ∠AHC Chứng minh đường thẳng l chia tam giác ABC hai phần có chu vi LỜI GIẢI Giả sử đường thẳng l cắt AB M H trực tâm tam giác ABC ⇒ ∠AHC + ∠ABC = 180° Dựng đường phân giác góc ∠ABC cắt AC E Gọi HF phân giác góc ∠AHC => HF song song BE CD Dựng hình hình hành AMCD ⇒ AM = Theo giả thiết MI song song HF => MI song song BE ⇒ ∠AMI = ∠MBE = ∠EBC = ∠MNB ⇒ΔBMN cân B ⇒ BM = BN Tương tự ΔCDN cân C ⇒ CD = CN ⇒ BC + BM = CN = CD = AM ⇒ AM + AI = BM + BC + CI Bài 48 Cho tam giác ABC D, E, F thứ tự BC, CA, AB AD cắt EF P PD EC FB Chứng minh = BC DB + DC PA EA FA LỜI GIẢI Từ A kẻ đường a thẳng song song với BA Đường thẳng EF cắt BC M cắt a N Theo định lí Thales PD MD EC MC FB MB ,= ,= ⇒= PA AN EA AN FA AN Vế phải đẳng thức cần chứng minh thay MC MB MC.DB + MB.DC DB + DC = AN AN AN DB ( MD + DC ) + DC ( MD − BD ) MD.DB + MD.DC MD ( DB + DC ) = = AN AN AN = MD PD = BC BC AN PA Liên hệ tài liệu word toán SĐT (zalo): 039.373.2038 34 Website: tailieumontoan.com Bài 49 Cho tứ giác ABCD Đường chéo AC BD cắt I E điểm tùy ý AB Đường thẳng EI cắt CD F Chứng minh EB FC IB IC = EA FD ID IA LỜI GIẢI Sử dụng cơng thức diện tích hai tam giác có đường cao: S S IBE BE CF IFC = = S IFD FD S IAE EA Nhân hai vế với ta S IBE S IFC BE CF ; (1) = S IAE S IFD EA FD S S IBE IB.IE IC.IF ICF = = S IDF ID.IF S IAE IA.IE Nhân hai vế với ta S IBE S IFC IB IC = ( 2) S IAE S IFD IA ID Từ (1) (2) suy S IBE S IFC EB FC IB IC = = S IAE S IFD EA FD ID IA Bài 50 Cho tứ giác ABCD nội tiếp đường tròn AB BC cắt P Đường chéo AC BD cắt E H hình chiếu E BC Gọi M, N trung điểm CD PE Chứng minh tam giác PAC tam giác NHM đồng dạng LỜI GIẢI Gọi I trung điểm CE K trung điểm CP ⇒ MI song song DB IK song song PE, NI song song PC, KM song song PD Ta có: ∠KMI = ∠DMI − ∠DMK = (180 o − ∠CMI ) − (180o − ∠CMK ) = (180 o − ∠BDC ) − (180o − ∠CDA ) = ∠CDA − ∠CDB = ∠ADB Mặt khác ∠ADB = ∠ACB = ∠EIN = ∠INK ⇒ ∠KMI = ∠KNI ⇒ tứ giác MNKI nội tiếp EH vuông góc với BC, IE = IC ⇒ ∠IHK = ∠ICK = ∠INK = ∠IMK ⇒ tứ giác HKIM nội tiếp ⇒ N, H, K, I, M nằm đường tròn ⇒ ∠HNM = ∠CKM = ∠CAD; Liên hệ tài liệu word toán SĐT (zalo): 039.373.2038 35 Website: tailieumontoan.com ∠HMN = ∠HIN = ∠IHC = ∠PCA ⇒ tam giác PAC tam giác NHM đồng dạng (g.g) Bài 51 Cho tam giác vuông ABC vuông A Đường phân giác góc B C cắt cạnh AC, AB D E Đường cao AH cắt BD, CE P Q Chứng minh đường thẳng nối trung điểm PD QE song song với BC LỜI GIẢI Giả thiết BD phân giác góc ABC ⇒ ∠ABD = ∠DBC ⇒ ∠ADB = 90o − ∠ABC Giả thiết AH vng góc cạnh huyền BC ⇒ ∠APD = ∠BPH = ∠PAB + ∠PBA 1 = 90o − ∠ABC + ∠ABC = 90o − ∠ABC 2 ⇒ ∠ADB = ∠APD ⇒ tam giác APD cân A Tương tự tam giác AEQ cân A Gọi I, J trung điểm PD QC ⇒ AI vng góc PD AI cắt BC M ⇒ BI đường cao đồng thời phân giác ∠ABM ⇒ tam giác BAM cân B ⇒ IA = IM Tương tự JA = JN ⇒ JI đường trung bình tam giác AMN ⇒ JI song song BC Bài 52 Cho hình thang ABCD (BC song song AD, BC < AD) Hai đường chéo cắt P Q điểm bên ngồi hình thang cho ∠AQD = ∠BQC Chứng minh ∠BQP = ∠DAQ LỜI GIẢI Từ B kẻ đường thẳng song song với DQ cắt PQ M Theo giả thiết: BC song song AD, theo tính chất góc có cạnh tương ứng song song ta có: ∠ADQ = ∠CBM (1) MB song song DQ BC song song AD, theo định lí Thales ta có: MB BP BC = = DQ DP AD Kết hợp (1) ⇒ tam giác MBC tam giác QDA đồng dạng (c.g.c) ∠DAQ ( ) ⇒ ∠BMC = ∠DQA ∠BCM = Giả thiết ∠BQC = ∠DAQ ⇒ ∠BMC = ∠BQC ⇒ tứ giác MBCQ nội tiếp ⇒ ∠BQM = ∠BCM ( 3) ; Kết hợp (2) (3) ⇒ ∠BQP = ∠DAQ Liên hệ tài liệu word toán SĐT (zalo): 039.373.2038 36 Website: tailieumontoan.com Bài 53 Cho tam giác ABC, đường cao AD, BE, CF Gọi M, N, P, Q thứ tự hình chiếu D AC, AB, BE, CF Chứng minh M, N, P, Q thẳng hàng LỜI GIẢI Theo giả thiết BE vng góc AC, DM vng góc AC ⇒ DM song song BE ⇒ DM CD = BE CB DQ vng góc CF CF vng góc AB ⇒ DQ song song AB ⇒ CD DQ = CB BF Từ hai đẳng thức suy DM DQ = BE BF Mặt khác ∠FBE = ∠QDM (góc có cạnh tương ứng song song) ⇒ tam giác QDM tam giác FBE đồng dạng (c.g.c) ⇒ EF song song MQ Tương tự, NP song song EF Gọi H trực tâm tam giác ABC HE AH HF = =, ∠FHE = ∠NDM DM AD DN ⇒ tam giác MDN tam giác EHF đồng dạng (c.g.c) ⇒ MN song song EF ⇒ M, N, P, Q thẳng hàng Bài 54 Cho tam giác ABC thỏa mãn ∠BAC = 45o , ∠ACB = 30o , M trung điểm BC Chứng minh ∠AMB = 45o BC.CA = AM AB LỜI GIẢI • Cách Kẻ BD vng góc AC Theo giả thiết ∠BAC = 45o ⇒ tam giác ABD vuông cân D Cũng theo giả thiết MB = MC ⇒ MB = MD = MC ∠DBC = 90o − ∠= C 60o ⇒ tam giác BDM tam giác ⇒ MD = DB = DA ⇒ tam giác ADM cân D ⇒ ∠DAM = ∠DMA = 15o ⇒ ∠AMC = ∠AMD + ∠DMC =15o + 120o =135o Liên hệ tài liệu word toán SĐT (zalo): 039.373.2038 37 Website: tailieumontoan.com ⇒ ∠AMB = 180o − ∠AMC = 45o AB BC AB = ⇒ = o o sin ∠30 sin ∠45 BC AC AM AC , = ⇒ = o o sin ∠30 AM sin ∠135 ⇒ BC.CA = AM AB • Cách Hai tam giác BAC BMA đồng dạng (g.g) ⇒ AB AC = MB AM AB AM =AC.MB ⇒ AB AM =AC.BC Bài 55 Cho hình bình hành ABCD, I điểm AC Đường thẳng DI cắt BC, AB F E cho tam giác IFC BFE có diện tích Tính AI IC LỜI GIẢI Theo giả thiết ABCD hình bình hành ⇒ AD song song BC AD = BC Theo định lí Thales ⇒ AI AD BC = = IC FC FC BC BF + FC BF Mặt khác = = +1 FC FC FC = BE BE + 1= +1 CD AB ⇒ AI BE = + (1) IC AB Cũng theo giả thiết tam giác IFC tam giác BFE có diện tích ⇒ IF FC = BF FE ⇒ BF FC = IF FE ⇒ tam giác BFI tam giác CFE đồng dạng (c.g.c) ⇒ ∠BIF = ∠CEF ⇒ BI song song CE ⇒ Kết hợp (1) (2) ta Giải phương trình ta có AI = IC AI AB = ( ) IC BE AI AI +1 ⇒ − 1= − AI IC IC IC AI + = IC Bài 56 Cho tam giác vuông ABC (BC > CA > AB) Gọi M, N thứ tự trung điểm BC AB Đường trung trực cạnh BC cắt cạnh AC D Chứng minh DM = DN AB : AC : BC = 1: : Liên hệ tài liệu word toán SĐT (zalo): 039.373.2038 38 Website: tailieumontoan.com LỜI GIẢI Theo giả thiết tam giác ABC vuông, thỏa mãn BC > CA > AB ⇒ ∠BAC =90o Đặt BC = a, CA = b, AB = c, a= b2 + c2 DM trung trực BC ⇒ MB = MC DM vng góc BC ⇒ tam giác ACB tam giác MCD đồng dạng (g.g) CD a a a2 ⇒ = ⇒ CD = CM = 2b CM b b ⇒ AD =AC − CD =b − 2 a 2b − a 2b − ( b + c ) b − c = = = 2b 2b 2b 2b DM =DN ⇔ DM =DN ⇔ AD + AN =DC − CM (theo định lí Pythagoras với tam giác AND tam giác MDC tam giác vuông) ⇔ DC − AD = AN + CM ⇔ ( DC + AD )( DC − AD ) = AN + CM ⇔ AC ( DC − AD ) = AN + CM ⇔ DC − AD = AN + CM c + a = AC 4b CD + AD CD − AD CA CD − AD b c + a ⇒ CD = + = + = + 2 2 8b = 4b + c + a 3b + 2a a2 3b + 2a = = ⇒ = 8b 8b 2ab 8b ⇔ 2= a 3b , 2b + 2= c 3b ⇔ b = 2c ⇔ 2a = 3b = 6c AB AC BC = = ⇔ AB : AC : BC = 1: : 3 Bài 57 Cho tam giác ABC (AB khác AC) Phân giác BE CF cắt I Đường cao AH cắt BE CF M N Trên cạnh BC lấy hai điểm P, Q thỏa mãn CP = CA, BQ = BA Chứng minh tam giác IMN tam giác APQ đồng dạng LỜI GIẢI Gọi giao điểm AQ BE D Theo giả thiết ∠ABE = ∠CBE BQ = BA suy tam giác ABQ cân B ⇒ BD ⊥ AQ Cũng theo giả thiết AH ⊥ BC suy tứ giác MDQH nội tiếp ⇒ ∠IMN = ∠DQH ∠KPH Tương tự ∠INM = ⇒ ∆IMN ∆AQP đồng dạng (g.g) Liên hệ tài liệu word toán SĐT (zalo): 039.373.2038 39 Website: tailieumontoan.com Bài 58 Cho tam giác ABC ( ∠BAC > 90o ) , D điểm BC cho BD = DA, ∠DAC = 90o Gọi M, N trung điểm AB AC Đường thẳng qua D song song với AB cắt MN F Chứng minh ∠ACF = ∠BCM LỜI GIẢI Theo giả thiết MA = MB, NA = NC ⇒ MN song song BC Gọi I trung điểm BC ⇒ IB = IC ⇒ MI song song AC ⇒ ∠ANM = ∠ACB = ∠MIB MF song song BD, DF song song AB ⇒ tứ giác BMFD hình bình hành ⇒ DF = BM = MA ⇒ tứ giác MAFD hình bình hành Mặt khác DB = DA ⇒ DM vng góc AB ⇒ AF vng góc AB ⇒ ∠MAF =90o Cũng theo giả thiết ∠DAC = 90o ⇒ ∠BAD = ∠FAC DB = DA ⇒ ∠ABD = ∠DAB ⇒ ∠FAC = ∠ABD ⇒ tam giác ANF tam giác BIM đồng dạng (g.g) ⇒ FN AN NC FN MI = = ⇒ = MI BI IC NC IC Tứ giác MNCI hình bình hành ⇒ ∠FNC = ∠MIC ⇒ tam giác FNC tam giác MIC đồng dạng (c.g.c) ⇒ ∠NCF = ∠ICM ⇒ ∠ACF = ∠BCM Bài 59 Cho tam giác ABC vng A D hình chiếu A cạnh BC DE phân giác góc ∠ADC Gọi M, N trung điểm DC AE Đường thẳng MN cắt AD F Chứng minh BD = 2AF LỜI GIẢI Liên hệ tài liệu word toán SĐT (zalo): 039.373.2038 40 Website: tailieumontoan.com Từ M kẻ đường thẳng song song với AD cắt AC P Từ P kẻ đường thẳng song song BC cắt AD Q MP song song AD, MD = MC ⇒ PC = PA PQ song song CD ⇒ QD = QA Theo giả thiết NE = NA, áp dụng định lí Thales đảo ⇒ QN song song DE ⇒ QN phân giác góc ∠AQP ⇒ AN AQ = (1) NP QB PM song song AD ⇒ AN AF = ( ) NP MP Tam giác BAC vuông A, AD vng góc BC, PQ song song BC ⇒ tam giác AQP tam giác BDA đồng dạng (g.g) ⇒ Từ (1), (2) (3) ⇒ AQ BD = ( 3) QP DA AF BD = PM DA Mặt khác PM = QD = AD ⇒ BD = AF Bài 60 Cho tam giác ABC Dựng tam giác ABC tam giác DBC, ECA, FAB thoả mãn ∠DBC = ∠ECA = ∠FAB ∠DCB = ∠EAC = ∠FBA Chứng minh AF + FB + BD + DC + CE + EA ≥ AD + BE + CF LỜI GIẢI Trước hết chứng minh tứ giác AB.CD + BC AD ≥ AC.BD Chứng minh: Dựng tam giác EDC thoả mãn ∠ADB = ∠EDC , ∠ABD = ∠ECD ⇒ tam giác DBA tam giác DCE đồng dạng (g.g) ⇒ AD AB BD = = ED EC CD ⇒ AB.CD = BD.EC (1) Cũng từ giả thiết ⇒ ∠ADE = ∠BDC ⇒ tam giác ADE tam giác BDC đồng dạng (c.g.c) ⇒ AD BD = ⇒ AD.BC = BD AE AE BC ( 2) = BD ( CE + AE ) ≥ BD AC Cộng (1) (2) ⇒ AB.CD + AD.BC Liên hệ tài liệu word toán SĐT (zalo): 039.373.2038 41 Website: tailieumontoan.com Áp dụng cho tứ giác ABDC: AB.CD AC.BD + ≥ AD BC BC Từ giả thiết tam giác DBC, ECA, FAB đồng dạng AB BF AC CE ⇒ = , = BC CD BC BD ⇒ AB.CD AC.BD = BF , = CE BC BC Cộng đẳng thức ta BF + CE ≥ AD Dấu xảy ABDC nội tiếp Tương tự AF + CD ≥ BE AE + BD ≥ CF Cộng đẳng thức ta điều phải chứng minh Dấu xảy tam giác ABC ∆BDC cân có góc đỉnh 120o Bài 61 Cho tam giác ABC D E hình chiếu A đường phân giác góc ∠ABC , ∠ACB Chứng minh DE song song với cạnh BC LỜI GIẢI Gọi I giao điểm hai đường phân giác góc ∠ABC , ∠ACB Theo giả thiết ta có AD vng góc BD AE vng góc CE ⇒ ∠ADI = ∠AEI = 90o ⇒ tứ giác ADIE nội tiếp ⇒ ∠IAE = ∠IDE (1) Tam giác AEI tam giác AEC tam giác vuông E ⇒ ∠IAE = ∠CAE − ∠CAI = 90o − ∠ACE − ∠CAI 1 90o − ∠ACB − ∠BAC = ∠ABC = = ∠DBC ( ) 2 Từ (1) (2) ⇒ ∠IDE = ∠DBC ⇒ DE song song BC Bài 62 Cho tam giác ABC, H hình chiếu A BC M điểm AH BM CM cắt AC, AB D E Chứng minh ∠AHD = ∠AHE LỜI GIẢI Qua M kẻ đường thẳng song song với BC cắt cạnh AC AB P Q, cắt HD HE I J PQ song song BC, theo định lí Thales MI BH MJ HC ⇒ = , = MP BC MQ BC Chia hai vế ta có ⇒ MI MQ BH = (1) MP MJ HC Liên hệ tài liệu word toán SĐT (zalo): 039.373.2038 42 Website: tailieumontoan.com Mặt khác, theo định lí Thales ⇒ Thay vào (1) ⇒ MQ HB = MP HC MI = ⇒ MI = MJ MJ ⇒ tam giác HIJ cân H ⇒ ∠MHI = ∠MHJ ⇒ ∠AHD = ∠AHE Bài 63 Cho tam giác ABC, đường tròn ngoại tiếp tâm O, H trực tâm tam giác Đường trung trực AH cắt AB, AC D E Chứng minh A tâm đường tròn bàng tiếp tam giác ODE LỜI GIẢI Theo giả thiết H trực tâm ⇒ AH ⊥ BC ⇒ ∠BAH = 90o − ∠ABC DE trung trực AH ⇒ DA = DH ∠DHA = ∠DAH = ∠BAH = 90o − ∠ABC OC O tâm đường tròn ngoại tiếp ∆ABC ⇒ OA = Theo tính chất góc tâm ta có ∠AOC = 2∠ABC ∆OAC cân O ⇒ ∠OAC = ∠OCA = 180o − ∠AOC ) = 90o − ∠ABC ( ⇒ ∠DHA = ∠DAH = ∠OAC = ∠OCA ⇒ ∆AHD ∆ACO đồng dạng (g.g) ⇒ AD AH ∠OAC = Vì ∠DAH = AO AC ⇒ ∠DAO = ∠HAC ⇒ ∆DAO ∆HAC đồng dạng (c.g.c) ⇒ ∠ADO = ∠AHC = 180o − ∠ABC ⇒ ∠ODB = ∠ABC DE trung trực AH ⇒ DE // BC ⇒ ∠ADE = ∠ABC ⇒ ∠ODB = ∠ADE ⇒ DA phân giác ngồi góc ∠ODE Tương tự EA phân giác ngồi góc ∠OED ⇒ A tâm đường trịn bàng tiếp tam giác ODE Bài 64 Cho tam giác ABC Gọi D, E, F trung điểm BC, CA, AB Đường phân giác góc ∠ADC cắt cạnh AC M Đường phân giác góc ∠ADB cắt cạnh AB N MN cắt AD O, FO cắt AC P, OE cắt AB Q Chứng minh PQ = AD LỜI GIẢI Theo tính chất đường phân giác DB = DC Liên hệ tài liệu word toán SĐT (zalo): 039.373.2038 43 Website: tailieumontoan.com ⇒ AM AD AD AN = = = MC DC DB NB ⇒ MN song song với BC ⇒ OM = ON Áp dụng tính chất đường trung tuyến tam giác ta PQ song song với EF Áp dụng định lí Menelaus với tam giác AMN, ba điểm Q, O, E thẳng hàng ⇒ QA ON EM QA EA = = 1⇒ QN OM EA NQ EM ⇒ AN AQ EA EC MC = −= −= −= NQ NQ EM EM EM ⇒ EM NQ CE AQ +1 = +1 ⇒ = MC AN MC AN Theo định lí Thales, ta có AQ QP = AB BC Từ tính chất đường phân giác suy (1) ( 2) BN AN = BD AD ( 3) Nhân ba đẳng thức (1), (2) (3) ta CE AQ BN AQ QP AN CE.BN BC QP = ⇒ = MC AB BD AN BC AD MC AB.BD AD Mà BC 2= = BD, CE AC BN CM = , AB CA ⇒ CA.BN = AB.CM ⇒ 2CE.BN = AB.CM Thay vào vế trái CE.BN BC AB.CM BD QP 1⇒ ⇒ QP = = = = AD MC AB.BD MC AB.2 BD AD Bài 65 Cho hình vng có độ dài cạnh a Trên cạnh lấy điểm tạo thành tứ giác Chứng minh chu vi tứ giác không nhỏ 2a LỜI GIẢI Gọi M, N, P, Q điểm nằm bốn cạnh hình vng ABCD Từ bốn đỉnh tứ giác hạ đường vng góc với đường chéo AC: MH ⊥ AC , NK ⊥ AC , PE ⊥ AC , QG ⊥ AC ⇒ tứ giác MNKH, PQGE hình thang vuông ⇒ MN ≥ HK , PQ ≥ EG Mặt khác MQ ≥ MH + QG, NP ≥ NK + PE Các tam giác AMH, CNK, CPE, AQG tam giác vuông cân ⇒ MH= AH , NK = CK , PE = CE , QG = AG MN + NP + PQ + QM ≥ HK + NK + PE + EG + QG + MH Liên hệ tài liệu word toán SĐT (zalo): 039.373.2038 44 Website: tailieumontoan.com = HK + CK + CE + EG + AG + AH = AC Dấu xảy M, N, P, Q trung điểm cạnh hình vng Vậy MN + NP + PQ + QM ≥ 2a Chu vi tứ giác 2a M, N, P, Q trung điểm cạnh hình vng Nhận xét: Bài tốn thay hình vng hình chữ nhật (hoặc hình bình hành) biết độ dài hai cạnh a, b: Tứ giác MNPQ có bốn đỉnh nằm cạnh hình chữ nhật ABCD Gọi I, O, J trung điểm MN, MP PQ ⇒ MN = BI , PN = 2OI , PQ = DJ , QM = 2OJ ⇒ MN + NP + PQ + QM= ( BI + IO + OJ + JD ) BD ≥ BD = a + b Dấu xảy M, N, P, Q trung điểm AB, BC, CD, DA Khi ABCD hình vng, a = b ta có kết Trường hợp hình bình hành hồn tồn tương tự Bài 66 Cho tam giác ABC, đường cao AH, phân giác AD thoả mãn AC − AB = 36, DC − DB = 24 Tính HC − HB LỜI GIẢI Đặt AB = x, DB = y ⇒ AC = x + 36, DC = y + 24 Theo tính chất đường phân giác ta có: AC CD = AB DB ⇒ x + 36 y + 24 x = ⇒ = ⇒ 2x = 3y x y y Mặt khác AH = AC − HC = AB − HB ⇒ AC − AB = HC − HB ⇒ ( AC − AB )( AC + AB ) = ( HC − HB )( HC + HB ) Thay vào cho ta kết ( x + 36 − x )( x + 36 + x )= ( HC − HB )( 2y + 24 ) , thay 36 ( 2x + 36 ) 36.3 ( y + 12 ) 2x =3y ⇒ HC − HB = = =54 2y + 24 ( y + 12 ) Bài 67 Cho tam giác ABC, BC = 8, CA = 7, AB = Trên AC lấy điểm D cho CD = AB Trên AB kéo dài phía A, lấy điểm E cho BE = CA Đường thẳng ED cắt cạnh BC F Tính độ dài BF LỜI GIẢI Liên hệ tài liệu word toán SĐT (zalo): 039.373.2038 45 Website: tailieumontoan.com Theo giả thiết ta có AD = AC − CD = − = AE = BE − AB = CA − AB = Qua A kẻ đường thẳng song song với EF cắt BC I Theo định lí Thales ta có BI AB IF AD , = = = = IF AE FC DC ⇒ BI IF FC = = =k 5 ⇒ BI= 5k, IF= 2k, FC = 5k ⇒ BC = BI + IF + FC = 5k + 2k + 5k = 12k ⇒K= 2 10 = ⇒ BI = k = , IF = 2k = 12 3 3 ⇒ BF = BI + IF = 14 Bài 68 Cho tam giác vuông ABC vuông A Đường phân giác BD CE cắt I Chứng minh BI ID AB = CI IE AC LỜI GIẢI giác BD phân giác CE cắt I Theo giả thiết ∠BAC =phân 90o , ⇒ ∠BIE = ∠CID = 45o ⇒ tam giác BAI tam giác BIE đồng dạng (g.g) ⇒ AI AB = IE IB Tương tự ID CI = AI CA Nhân hai đẳng thức với ta có ID AB.CI BI ID AB = ⇒ = IE IB.CA CI IE AC Bài 69 Cho tam giác nhọn ABC, đường cao BD CE P hình chiếu D AB, Q hình chiếu P BC PQ cắt BD K Chứng minh EK song song AC (Vòng loại Sharygin 2016 lớp 8) Liên hệ tài liệu word toán SĐT (zalo): 039.373.2038 46 Website: tailieumontoan.com LỜI GIẢI Gọi H trực tâm tam giác ABC Q hình chiếu P BC ⇒ AH vng góc với BC ⇒ AH song song PQ DP vng góc với AB ⇒ DP song song CE Theo định lí Thales ta có: BK BP BH BE = = BD BP BH BA Từ ta có đẳng thức: BK BK BH BP BE BE = = = BD BH BD BA BP BA Theo định lí Thales đảo ta có EK song song với AC Bài 70 Cho tứ giác ABCD, ∠A =90o , ∠C =∠D, BC =2 AD Chứng minh phân giác góc ∠BDC vng góc với AD LỜI GIẢI Gọi M giao điểm AD BC, N điểm đối xứng D qua A ⇒ AN = AD Theo giả thiết ta có: ∠A= 90o ⇒ tam giác BDN tam giác cân B ⇒ BA phân giác góc ∠DBN ∠D Giả thiết ∠C = ⇒ ∠MCD = 180o − ∠ = C 180o − ∠ = D ∠MDC ⇒ tam giác MCD cân M ⇒ MD = MC Mặt khác BC = AD ⇒ BC = DN ⇒ MC MD = CB DN ⇒ DC song song BN ⇒ ∠DBN = ∠BDC 1 DE phân giác ∠BDC ⇒ ∠DBN = ∠BDC ⇒ ∠ABD = ∠BDE 2 ⇒ AB song song DE Mà BA vng góc AD ⇒ DE vng góc với AD Bài 71 Cho tứ giác ABCD có ∠ABD = 60o , ∠DBC = 20o , ∠CDB = 30o , ∠BDA = 40o Chứng minh tam giác ABC cân LỜI GIẢI • Cách Xét tam giác ABD, theo giả thiết ta suy Liên hệ tài liệu word toán SĐT (zalo): 039.373.2038 47 Website: tailieumontoan.com o ∠BAD = 180o − ∠ABD − ∠ADB = 180o − 60o − 40 = 80o Theo giả thiết ∠ABD = 60o Dựng tam giác ABP (P BD) ⇒ ∠PAD = ∠BAD − ∠BAP = 20o Và ∠APD = 180o − ∠APB = 120o Kéo dài AP cắt BC Q giả thiết cho ∠DBC = ∠BPQ =mà 120o 20o ⇒ tam giác PAD tam giác PBQ (g.c.g) ⇒ AD = BQ, PD = PQ ⇒ tam giác PDQ Giả thiết cho ∠BDC = 30o ⇒ DC trục đối xứng PQ ⇒ CP = CQ Ta có ∠AQB = 180o − 2∠PQC = 100o = 180o − ∠ABQ − ∠BAQ = 40o ⇒ ∠PCQ Mặt khác ∠BPC = 180o − ∠DPC = 80o ⇒ tam giác BPC cân B ⇒ BP = BC ⇒ BC = AB • Cách Xét tam giác ABD, theo giả thiết ta suy o ∠BAD = 180o − ∠ABD − ∠ADB = 180o − 60o − 40 = 80o Tam giác BCD: ∠BCD = 180o − ∠CBD − ∠CDB = 130o Từ A kẻ đường thẳng vng góc với CD cắt CD H, cắt BD P BC Q ⇒ ∠DCQ = 180o − ∠BCD = 50o ⇒ ∠CQH = 90o − ∠DCQ = 40o ⇒ ∠BAQ = 180o − ∠ABQ − ∠BQA = 60o ⇒ tam giác ABP ⇒ ∠QPD = 60o ⇒ tam giác APD tam giác BPQ (g.c.g) ⇒ AD = BQ, PD = PQ ⇒ tam giác PDQ ⇒ PD = DQ ⇒ CD trục đối xứng PQ ⇒ CP = CQ ⇒ tam giác CPQ tam giác cân ⇒ ∠PCQ = 180o − 2∠CQP = 100o ⇒ ∠PCB = 80o Liên hệ tài liệu word toán SĐT (zalo): 039.373.2038 48 Website: tailieumontoan.com ⇒ ∠BPC = 180o − ∠DPQ − ∠QPC = 80o ⇒ Tam giác BPC cân B ⇒ BP = BC ⇒ BC = AB Bài 72 Cho tam giác ABC, phân giác BD CE cắt I Đường thẳng đối xứng với DC qua BD cắt BC M Đường thẳng đối xứng với EB qua BD cắt BC N Đường thẳng MD NE cắt P Chứng minh PI vng góc với BC LỜI GIẢI Theo giả thiết ⇒ tam giác BAD tam giác BMD (c.g.c) (1) ⇒ ∠IAB = ∠IMB = ∠IAD = ∠IMD ∠IAC = ∠INE = ∠IAE Tương tự ∠INC = ( 2) ⇒ ∠INM = ∠IMN ⇒ tam giác IMN cân I ⇒ I tâm đường tròn nội tiếp tam giác cân MPN (PM = PN) ⇒ PI vng góc với BC Bài 73 Cho tam giác ABC vuông A, ∠ABC = 50o D điểm AC cho ∠DBC = 10o Đường trung trực cạnh BC cắt AC E Tính tỉ số AE DC LỜI GIẢI Theo giả thiết ∠ABC = 50o ∠BAC = 90o o ⇒ ∠ACB = 90o − ∠ABC = 90o − 50= 40o Gọi H trung điểm BC ⇒ HB = HC ⇒ EB = EC ⇒ tam giác BEC cân E ⇒ ∠EBC =40o HE cắt BD N ⇒ ∠DBC = ∠NCB = 10o ⇒ ∠DCN = ∠DCH − ∠NCH = 30o Hạ DK vng góc với CN Ta có ∠DNC = 2∠NBC = 20o , DK = DC ⇔ ∠NDK = 90o − 20o = 70o ⇒ ∠ABE = ∠BAC − ∠BEC =− 90o 80o = 10o Trung trực BN cắt AB M ⇒ MB = MN ⇒ ∠MBN = ∠MNB = 40o ⇒ hai tam giác EBC MBN đồng dạng (g.g) ⇒ BE BC = ⇒ hai tam giác MBE NBC đồng dạng (g.c.g) BM BN Liên hệ tài liệu word toán SĐT (zalo): 039.373.2038 49 Website: tailieumontoan.com ⇒ ∠MEB = ∠NCB = 10o ⇒ ∠AME = 2∠MBE = 20o ⇒ ME = MB = MN o Mà ∠EMN = 180o − ∠AME − ∠NMB = 180o − 20o − 100 = 60o ⇒ tam giác MNE ⇒ ME = NE Xét tam giác NDE có ∠NDE =50o , ∠DNE =80o ⇒ ∠NED =50o ⇒ NDE cân N ⇒ ND = NE = ME ⇒ hai tam giác AME KND ⇒ DK = AE ⇒ DC = AE ⇒ AE = DC Bài 74 Cho tam giác ABC, AB = AC Đường phân giác BD thoả mãn BC = BD + AD Tính số đo góc tam giác ABC LỜI GIẢI Trên BC lấy điểm E cho BD = BE Từ giả thiết BC = BD + AD = BE + AD ⇒ EC = AD AD phân giác ⇒ ⇒ AD AB CA = = DC CB CB CE AC = CD BC ⇒ tam giác CED đồng dạng với tam giác CAB ⇒ tam giác CED cân E ⇒ ∠ECD = ∠CDE ∠ABD = ∠DBC ⇒ ∠DCE = 2∠DBC BD = BE ⇒ ∠BDE = ∠BED = 180o − ∠DBC , ∠CED = 180o − 2∠ECD = 180o − 4∠DBC 180o − ∠DBC ⇒ ∠DEB += ∠DEC + 180o − 4= ∠DBC 180o ⇒ 9∠DBC = 180o ⇒ ∠DBC = 20o ⇒ ∠ABC = ∠ACB = 40o ⇒ ∠BAC = 100o Bài 75 Cho tam giác cân ABC (AB = AC) có trung tuyến AD phân giác BE thoả mãn BE = 2AD Tính góc tam giác ABC Từ tính cos 36o (Khơng dùng máy tính bỏ túi) LỜI GIẢI Tam giác cân ABC, AB = AC ⇒ trung tuyến AD đồng thời đường cao phân giác tam giác xuất phát từ đỉnh A Từ D kẻ đường thẳng song song với BE cắt AC I DB = DC Liên hệ tài liệu word toán SĐT (zalo): 039.373.2038 50 Website: tailieumontoan.com ⇒ DI = BE Giả thiết cho BE = 2AD ⇒ DI = AD ⇒ tam giác ADI cân D ⇒ ∠DAI = ∠DIA = ∠BEA Góc ∠BEA góc ngồi tam giác BEC ⇒ ∠BEA = ∠EBC + ∠BCE = ( 90o − ∠BAD ) + ( 90o − ∠CAD ) ⇒ 2∠BEA = 270o − 3∠BEA ⇒ ∠BEA = 54o ⇒ ∠BAC = 108o , ∠ABC = ∠ACB = 36o Trung trực AB cắt cạnh BC M ⇒ tam giác MAB cân M ⇒ MA = MB ∠MAC = ∠BAC − ∠BAM = 72o , ∠AMC = 2∠B = 72o ⇒ ∠MAC = ∠AMC ⇒ tam giác CAM cân C ⇒ CM = CA Tam giác ABC tam giác MAB đồng dạng (g.g) ⇒ AB MA = ⇒ AB = BC.MA = BC.MB = BC ( BC − MC ) BC AB = BC ( BC − AC= ) BC ( BC − AB=) BC − BC AB Chia hai vế cho AB2 ta có: BC BC BC + BC + − −1 = ⇒ = ⇒ cos 36o = = 2 AB AB AB AB Bài 76 Cho tam giác ABC thoả mãn 2∠B + 3∠C = 180o số đo cạnh ba số nguyên liên tiếp Tính độ dài cạnh tam giác ABC LỜI GIẢI Theo giả thiết 2∠B + 3∠C = 180o , từ ta có 2∠B + 3∠C = ∠A + ∠B + ∠C ⇒ ∠A = ∠B + 2∠C ⇒ góc ∠A lớn ⇒ cạnh BC lớn Trên cạnh BC lấy điểm D cho CD = AC 180o − ∠C ⇒ ∠ADC = ∠DAC = 180o − ∠C ⇒ ∠ADB = 180 − ∠ADC = 180 − o o Thay 2∠B + 3∠C = 180o vào ta nhận ∠ADB = 180o − 2∠B + 3∠C − ∠C = 180o − ( ∠B + ∠= C ) ∠A Liên hệ tài liệu word toán SĐT (zalo): 039.373.2038 51 Website: tailieumontoan.com ⇒ tam giác BDA đồng dạng với tam giác BAC (g.g) ⇒ BC AB AB AB = = = AB BD BC − CD BC − AC ⇒ BC − BC AC = AB ⇒ AB = BC ( BC − AC ) Theo giả thiết số đo ba cạnh nguyên liên tiếp Theo chứng minh trên, cạnh BC lớn ⇒ BC − AC = BC − AC = Nếu BC − AC =1 ⇒ AB = BC = AB + ⇒ AB = 2, BC = 4, AC = Nếu BC − AC =⇒ AB = BC , phương trình khơng thoả mãn đề Bài 77 Tính sin 22o30′, cos 22o30′, tan 22o30′ LỜI GIẢI Dựng tam giác vuông cân ABC A Không ảnh hưởng tới kết quả, ta đặt: AB = AC = 1, A = 90o ⇒ BC = Theo tính chất đường phân giác Gọi AD phân giác góc B ⇒ AD AB AD AB = ⇒ = DC BC AC BC + AB = − +1 ⇒ AD = Áp dụng định lý Pythagos ⇒ BD = AB + AD ⇒ BD =+ ( ) ( 2 − =− 2 ⇒ BD = 2 − ( ) ) 2 −1 −1 2− AD ⇒ sin 22o30′ == = = BD 2 − 2− ) ( cos 22o= 30′ AB = BD tan 22o30 =′ sin 22o30′ = cos 22o30′ = 2− ( ) ( ) 2+ 2 − Bài 78 Cho tam giác ABC, ∠BAC = 30o , đường phân giác AD Chứng minh 6+ 2 = + AD AB AC LỜI GIẢI Trước hết ta tính sin15o Dựng tam giác vuông ABC: ∠BAC =90o , ∠ABC =30o , AC =1 Liên hệ tài liệu word toán SĐT (zalo): 039.373.2038 52 Website: tailieumontoan.com ⇒ BC= 2, AB= Dựng phân giác AD ⇒ ⇒ AD = 2+ AD AB AD 3 = = ⇒ = DC BC AD + DC + = −3 ( ⇒ BD = AB + AD = + 12 + − 12 = 12 − ⇒ BD = ( ) −1 = ( ) ) −1 ( )( ) 2− 3 +1 AD −3 6− ⇒ sin15o = = = = BD ( − 1) −1 ( ) Từ trở lại toán: S ABC = S ABD + S ADC ⇒ 1 AB AC.sin 30o = AB AD.sin15o + AC AD.sin15o 2 6+ 2 ⇒ AB AC =AD.sin15o ( AB + AC ) ⇒ = + AD AB AC Bài 79 Cho tam giác ABC ( AB ≠ AC ) Gọi D trung điểm BC, AE, AF thứ tự đường phân giác ngồi góc ∠BAC , H hình chiếu A BC Chứng minh EF DH = AB AC LỜI GIẢI Giả sử AB < AC AE, AF đường phân giác ngồi góc ∠BAC , theo tính chất đường phân giác EB FB AB = = EC FC AC EB AB EB AB ⇒ = ⇒= EC + EB AC + AB EC AC + AB AB.BC FB AB AB.BC ⇒= EB , = ⇒= FB AC + AB FC − FB AC − AB AC − AB AB.BC AB.BC AB.BC.CA ⇒ EF = EB + BF = + = (1) AC + AB AC − AB AC − AB Áp dụng định lí Pythagore tam giác ACH, ABH: ⇒ BC AC = AH + CD = AH + + DH 2 2 2 BC AB = AH + BH = AH + − DH ⇒ AC − AB = BC.DH 2 2 Liên hệ tài liệu word toán SĐT (zalo): 039.373.2038 53 Website: tailieumontoan.com AC − AB ( 2) ⇒ DH = BC AB.BC.CA AC − AB Từ (1) (2) suy EF DH = = AB AC AC − AB 2 BC Liên hệ tài liệu word tốn SĐT (zalo): 039.373.2038 54 Website: tailieumontoan.com III ĐƯỜNG TRỊN TỨ GIÁC NỘI VÀ NGOẠI TIẾP Đường tròn tứ giác nội, ngoại tiếp kiến thức sách giáo khoa phổ thông, sử dụng nhiều kì thi, kể kì thi quốc tế Chỉ với kiến thức sách giáo khoa trung học đủ để giải đề thi nước quốc tế Riêng phần xin nhắc lại bổ sung kiến thức Bài tốn quỹ tích cung chứa góc Bài tốn Cho đoạn thẳng AB góc α ( 0° < α < 180° ) Tìm quỹ tích (tập hợp) điểm M thỏa mãn ∠AMB = α Kết luận: Với đoạn thẳng AB góc α ( 0° < α < 180° ) cho trước quỹ tích điểm M thỏa mãn α hai cung chứa góc α dựng đoạn AB ∠AMB = - Hai cung chứa góc α hai cung đối xứng qua AB - Hai điểm A B coi thuộc quỹ tích Am ′B hai nửa đường trịn đường kính AB Quỹ tích điểm nhìn - Khi α= 90° hai cung AmB đoạn thẳng AB cho trước góc vng đường trịn đường kính AB cung chứa góc α cung AnB chứa góc 180° − α - Cung AmB Tiêu chuẩn để tứ giác nội tiếp đường tròn *Tiêu chuẩn Tứ giác ABCD nội tiếp ∠A + ∠C = ∠B + ∠D = 180° *Tiêu chuẩn Tứ giác ABCD nội tiếp ∠ABD = ∠ACB *Tiêu chuẩn Điều kiện cần: Cho đường tròn ( O ) điểm M khơng thuộc đường trịn Từ M kẻ hai cát tuyến tùy ý MAB MCD Chứng minh MA.MB = MC.MD Liên hệ tài liệu word toán SĐT (zalo): 039.373.2038 Website: tailieumontoan.com Điều kiện đủ: Cho hai đường thẳng d1 d cắt điểm M Trên d1 lấy hai điểm phân biệt A, B d lấy hai điểm C, D khác thỏa mãn: - MA.MB = MC.MD - Điểm M nằm đoạn AB CD điểm M nằm đoạn AB, CD Khi bốn điểm A, B, C, D thuộc đường tròn Chứng minh: Giả sử A, B, C, D nằm đường tròn ⇒ tam giác MAD tam giác MCB đồng dạng ( ∠MBC = ∠MDA ∠AMD = ∠MCB ) ⇒ MA MC = MD MB ⇒ MA.MB = MC.MD Ngược lại: Giả sử MA.MB = MC.MD Qua ba điểm A, B, C xác định đường tròn cắt MC E Theo phần thuận MA.MB = MC.ME ⇒ MC.MD = MC.ME ⇒ MD = ME ⇒ D ≡ E ⇒ A,B,D,C nằm đường tròn *Tiêu chuẩn Định lí Ptoleme Tứ giác ABCD nội tiếp đường tròn AB.CD + AD.BC = AC.BD Chứng minh: Thuận: Tại đỉnh A dựng phía ngồi góc ∠BAx = ∠CAD Tia Ax cắt cạnh BC E Tứ giác ABCD nội tiếp ⇒ ∠EBA = ∠ADC ⇒ Hai tam giác ABE, ADC đồng dạng (g.g) ⇒ EB AB AB.CD (1) = ⇒ EB = CD AD AD ∠ADC = ∠ACB ⇒ hai tam giác EAC, BAD đồng dạng (g.g) ⇒ EC AC AC.BD = ⇒ EC = (2) BD AD AD Mặt khác EC = EB + BC Thay (1), (2) vào ⇒ BD.AC AB.CD = + BC AD AD ⇒ BD.AC = AB.CD + BC.AD Ngược lại, xét AB.CD + AD.BC = AC.BD Giả sử tứ giác ABCD không tứ giác nội tiếp ⇒ C khơng thuộc đường trịn ngoại tiếp tam giác ABD Liên hệ tài liệu word toán SĐT (zalo): 039.373.2038 Website: tailieumontoan.com E thỏa mãn ∠EAB = ∠CAD, ∠AEB = ∠ADC ⇒ E ∉ BC ⇒ EC < EB + BC Thay (1) (2) vào ta được: AB.CD + AD.BC > AC.BD Người ta thường gọi bất đẳng thức hình học bất đẳng thức Ptolemy Nhận xét: Tam giác ABC vuông A Dựng hình chữ nhật ABDC Theo định kí Ptolemy ta có: AC.BD + AB.CD= BC AD ⇒ AC + AB= BC ⇒ định lí Pythagoras trường hợp đặc biệt định lí Ptolemy Tiêu chuẩn tứ giác ngoại tiếp đường tròn Tứ giác ABCD ngoại tiếp đường tròn AB + CD = AD + BC Chứng minh: Điều kiện cần: Giả sử tứ giác ABCD ngoại tiếp đường tròn ( I;r ) Gọi E, F, H, K hình chiếu I cạnh AB, BC, CA, AD Theo tính chất hai tiếp tuyến xuất phát từ điểm: ⇒ AE = AK, BE = BF, CF = CH, DH = DK Cộng đẳng thức ta có: AE + BE + CH + DH = BF + CF + DK + AK ⇒ AB + CD = AD + BC Điều kiện đủ: Tứ giác ABCD thỏa mãn AB + CD = AD + BC Ta ln xác định đường trịn (I) tiếp xúc với ba cạnh tứ giác I giao hai đường phân giác góc tạo ba cạnh liên tiếp Giả sử ba cạnh AB, BC, CD Kẻ đường thẳng từ A tiếp xúc với đường tròn ( I;r ) cắt cạnh CD D′ Từ điều kiện cần, ta có AB + CD′ = AD′ + BC ⇒ DD′ =AD − AD′ (mâu thuẫn với bất đẳng thức tam giác) ⇒ đường tròn (I) nội tiếp tứ giác ABCD Liên hệ tài liệu word toán SĐT (zalo): 039.373.2038 Website: tailieumontoan.com CÁC BÀI TOÁN ÁP DỤNG Bài Cho tam giác nhọn ABC, D hình chiếu A cạnh BC Xác định điểm E cạnh AB để E thuộc đường tròn qua A, C, D LỜI GIẢI Theo giả thiết D hình chiếu A cạnh AC ⇒ AD vng góc BC ⇒ tam giác ADC vuông D Gọi I trung điểm cạnh AC Theo tính chất tam giác vng ta có IA = ID = IC Điểm E cạnh AB cho E thuộc đường tròn qua A, C, D Suy E thỏa mãn IE = IA = IC Do I trung điểm AC ⇒ EI trung tuyến tam giác AEC ⇒ IE =AC ⇒ tam giác AEC vuông E Nghĩa AE vng góc với CE ⇒ E chân đường cao từ C đến cạnh AB Bài Cho tam giác nhọn ABC Gọi D, E, F trung điểm cạnh BC, CA, AB, H trực tâm tam giác ABC, K chân đường cao kẻ từ A, J trung điểm AH Chứng minh điểm D, E, F J, K nằm đường trịn Xác định tâm đường trịn LỜI GIẢI Theo giả thiết = DB DC,EC = EA, = FA FB ⇒ DE,EF,FD đường trung bình tam giác ABC AB EF song song BC ⇒ DE = AK vng góc BC ⇒ KF đường trung tuyến tam giác AKB ⇒ KF = AB ⇒ DE = KF mà KF không song song với DE ⇒ tứ giác FEDK hình thang cân ⇒ K thuộc đường tròn ngoại tiếp tam giác DEF Tam giác KJD tam giác vuông Gọi I trung điểm DJ ⇒ IJ = IK = ID ⇒ I nằm trung trực KD Giả thiết = JA JH, = EA EC Liên hệ tài liệu word toán SĐT (zalo): 039.373.2038 Website: tailieumontoan.com ⇒ EJ song song CH ⇒ EJ vng góc AB Mà DE song song AB ⇒ JE vng góc với DE ⇒ tam giác DEJ vuông E ⇒ IJ = IE = ID ⇒ IJ = IF = IK = ID = IE ⇒ I cách điểm D, E, F, J, K I tâm đường trịn Với cách chứng minh chân ba đường cao, trung điểm ba cạnh trung điểm ba đoạn thẳng nối từ trực tâm đến ba đỉnh nằm đường trịn Đường trịn có tên đường trịn điểm Euler Bài Cho ba điểm A, B, C cố định nằm đường thẳng theo thứ tự Đường trịn tâm O thay đổi ln qua A, C Qua B kẻ cát tuyến vng góc với OA cắt đường tròn O P Q Chứng minh P Q thuộc đường tròn cố định LỜI GIẢI Gọi H giao điểm AO với PQ Theo giả thiết, BH vng góc OA, OP HQ = OQ ⇒ tam giác OPQ cân O ⇒ HP = AO cắt (O) D ⇒ AD đường kính ⇒ DP = DQ Theo hệ thức tam giác vng ta có: AP = AH.AD (1) Mặt khác ∠BHD = ∠BCD = 90° ⇒ B,H,D,C nằm đường trịn đường kính BD ⇒ tam giác ACD tam giác AHB hai tam giác vuông đồng dạng (g.g) ⇒ AC AH = AD AB ⇒ AH.AD = AC.AB (2) Từ (1) (2) ta có AP = AB.AC ( ) Mà AP = AQ ⇒ P,Q thuộc đường tròn cố định A, AB.AC Bài Cho đường tròn ( O, R ) điểm P cố định đường trịn Một góc vng quay quanh P cắt đường tròn tâm (O) A B Gọi D trung điểm AB H hình chiếu P AB Chứng minh góc vng quay quanh P tổng bình phương DP + DO HP + HO khơng đổi Từ suy H, D nằm đường thẳng cố định LỜI GIẢI Liên hệ tài liệu word toán SĐT (zalo): 039.373.2038 Website: tailieumontoan.com Theo giả thiết ∠APB= 90° ⇒ tam giác APB vuông Cũng theo giả thiết DA = DB ⇒ PD trung tuyến tam giác APB ⇒ PD = DB = AB OD vng góc với AB Theo định lí Pythagoras: DO =OB2 − BD =R − ⇒ DO + DP =R − AB2 AB2 AB2 AB2 + PD =R − + =R 4 Áp dụng định lí Pythagoras cho tam giác HPD tam giác HOD, ta có: HP = PD − HD ,HO =HD + OD = HD + OB2 − BD ⇒ HP + HO = PD + OB2 − BD = R Từ ta có DP + DO = HP + HO = R Gọi I trung điểm OP Theo công thức đường trung uyến tam giác, ta có: ID ) ( DP + DO − OP 2R − OP = 4 2R − OP I cố định ID = không đổi ⇒ H D nằm đường trịn tâm I bán kính 2R − OP góc vng quay quanh điểm P Bài Cho hai đường tròn ( O1 ,r1 ) đường tròn ( O ,r2 ) tiếp xúc nhau, đồng thời tiếp xúc với đường thẳng d A B Chứng minh AB=2 r1r2 LỜI GIẢI Giả sử r1 ≥ r2 Ta có O1A,O B vng góc với đường thẳng d Từ O kẻ đường thẳng vuông góc với O1A E ⇒ tam giác O1O E vng góc E O1O = r1 + r2 , O1E = r1 − r2 , O E = AB Theo định lí Pythagoras: EO = O1O 2 − O1E 2 ⇒ EO 2 = ( r1 + r2 ) − ( r1 − r2 ) = 4r1r2 ⇒ AB = r1r2 2 Liên hệ tài liệu word toán SĐT (zalo): 039.373.2038 Website: tailieumontoan.com Bài Cho nửa đường trịn đường kính AB = 2R M điểm tùy ý nửa đường tròn AM, BM cắt nửa đường tròn C D Chứng minh AM.AC + BM.BD không phụ thuộc vào vị trú điểm M LỜI GIẢI Theo giả thiết AM cắt nửa đường tròn C ⇒ ∠ACB= 90° Từ M hạ MH vng góc AB ⇒ ∠AHM= 90° ⇒ tam giác AHM tam giác ACB hai tam giác vuông đồng dạng (g.g) ⇒ AH AM = AC AB ⇒ AH.AB = AM.AC Tương tự BM.BD = BH.BA Cộng hai đẳng thức AM.AC + BM.BD = AH.AB + BH.AB = AB2 = 4R : không đổi BP DQ AB Bài Cho hình vng ABCD Điểm P BC điểm Q CD cho = = Chứng minh PQ tiếp xúc với đường trịn tâm A bán kính AB LỜI GIẢI Theo giả thiết BP DQ AB AB = = ⇒ BP = ⇒ CP = AB − BP = 2AB AB , DQ = = CQ Tam giác vuông CPQ: 4AB2 AB2 25AB2 PQ = CP + CQ = + = 36 2 5AB ⇒ PQ = Các tam giác ABP, ADQ, CPQ tam giác vuông ⇒ S= ABP AB2 AB2 AB2 ;S= ;S = ADQ CPQ 6 ⇒ SAPQ = AB2 − SABP − SADQ − SCPQ = Kẻ AM vng góc PQ ⇒ SAPQ = 5AB2 12 PQ.AM 5AB.AM = 12 Liên hệ tài liệu word toán SĐT (zalo): 039.373.2038 Website: tailieumontoan.com ⇒ 5AB.AM 5AB2 = 12 12 ⇒ AM =AB ⇒ PQ tiếp xúc với đường tròn tâm A bán kính AB Bài Cho đường trịn tâm ( O ) dây cung AB M điểm AB Đường tròn ( O1 ) qua A, M tiếp xúc với ( O ) , đường tròn tâm ( O ) qua M, B tiếp xúc với ( O ) Hai đường tròn ( O1 ) ( O ) cắt điểm điểm thứ hai N Chứng minh tam giác MON vuông LỜI GIẢI Theo giả thiết đường tròn ( O1 ) qua A, M đồng thời tiếp xúc với ( O ) , đường tròn ( O1 ) qua M, B tiếp xúc với ( O ) , ( O ) tiếp xúc với ( O1 ) A ⇒ O,O1 ,A thẳng hàng O,O ,B thẳng hàng ⇒ ∠OAB = ∠OBA ∠O1AM = ∠O1MA ⇒ ∠OBA = ∠O1MA ⇒ O1M song song OB Tương tự O M song song OA ⇒ tứ giác OO1MO hình bình hành Gọi I giao điểm OM O1O ⇒ I trung điểm OM Theo tính chất hai đường trịn giao nhau, ta có: MN vng góc O1O cắt trung điểm ⇒ O1O song song ON ⇒ ∠MON= 90° ⇒ tam giác MON vuông N Bài Cho đường tròn ( O,R ) , đường thẳng cố định a cắt đường tròn ( O ) A, B M điểm thay đổi a nằm ( O )( MA < MB ) Qua M kẻ tiếp tuyến MC, MD Chứng minh M thay đổi a đường CD qua điểm cố định LỜI GIẢI Theo giả thiết MC, MD tiếp tuyến ⇒ OC vuông góc MC, OD vng góc MD ⇒ MO phân giác góc ∠CMD Liên hệ tài liệu word tốn SĐT (zalo): 039.373.2038 Website: tailieumontoan.com Tam giác OCM tam giác vng, OM vng góc CD ⇒ R = OC2 = OH.OM (1) Gọi I trung điểm AB Kéo dài CD cắt OI K Tam giác OHK tam giác OIM hai tam giác vuông có góc nhọn ∠MOK chung ⇒ hai tam giác đồng dạng ⇒ OK OH OM.OH Từ (1) = ⇒ OK = OM OI OI R2 ⇒ OK = OI Do OI khơng đổi, đường trịn ( O,R ) cố định ⇒ OK không đổi ⇒ CD qua điểm cố định D M thay đổi a Bài 10 Cho tam giác ABC, I tâm đường tròn nội tiếp D E tiếp điểm đường tròn ( I ) với cạnh AC, AB Gọi O tâm đường tròn ngoại tiếp tam giác BIC Chứng minh ∠ODC = ∠OEB (EGMO 2012) LỜI GIẢI Đường thẳng AI phân giác góc ∠BAC , AI cắt đường tròn ngoại tiếp tam giác ABC K ⇒ KB = KC = KI ⇒ K ≡ O ⇒ O tâm đường tròn ngoại tiếp tam giác BIC D, E tiếp điểm cạnh AC AB, theo tính chất tiếp tuyến ⇒ AD = AE Mặt khác, AO phâm giác ∠BAC ⇒ ∠BAC = ∠CAO Tam giác EAO tam giác DAO có cạnh AO chung ⇒ hai tam (c.g.c) ⇒ ∠ADO = ∠AEO ⇒ ∠OEB = 180° − ∠AEO = 180° − ∠ADO = ∠OEC Bài 11 Cho tam giác ABC, I tâm đường tròn nội tiếp J A ,J B ,J C tâm đường trịn bàng tiếp góc A, B, C Chứng minh I trực tâm tam giác J A J B J C Liên hệ tài liệu word toán SĐT (zalo): 039.373.2038 Website: tailieumontoan.com LỜI GIẢI Ta có ∠IBJ A = ∠IBC + ∠CBJ A = ( ∠ABC + 180° − ∠ABC )= 90° ⇒ BI vng góc BJ A Vậy J B B vng góc J A J B Tương tự, AJ A ,BJ B ,CJ C đường cao tam giác J A J B J C ⇒ I trực tâm tam giác J A J B J C Bài 12 Cho tam giác ABC D điểm cạnh BC cho bán kính hai đường tròn nội tiếp tam giác ABD ADC Chứng minh rằng= AD p (p − a ) LỜI GIẢI Gọi đường tròn nội tiếp tam giác ABC ( I,r ) , ( O1 ,r1 ) , ( O2 ,r2 ) Hạ đường thẳng IH,O1E,O K vng góc với cạnh BC ⇒ IH= r, O1E= r1 , O K= r2 Theo giả thiết bán kính đường trịn nội tiếp tam giác ABD ADC ⇒ r1 = r2 ⇒ tứ giác O1O KE hình chữ nhật ⇒ O1O song song với BC O1O = EK ⇒ O1O IH − O1E r − r1 = = BC IH r Đặt p1 = (1) 1 ( c + BD + AD ) , p2 = ( b + CD + AD ) , p = ( a + b + c ) 2 ⇒ O1O = EK = ED + DK = ( p1 − c ) + ( p − b ) = p + AD − b − c (2) r − r1 p + AD − b − c AD + a − p Từ (1) (2)= suy = r a a (3) Theo cơng thức diện tích: S = pr = ( p1 + p ) r1 , p1 + p = ( a + b + c + 2AD ) = p + AD Liên hệ tài liệu word toán SĐT (zalo): 039.373.2038 10 Website: tailieumontoan.com Thay vào (3) ta S S S a : − = p p p + AD AD + a − p ⇒ AD = p(p − a) ⇒ AD = p(p − a) Bài 13 Cho tam giác ABC Đường tròn nội tiếp tâm I tiếp xúc với cạnh BC, CA, AB D, E, F J A tâm đường trịn bàng tiếp ứng với góc ∠BAC Gọi K, M N tiếp điểm ( J A ) với cạnh BC hai cạnh AC, AB kéo dài Chứng minh BK= BN= p − c BD= BF= p − b Từ suy D K đối xứng qua trung điểm cạnh BC LỜI GIẢI Theo giả thiết D, E F tiếp điểm đường tròn nội tiếp tam giác với cạnh BC, CA AB Theo tính chất tiếp tuyến đường trịn, ta có: = BD BF,CD = CE, = AE AF ⇒ BD + CE + AF = p BC + CA + AB p ⇒ BD = BF = p − b p = ⇒ BD + CE + AE = Tương tự CD= CE= p − c AE= AF= p − a Với giả thiết K, M, N tiếp điểm đường tròn bàng tiếp góc ∠BAC tiếp xúc với cạnh BC, CA, AB K, M, N Theo tính chất tiếp tuyến với đường trịn, ta có: = AN AM, = BK BN, = CM CK ⇒ AM + AN = CA + CK + CB + BK = AC + AB + BC = 2p ⇒ AM = AN =⇒ p CM = CK = AM − AC =− p b Tương tự BN= BK= p − c Từ ta suy BD= CK= p − b ⇒ D K đối xứng qua trung điểm cạnh BC Bài 14 Cho tam giác ABC đường tròn nội tiếp tâm I tiếp xúc cạnh BC D, đường tròn bàng tiếp góc ∠BAC tâm J tiếp xúc cạnh BC F, DI cắt đường tròn ( I ) E Chứng minh A, E, F thẳng hàng Gọi H hình chiếu A BC, Từ suy ta IF qua trung điểm AH Liên hệ tài liệu word toán SĐT (zalo): 039.373.2038 11 Website: tailieumontoan.com LỜI GIẢI Gọi N tiếp điểm ( I ) với cạnh AC, K tiếp điểm ( J ) với AC ⇒ IN,JK vng góc với AC ⇒ IN JK song song ⇒ AI IN = AJ JK Mà IN = IE, JF = JK ⇒ AI IE = AJ JF Mà IE song song JF, theo định lí Thales đảo suy A, E, F thẳng hàng Ta có AH DE song song với nhau, IE = ID , theo định lí Thales ta có: AM EI == ⇒ AM = AH MH ID Vậy IF qua trung điểm AH Bài 15 Cho tam giác ABC Các điều kiện sau tương đương: 1) I tâm đường tròn nội tiếp tam giác ABC 2) I thuộc đường phân giác góc A ∠BIC= 90° + ∠BAC 3) I nằm tam giác ABC thỏa mãn 1 ∠BIC= 90° + ∠BAC, ∠AIC= 90° + ∠ABC 2 LỜI GIẢI 1) ⇔ 2) Giả sử I tâm đường tròn nội tiếp tam giác ABC, AI cắt cạnh BC D ⇒ ∠BIC = ∠BID + ∠DIC ∠BID góc ngồi tam giác ABI ⇒ ∠BID = ∠IAB + ∠IBA = = ⇒ ∠BIC= ( ∠BAC + ∠ABC ) 1 CID (180° − ∠ACB ) Tương tự ∠= (180° − ∠ABC ) 2 1 (180° + ∠BAC )= 90° + ∠BAC 2 Liên hệ tài liệu word toán SĐT (zalo): 039.373.2038 12 Website: tailieumontoan.com Ngược lại: Giả sử điểm J phân giác AD thỏa mãn ∠BJC= 90° + ∠BAC ⇒ ∠BIC = ∠BJC ⇒ I ≡ J 1 3) ⇒ 1) Từ ∠BIC= 90° + ∠BAC ∠AIC= 90° + ∠ABC 2 1 I thuộc hai cung chứa góc 90° + ∠BAC dựng BC phía với A 90° + ∠ABC dựng 2 AC phía với B ⇒ I tâm đường tròn nội tiếp tam giác ABC Bài 16 Gọi I tâm đường tròn nội tiếp tam giác ABC Đường thẳng AI cắt đường tròn ngoại tiếp tam giác ABC D Chứng minh DI = DB = DC LỜI GIẢI Theo giả thiết I tâm đường tròn nội tiếp tam giác ABC, AI cắt đường tròn ngoại tiếp D = ⇒ BD = ⇒ ∠BAD = ∠CAD ⇒ BD CD DC Xét tam giác DBI: ∠BID = ∠IAB + ∠IBA = ∠IBD = ∠IBC + ∠CBD = ( ∠BAC + ∠ABC ) , 1 ∠ABC + ∠DAC + ( ∠ABC + ∠BAC ) 2 ⇒ ∠BID = ∠DBI ⇒ tam giác DBI cân D ⇒ DB = DI ⇒ DI = DB = DC Bài 17 Gọi I tâm đường tròn nội tiếp tam giác ABC Đường thẳng AI, BI, CI cắt đường tròn ngoại tiếp tam giác ABC D, E F Đường thẳng DE cắt cạnh AC M, đường thẳng DF cắt cạng AB N Chứng minh MN song song với cạnh BC LỜI GIẢI Theo giả thiết BE phân giác góc ∠ABC ⇔ ∠ABE = ∠EBC ⇒ ∠ADE = ∠EDC Áp dụng tính chất đường phân giác, ta được: AM AD = MC DC Liên hệ tài liệu word toán SĐT (zalo): 039.373.2038 13 Website: tailieumontoan.com Tương tự phân giác CF, ta có AN AD = NB DB Theo giả thiết AD phân giác góc ∠BAC ⇒ DB = DC ⇒ AD AD AM AN = ⇒ = DB DC MC NB Theo định lí Thales đảo, ta có MN song song BC Bài 18 Cho ba đường tròn ( O1 ) , ( O ) , ( O3 ) tiếp xúc với đôi Gọi tiếp điểm ( O1 ) , ( O ) A, ( O ) , ( O3 ) B, ( O3 ) , ( O1 ) C AB, AC kéo dài cắt đường tròn ( O3 ) P Q Chứng minh P,O3 ,Q thẳng hàng LỜI GIẢI Theo giả thiết ba đường tròn ( O1 ) , ( O ) , ( O3 ) tiếp xúc với đôi tam giác O1AC,O AB,O3 BC cân tương ứng O1 ,O ,O3 ⇒ ∠AO1= C 180° − 2∠O1AC ∠AO 2= B 180° − 2∠O BA ∠BO3= C 180° − 2∠O3CB Mặt khác ∠AO1C + ∠AO B + ∠BO3= C 180° Cộng ba đẳng thức ta ∠O1AC + ∠O BA + ∠O3CB = 180° Mà ∠O BA = ∠O3 BQ, ∠O1CA = ∠O3CP ⇒ ∠BO3Q = 180° − 2∠O3 PQ = 180° − 2∠O BA Tương tự, ∠CO3= P 180° − 2∠O1AC ⇒ ∠BO3Q + ∠BO3C + ∠CO3 P = 180° − 2∠O BA − 180° − 2∠O3CB − 180° − 2∠O1AC = 180° ⇒ P,O3 ,Q thẳng hàng Bài 19 Cho ba đường tròn O1 ,O ,O3 có bán kính R cắt điểm O Gọi A, B, C giao điểm Liên hệ tài liệu word toán SĐT (zalo): 039.373.2038 14 Website: tailieumontoan.com lại ba đường tròn Chứng minh đường trịn ngoại tiếp tam giác ABC có bán kính R LỜI GIẢI Gọi giao điểm ( O1 ) ( O ) A, ( O ) ( O3 ) B, ( O3 ) ( O1 ) C ⇒ O1O vng góc AO Theo giả thiết OO1 = OO ⇒ OA trung trực O1O Tương tự, OB trung trực O O3 , OC trung trực O1O3 ⇒ ( O,R ) đường tròn ngoại tiếp tam giác O1O O3 Gọi M giao điểm O1O với AO, N giao điểm O O3 với OB E giao điểm O1O3 với OC ⇒ MN, NE, EM đường trung bình tam giác O1O O3 Dễ dàng nhận thấy AB 2MN, BC 2NE, AC 2ME = = = ⇒ tam giác O1O O3 tam giác BCA ⇒ đường tròn qua A, B, C có bán kính R Bài 20 (Định lí Miquel) Trên cạnh tam giác lấy ba điểm Chứng minh ba đường tròn ngoại tiếp tam giác tạo đỉnh hai điểm hai cạnh xuất phát từ đỉnh giao điểm LỜI GIẢI Ba điểm D, E, F cạnh BC, CA, AB Giả sử đường tròn ngoại tiếp ∆AEF ∆BDF cắt M ⇒ ∠A + ∠EMF = 180°, ∠B + ∠FMD = 180° ⇒ ∠A + ∠EMF + ∠B + ∠FMD = 360° ⇒ 180° − ∠C + 360° − ∠EMD = 360° ⇒ ∠C + ∠EMD = 180° ⇒ tứ giác EMDC nội tiếp ⇒ ba đường tròn cắt M Điểm M gọi điểm Miquel tam giác Bài 21 Gọi I tâm đường tròn nội tiếp tam giác ABC D, E thứ tự giao điểm đường thẳng BI CI với cạnh AC, AB Chứng minh tam giác ABC vuông A 2BI.CI = BD.CF Liên hệ tài liệu word toán SĐT (zalo): 039.373.2038 15 Website: tailieumontoan.com LỜI GIẢI Theo giả thiết BD, CE đường phân giác ⇒ giao điểm BD CE I Theo tính chất đường phân giác ⇒ DA BA AC.BA = ⇒ DA = DC BC BC + BA (1) I tâm đường tròn nội tiếp tam giác ABC ⇒ AI phân giác tam giác ABD ⇒ IB AB IB AB = ⇒ = ID AD BD AB + AD Thay (1) vào (2) ta (2) IB BC + AB = BD AB + BC + CA IC BC + AC IB IC Tương tự = ⇒ = CE AB + BC + CA BD CE Do BI.CI = ( BC + AB )( BC + AC ) ( AB + BC + CA ) BD.CF ⇔ ( BC + AB )( BC + AC ) = ( ⇔ BC2 + BC.AC + AB.BC + AB.AC ( AB + BC + CA ) ) = AB2 + BC2 + CA + ( AB.BC + BC.CA + CA.AB ) ⇔ BC2= AB2 + BC2 ⇔ ∠BAC= 90° Bài 22 Cho tứ giác ABCD nội tiếp đường trịn ( O ) có hai đường chéo AC BD vng góc với nhai M Chứng minh đường thẳng nối trung điểm cạnh với M vng góc cạnh đối diện ngược lại (Định lí Brahmagupta) Brahmagupta nhà tốn học, nhà thiên văn học người Ấn Độ (598-660) Tác phẩm ông phần lớn nghiên cứu đại số số học có phép giải phương trình bậc hai có nghiệm thực LỜI GIẢI Gọi I trung điểm cạnh AB MI cắt CD E Theo giả thiết AC vng góc BD ⇒ ∠ABM = ∠AMI Ta có ∠AMI = ∠CME (đ.đ) Mặt khác ABCD nội tiếp ⇒ ∠ABM = ∠MCE ⇒ ∠MAB + ∠MBA = ∠EMC + ∠ECM ⇒ ∠AMB = ∠MEC ⇒ ∠MEC = 90° Liên hệ tài liệu word toán SĐT (zalo): 039.373.2038 16 Website: tailieumontoan.com ⇒ MI vng góc với CD Bài 23 Cho tam giác ABC, ∠BAC < 60° P điểm tam giác H, K hình chiếu P cạnh AB cạnh AC thỏa mãn AC + AH = BC + BH AB + AK = BC + CK Chứng minh góc ∠BPC < 120° LỜI GIẢI Gọi O tâm đường tròn ngoại tiếp tam giác ABC M N trung điểm AB AC ⇒ OM vng góc AB ON vng góc AC Gọi I tâm đường trịn nội tiếp tam giác ABC D, E tiếp điểm đường tròn nội tiếp với cạnh AB, AC ⇒ ID vng góc AB IE vng góc AC ⇒ AD = AE = CA + AB − BC , BD = AB − AD = AB + BC − CA Từ AC + AH = BC + BH ⇒ AC + 2AH = BC + AH + BH AB + BC − CA ⇒ AH = Tương tự AB + AK = BC + CK ⇒ AK = BC + CA − AB ⇒ BD = AH ⇒ MH = MD Tương tự NE = NK O tâm đường tròn ngoại tiếp ⇒ ∠BOC = 2∠BAC I tâm đường tròn nội tiếp ⇒ ∠BIC= 90° + ∠BAC Theo giả thiết, ∠BAC < 60° ⇒ 2∠BAC < 90° + ∠BAC ⇒ ∠BOC < ∠BIC ⇒ ∠BPC < ∠BOC = 2∠BAC < 120° Bài 24 Cho tam giác ABC Gọi I, O tâm đường tròn nội, ngoại tiếp tam giác ABC, G trọng tâm tam giác ABC Chứng minh ∠AIO = 90° IG song song với BC Liên hệ tài liệu word toán SĐT (zalo): 039.373.2038 17 Website: tailieumontoan.com LỜI GIẢI Đường thẳng AI cắt BC D đường tròn ngoại tiếp tam giác ABC E ⇒ ∠BIE = ( ∠BAC + ∠ABC ) , ∠CAE = ∠CBE ⇒ ∠IBE = ∠IBC + ∠CBE = ( ∠BAC + ∠ABC ) ⇒ tam giác EBI cân E ⇒ EB = EI ∠AIO = 90° ⇔ OI vng góc AE ⇔ IA = IE Mặt khác tam giác EAB tam giác EBD đồng dạng (g.g) ⇔ AB AE AE AI = = = 2, ∠ABI = ∠IBD ⇔ = BD BE IE ID G trọng tâm tam giác ABC, AG cắt BC M AG AI AG = 2⇔ = ⇔ IG song song BC GM ID GM Bài 25 Cho tam giác vuông ABC ( ∠BAC = 90°, AB < AC ) , H chân đường cao kẻ từ A Gọi I J tâm đường tròn nội tiếp tam giác ABH ACH, O tâm đường tròn ngoại tiếp tam giác AIJ Đường tròn cắt cạnh AB, AC D E Đường thẳng DE cắt cạnh BC kéo dài K Chứng minh I tâm đường tròn nội tiếp tam giác OKH J tâm đường tròn bàng tiếp tam giác OKH LỜI GIẢI ∠BAC= 90° ⇒ DE đường kính đường tròn ngoại tiếp tam giác AIJ ⇒ O giao điểm AH DE I, J tâm đường tròn nội tiếp tam giác ABH tam giác AHC ⇒ ∠IAJ= 45° ⇒ ∠IOJ= 90° AH vng góc BC ⇒ ∠IHJ= 90° ⇒ tứ giác OIHJ nội tiếp ⇒ ∠OHJ = ∠OIJ = 45° = ∠AHJ, ∠DOI = 2∠DAI = 2∠IAH = ∠IOH Liên hệ tài liệu word toán SĐT (zalo): 039.373.2038 18 Website: tailieumontoan.com ⇒ I tâm đường tròn nội tiếp tam giác OKH Tương tự ∠EOJ = ∠JOH ∠OHJ = ∠JHC = 45° ⇒ J tâm đường trịn bàng tiếp góc K tam giác OHK Bài 26 Cho tam giác ABC có AB = 2AC phân giác AD Gọi r,r1 ,r2 bán kính đường trịn nội tiếp tam giác ABC, ACD, ABD Chứng minh AD= pr + − p (p nửa chu vi tam giác r1 r2 ABC) LỜI GIẢI Sử dụng cơng thức diện tích tam giác: = = SABC pr,S ADC r1 ( AC + CD + AD ) r ( AB + BD + AD ) = ,SABD 2 Gọi H, K hình chiếu D lên hai cạnh AC,AB ⇒ DH = DK AB = 2AC ⇒ SABD = 2SADC ⇒ SABC = 3SADC pr AC + CD + AD pr ⇒ SACD = ⇒ = , (1) 3r1 AB + BD + AD 2pr (2) = SABD = SABC ⇒ 3r2 Cộng (1) (2) ta được: AC + CD + AD + AB + BD + AD pr pr = + ⇒ AD= + − p r1 r2 r1 r2 Bài 27 Chứng minh tam giác, đường tròn ngoại tiếp qua trung điểm đoạn thẳng nối tâm đường tròn nội tiếp tâm đường tròn bàng tiếp tam giác LỜI GIẢI Gọi I, J tâm đường trịn nội tiếp tâm đường trịn bàng tiếp góc ∠ACB Khi I, J nằm đường phân giác góc ∠ACB Liên hệ tài liệu word toán SĐT (zalo): 039.373.2038 19 Website: tailieumontoan.com Gọi E giao điểm CJ với đường tròn ngoại tiếp tam giác ABC Theo tính chất đường phân giác cắt đường tròn ngoại tiếp, ta suy EA = EI Gọi J tâm đường trịn bàng tiếp góc ∠ACB , theo tính chất J giao điểm đường phân giác góc ∠ACB phân giác ngồi góc ∠BAC ⇒ AI vng góc AJ ⇒ tam giác AIJ vuông A Mà EA = EI ⇒ ∠EAI = ∠EIA, ∠JAE + ∠IAE = 90° = ∠AJE + ∠AIJ ⇒ ∠JAE = ∠IJA ⇒ EA = EJ ⇒ EJ = EI Bài 28 Cho tam giác ABC nội tiếp đường trịn (O) Đường phân giác góc ∠ABC ∠ACB cắt đường tròn (O) D, E Đường tròn tâm D tiếp xúc cạnh AC, đường tròn tâm E tiếp xúc cạnh AB Chứng minh tâm đường tròn nội tiếp tam giác ABC nằm tiếp tuyến chung đường hai trịn (D) (E) (Vơ địch Nga 2002) LỜI GIẢI Gọi I tâm đường tròn nội tiếp tam giác ABC Từ I kẻ tiếp tuyến IH với đường tròn (E), tiếp tuyến IK với đường tròn (D) (H K tiếp điểm E, B khác phía IH; D, C khác phía IK) Theo giả thiết B, I, D thẳng hàng C, I, E thẳng hàng Gọi G tiếp điểm đường tròn (E) với cạnh AC ⇒ ∠BIE = ∠IBC + ∠ICB = ∠IBA + ∠ICA = ∠DBA + ∠ABE = ∠DBE ⇒ tam giác EBI cân E ⇒ EI = EB = EA Hai tam giác EBG tam giác EHI hai tam giác vng có EH = EG EB = EI ⇒ hai tam giác nên ⇒ ∠EIH = ∠EBG = ∠ECA = ∠ECB ⇒ IH song song BC Tương tự, IK song song với BC ⇒ H,I,K thẳng hàng Bài 29 Cho tứ giác ABCD nội tiếp đường tròn Gọi M, N, P, Q thứ tự tâm đường tròn nội tiếp tam giác ABD, ABC, BCD, CDA Chứng minh MNPQ hình chữ nhật LỜI GIẢI Theo tính chất tâm đường tròn nội tiếp tam giác 1 ⇒ ∠AMB= 90° + ∠ADB= 90° + ∠ACB= ∠ANB 2 Liên hệ tài liệu word toán SĐT (zalo): 039.373.2038 20 Website: tailieumontoan.com Từ suy tứ giác AMNB nội tiếp Tương tự, ta có tứ giác BNPC, CPQD, DQMA nội tiếp ⇒ ∠QMN = 360° − ( ∠QMD + ∠DMA + ∠AMB + ∠BMN ) = 360° − ( ∠QAD + ∠DMA + ∠AMB + ∠BAN ) 1 = 360° − ∠DAC + 90° 2 1 + ∠DBA + 90° + ∠ADB + ∠BAC 2 = 180° − ( ∠DAC + ∠DBA + ∠ADB + ∠BAC ) = 180° − 90°= 90° ⇒ ∠QMN= 90° Tương tự, ∠MNP = ∠NPQ = 90° ⇒ tứ giác MNPQ hình chữ nhật Bài 30 Cho tam giác ABC nội tiếp đường tròn (O) ngoại tiếp đường tròn (I) AI, BI, CI cắt đường tròn (O) D, E, F DE cắt CF M, DF cắt BE N, AD cắt EF P Gọi J tâm đường tròn ngoại tiếp tam giác DMN Chứng minh M, N, J, P nằm đường trịn LỜI GIẢI Theo tính chất tâm đường trịn nội tiếp tam giác ⇒ ∠NIM + ∠MDN = ∠BIC + ∠FDE = 90° + ∠BAC + ∠FDA + ∠ADE = 90° + ∠BAC+∠FCA + ∠ABC = 90° + ( ∠BAC + ∠ABC + ∠BCA )= 180° ⇒ tứ giác INDM nội tiếp ⇒ ∠INM = ∠IDM = ∠EBC ⇒ MN song song BC Tương tự PN song song AB, PM song song AC ⇒ ∠NPM = ∠BAC J tâm đường tròn ngoại tiếp ∆MDN ⇒ ∠NJM = 2∠NDM = ∠ABC + ∠ACB ⇒ ∠NJM + ∠NPM = ∠ABC + ∠ACB + ∠BAC = 180° ⇒ P,M,D,N nằm đường trịn Liên hệ tài liệu word tốn SĐT (zalo): 039.373.2038 21 Website: tailieumontoan.com Bài 31 Cho tam giác nhọn ABC có đường cao AD, BE, CF Gọi M, N tâm đường tròn nội tiếp tam giác BFD, CDE P, Q tâm đường tròn ngoại tiếp tam giác ABM, ACN Chứng minh MN song song PQ (IOM Shortlist 2012) LỜI GIẢI AD, BE, CF đường phân giác ⇒ ∠FBD = ∠DEC ∠BFD = ∠ECD ⇒ tam giác BFD đồng dạng ∆ECD Theo giả thiết M, N tâm đường tròn nội tiếp hai tam giác BFD, CDE ⇒ FD DM = CD DN 1 Mà ∠MDN = 180° − ∠MDB − ∠NDC = 180° − ∠FDB − ∠EDC 2 1 = 180° − ∠BAC − ∠BAC = 180° − ∠BAC = 180° − ∠FDB 2 ⇒ tam giác MDN tam giác FDC đồng dạng (c.g.c) ⇒ ∠DMN = ∠DFC = ∠DAC ⇒ ∠BMN = ∠BMD + ∠DMN = 90° + ∠BFD + ∠DFC 1 = 90° + ∠ACB + 90° − ∠ACB = 180° − ∠ACB = 180° − ∠NCB 2 ⇒ tứ giác BMNC nội tiếp Gọi K tâm đường tròn nội tiếp tam giác AEF Tương tự trên, ta chứng minh tứ giác BMKA, AKNC tứ giác nội tiếp ⇒ AK dây chung hai đường tròn (BMKA) (AKNC) Theo giả thiết P, Q tâm đường tròn ngoại tiếp tam giác ABM, ACN ⇒ P, Q tâm hai đường trịn (BMKA) (AKNC) ⇒ AK vng góc PQ Mặt khác M, N, K tâm đường tròn nội tiếp tam giác BDF, CDE, AEF ⇒ M, N, K nằm BI, CI, AI ⇒ ∠IKN = ∠NCA = 1 ∠NCB = ∠ACB ∠ACB ∠IMN = 2 ⇒ ∠IKN = ∠IMN Tương tự ∠IMK = ∠INK, ∠IKM = ∠INM ⇒ I trực tâm ∆KMN ⇒ KI vng góc MN ⇒ PQ song song MN Liên hệ tài liệu word toán SĐT (zalo): 039.373.2038 22 Website: tailieumontoan.com Bài 32 Cho hai đường tròn ( O1 ,R1 ) ( O ,R ) tiếp xúc ngồi A Góc vng ∠xAy quay quanh A, cắt đường tròn ( O1 ,R1 ) , ( O ,R ) B C, H hình chiếu A BC Xác định vị trí B, C để AH có độ dài lớn LỜI GIẢI Kẻ đường kính BD ⇒ ∠BAD= 90° Giả thiết ∠BAC = 90° ⇒ D,A,C thẳng hàng ⇒ ∠O1AD = ∠O AC, ∠O1AD = ∠O1DA, ∠O AC = ∠O CA ⇒ ∠O1DA = ∠O CA ⇒ BD song song với CO Từ A kẻ AE song song BD ⇒ AE ⇒= AE CA O A = = BD CD O O1 BD.O A 2R1.R = O O1 R1 + R AH vng góc với BC ⇒ AH ≤ AE ⇒ AE = 2R1.R ≥ AH R1 + R ⇒ AH lớn AH = 2R1.R R1 + R Đẳng thức xảy H ≡ E ⇒ O1B vng góc với BC O C vng góc với BC ⇒ BC tiếp tuyến chung hai đường tròn ( O1 ,R1 ) ( O ,R ) 60° ( AB > AC ) Gọi I tâm đường tròn nội tiếp H trực Bài 33 Cho tam giác nhọn ABC có ∠BAC = tâm tam giác ABC Chứng minh 2∠AHI = 3∠ABC Liên hệ tài liệu word toán SĐT (zalo): 039.373.2038 (APMO 2017) 23 Website: tailieumontoan.com LỜI GIẢI Ta có ∠BIC= 90° + ∠BAC= 120° H trực tâm, có ∠BAC = 60° ⇒ ∠BHC = 180° − 60= ° 120° ⇒ tứ giác BIHC nội tiếp ∠IHA = 180° − ∠IHD = 180° − ∠IHB − ∠BHD = 180° − ∠ICB − ∠AHE = 180° − ∠ACB − ∠ACB 3 = 180° − ∠ACB = 180° − (120° − ∠ABC= ) ∠ABC 2 Bài 34 Cho đường tròn ( O ) đường kính AB , dây CD song song với AB Từ C kẻ cát tuyến song song với AD cắt đường tròn ( O ) E Đường thẳng BE cắt CD F Qua F kẻ đường thẳng song song AD cắt AB M Chứng minh MC tiếp tuyến đường tròn ( O ) (Dự tuyển IMO 1995) LỜI GIẢI * Trường hợp Xét tam giác MAC tam giác FDB : , ∠MAC = CD // AB ⇒ AC = BD ∠FDB MF // AD ⇒ tứ giác MADF hình bình hành ⇒ MA = FD ⇒ hai tam giác MAC FDB (c.g.c) ⇒ ∠MCA = ∠FBD (1) AC = ED CE song song AD ⇒ ) ⇒ ∠CDA = ∠EAD, ∠EAD = ∠EBD (chắn cung ED Từ (1) suy ∠MCA = ∠CDA ⇒ MC tiếp tuyến đường tròn ( O ) * Trường hợp Vị trí CD thay đổi dẫn tới hình vẽ khác so với trường hợp 1, yêu cầu làm phải xét hai trường hợp cách chứng minh hoàn toàn tương tự (bạn đọc tự chứng minh) Bài 35 Cho tam giác ABC đường tròn nội tiếp tâm I Gọi D, E , F tiếp điểm đường tròn bàng Liên hệ tài liệu word toán SĐT (zalo): 039.373.2038 24 Website: tailieumontoan.com tiếp góc A tam giác ABC với cạnh tam giác Chứng minh S IBC > S DEF LỜI GIẢI Theo tính chất tiếp điểm, ta có: = AD AF = , BD BE ⇒ ∠EDF = ∠ADF − ∠BDE 1 = 90° − ∠BAC − 90° − ∠EBD 2 1 ( ∠CBD − ∠A) = (180° − ∠B − ∠A) = ∠IBC 2 = Tương tự, ∠EFD = ∠ICB ⇒ tam giác IBC tam giác EFD đồng dạng (g.g) S IBC BC ⇒ = S EFD FD Mặt khác, BC = 1 ( BD + BE + EC + CF ) > ( DE + EF ) > DF 2 BC 1 > ⇒ S IBC > S DEF DF 4 ⇒ Bài 36 Cho tam giác ABC , I tâm đường tròn nội tiếp Các đường thẳng AI , BI , CI cắt đường tròn ngoại tiếp tam giác ABC thứ tự D, E , F Chứng minh AD + BE + CF > AB + BC + CA LỜI GIẢI Theo tính chất đường phân giác cắt đường tròn ngoại tiếp = FB = FI ⇒ DB = DC = DI , EA = EC = EI , FA Suy AD + BE + CF = ( AI + ID ) + ( BI + IE ) + ( CI + IF ) = ( IA + IB + IC ) + ( ID + IE + IF ) = ( IB + IC + IC + IA + IA + IC ) + > ( DC + DC + EC + EB + FA + FC ) 1 ( BC + CA + AB ) + ( BC + CA + AB ) = AB + BC + CA 2 Bài 37 Cho tam giác ABC , đường phân giác góc ∠A cắt đường trịn ngoại tiếp tam giác ABC D Chứng minh AD > ( AB + AC ) (IMO XIX) Liên hệ tài liệu word toán SĐT (zalo): 039.373.2038 25 Website: tailieumontoan.com LỜI GIẢI Từ C kẻ đường thẳng CE song song với AD = AE ⇒ ∠DAC = ∠ACE DE DC ⇒ AC = AD phân giác góc ∠BAC ⇒ ∠BAD = ∠DAC ⇒ ∠BAD = ∠ADE ⇒ AB song song DE ⇒ AD = BE Gọi giao điểm AD BE I ⇒ IA + IB > AB, ID + IE > DE Cộng hai bất đẳng thức ta IA + IB + ID + IE > AB + DE ⇒ AD + BE > AB + DE ⇒ AD > AB + AC ⇒ AD > ( AB + AC ) Bài 38 Cho tam giác ABC với tâm đường tròn nội tiếp I , P điểm tam giác thỏa mãn ∠PBA + ∠PCA = ∠PBC + ∠PCB Chứng minh AP ≥ AI Đẳng thức xảy P ≡ I (IMO 2006) LỜI GIẢI Theo giả thiết ∠PBA + ∠PCA = ∠PBC + ∠PCB Mà ∠PBA + ∠PCA + ∠PBC + ∠PCB = ∠ABC + ∠ACB ⇒ ( ∠PBC + ∠PCB ) = ∠ABC + ∠ACB = 180° − ∠BAC ⇒ ∠PBC + ∠PCB= 90° − ∠BAC ⇒ ∠BPC= 180° − ∠PBC − ∠PCB= 90° + ∠BAC I tâm đường tròn nội tiếp tam giác ABC ⇒ ∠BIC = 90° + ∠BAC ⇒ ∠BPC = ∠BIC ⇒ B, P, I , C nằm cung trịn nhìn đoạn BC góc 90° + ∠BAC ⇒ DB = DI = DC (với D giao điểm AI với đường tròn ngoại tiếp tam giác ABC ) ⇒ D tâm đường tròn qua điểm B, P, I , C Tam giác APD có AP + PD ≥ AD = AI + ID ⇒ AP ≥ AI Dấu xảy P thuộc đoạn thẳng AD ⇔ P ≡ I Bài 39 Gọi I tâm đường tròn nội tiếp tam giác ABC Đường thẳng AI cắt đường tròn ngoại tiếp tam Liên hệ tài liệu word toán SĐT (zalo): 039.373.2038 26 Website: tailieumontoan.com , F cạnh BC thoả mãn ∠BAF = giác ABC D.E điểm cung nhỏ CD ∠CAE < ∠BAC Gọi G trung điểm IF Chứng minh giao điểm DG EI nằm đường tròn ngoại tiếp tam giác ABC LỜI GIẢI Gọi P giao điểm EI với đường tròn ngoại tiếp tam giác ABC Đường thẳng AI cắt BC J AF cắt đường tròn ngoại tiếp tam giác ABC K cắt DP Q Theo giả thiết ∠BAF = ∠CAE < < ∠BAC ⇒ BK = CE ⇒ BC song song với KE AD phân giác góc A ⇒ ∠KAD = ∠DAE = ∠DPE ⇒ tứ giác APQI nội tiếp ⇒ ∠AQI = ∠API = ∠AKE ⇒ QI song song KE ⇒ QI song song BC ⇒ QF JI = QA IA AI cắt BC J I tâm đường tròn nội tiếp tam giác ABC ⇒ JI CJ = IA CA Mặt khác ∠BCD = ∠BAD = ∠CAD ⇒ tam giác DCJ tam giác DAC đồng dạng (g.g) ⇒ CJ DC FQ JI CJ DC ID = Mà DI =DC ⇒ = = = = AC DA QA AI AC AD AD Theo định lí Menelaus với tam giác AIF cát tuyến QD, QD cắt FI G′ Ta có: QA GF DI =1 QF GI DA Kết hợp với (1) suy G′F = G′I ⇒ G ≡ G′ ⇒ GI = GF ⇒ DG DI cắt đường tròn ngoại tiếp tam giác ABC Bài 40 Cho tam giác ABC , D trung điểm BC Gọi I , J tâm đường tròn nội tiếp tam giác ABD ADC , P, Q tâm đường tròn bàng tiếp D tam giác ABD ADC Chứng minh bốn điểm I , J , P, Q nằm đường tròn LỜI GIẢI I tâm đường tròn nội tiếp tam giác ABD, P tâm đường trịn bàng tiếp góc ∠ADB ⇒ I P nằm đường phân giác góc ∠ADB ⇒ ∠IDA = ∠PDB BD phân giác ngồi góc ∠ABC Liên hệ tài liệu word toán SĐT (zalo): 039.373.2038 27 Website: tailieumontoan.com ⇒ ∠PBD= 90° + ∠ABD ⇒ ∠PBD= 90° + ( 90° − ∠IAD − ∠IDB ) = 180° − ∠IAD − ∠IDB = ∠AID ⇒ tam giác DIA đồng dạng với tam giác DBP (g.g) ⇒ DI DA = ⇒ DI DP = DA.DB DB DP Tương tự DJ DQ = DA.DC (1) (2) Mặt khác, DB = DC Kết hợp (1) (2) ta DI DP = DJ DQ ⇒ bốn điểm I , J , P, Q nằm đường tròn Bài 41 Cho tứ giác ABC có ∠BAC = 20°, ∠BCA = 35°, ∠BDC = 40°, ∠BDA = 70° Tính góc hai đường chéo LỜI GIẢI Dựng đường tròn ngoại tiếp tam giác ABC Đường thẳng BD kéo dài cắt đường tròn E ), Ta có ∠BEC = ∠BAC = 20° (chắn cung BC ∠BDC = ∠DEC + ∠DCE ⇒ 40° = 20° + ∠DCE ⇒ ∠DCE= 20° ⇒ DB= DC Tương tự, DE = DA ⇒ D tâm đường tròn ngoại tiếp tam giác ABC ⇒ ∠AEB = ∠ACB = 35° ⇒ ∠ADE = 180° − 70= ° 110° ⇒ ∠ACE= 55° ⇒ ∠ACD= 35° ⇒ ∠AID = ∠ACD + ∠BDC = 35° + 40° = 75° Bài 42 Cho tam giác ABC nội tiếp đường tròn ( O ) , A điểm cố định, B C thay đổi, đường cao BH CK Chứng minh HK song song với đường thẳng cố định LỜI GIẢI * Cách 1: Qua A dựng tiếp tuyến xy với đường tròn ⇒ ∠ACB = ∠BAx (chắn cung AB ) BH vng góc AC , CK vng góc AB ⇒ B, K , H , C nằm đường trịn đường kính BC ⇒ ∠AKH = ∠ACB Liên hệ tài liệu word toán SĐT (zalo): 039.373.2038 28 Website: tailieumontoan.com Mà ∠ACB = ∠xAB ⇒ ∠AKH = ∠KAx ⇒ KH song song Ax OA cố định, OA vuông góc Ax ⇒ Ax cố định ⇒ KH ln song song với đường thẳng cố định * Cách 2: Đường thẳng HK cắt đường tròn ngoại tiếp M , N ( ) AN + MB sđ ∠AKN = (1) CK vng góc AB BH vng góc AC ⇒ B, K , H , C nằm đường trịn đường kính BC ⇒ ∠BKH = ∠BKC + ∠CKH = 90° + ∠HBC ⇒ ∠AKN = 180° − ∠BKN = 180° − 90° − ∠HBC = 90° − ∠HBC = ∠ACB = ( ) + MA sđ BM Từ (1) suy AM = AN ⇒ OH vuông góc HK ⇒ KH ln song song với tiếp tuyến A đường tròn ( O ) Bài 43 Cho hai đường tròn ( O1 ) ( O2 ) cắt A B M điểm tùy ý ( O1 ) Qua M kẻ tiếp tuyến MD với đường tròn ( O2 ) ( D tiếp điểm) Chứng minh biểu thức MD không phụ thuộc MA.MB vào điểm M LỜI GIẢI Theo giả thiết MD tiếp tuyến với đường tròn ( O2 ) ⇒ ∠MDA = ∠DCA ⇒ hai tam giác MDA MCD đồng dạng (g.g) ⇒ MD MA = MC MD ⇒ MD = MA.MC Chia hai vế cho MC.MB ta MD MA.MC MC = = MA.MB MA.MB MB (1) Vì ∠AO1O2 = ∠AMB ∠AO2O1 = ∠MCB ⇒ tam giác MBC đồng dạng với tam giác O1 BO2 (g.g) ⇒ MC O1O2 = MB O1 B Liên hệ tài liệu word toán SĐT (zalo): 039.373.2038 29 Website: tailieumontoan.com Thay vào đẳng thức (1) ta OO MD = MA.MB O1 B Vế phải không đổi nên giá trị biểu thức MD khơng phụ thuộc vào vị trí M MA.MB Bài 44 Từ điểm M đường tròn ( O ) , kẻ hai tiếp tuyến MA, MB C điểm tùy ý cung nhỏ AB đường tròn ( O ) Qua C kẻ tiếp tuyến với đường tròn ( O ) cắt MA, MB E F Đường thẳng AB cắt OE , OF P Q Chứng minh tỉ số PQ không phụ thuộc vào vị trí C EF LỜI GIẢI Điểm C tùy ý cung nhỏ AB theo tính chất từ điểm kẻ tiếp tuyến với ( O ) ⇒ AB vng góc với MO Suy : ∠AEO = ∠CEO = 90° − ∠MEF ∠MAB = ∠MBA = 90° − ∠AMB Tam giác APE có ∠APE = 180° − ∠MAB − ∠AEO = 180° − 1 (180° − ∠EMF ) − (180° − ∠MEF ) 2 ⇒ ∠APE = 1 ( ∠EMF + ∠MEF ) = ∠EFB = ∠OFE 2 ⇒ ∠OPQ = ∠OFE ⇒ hai tam giác OPQ OFE đồng dạng (g.g) ⇒ PQ OH = FE OC M cố định ⇒ A, B cố định ⇒ OH khơng đổi, OC bán kính đường trịn ⇒ OH không đổi ⇒ tỉ số OC PQ không đổi PE Bài 45 Cho tam giác ABC nội tiếp đường trịn ( O ) có AB < AC Đường phân giác AD cắt đường tròn (O ) E Gọi M giao điểm AB CE Tiếp tuyến C đường tròn ( O ) cắt AD N Tiếp tuyến E cắt CN F Chứng minh BC , MN , EF song song với Từ suy Liên hệ tài liệu word toán SĐT (zalo): 039.373.2038 30 Website: tailieumontoan.com 1 = + CF CN CD LỜI GIẢI Theo giả thiết AD phân giác = , ∠CAE = ) ⇒ BE EC ∠BCE , ∠CEF = ∠CAE (chắn cung CE ⇒ ∠CEF = ∠BCE ⇒ BC song song EF EF FC tiếp tuyến ⇒ ∠ECF = ∠CAN = ∠NAM ⇒ M , A, C , N nằm cung tròn ⇒ ∠CMN = ∠CAN ⇒ ∠CMN = ∠CEF ⇒ EF song song MN Vậy BC , MN , EF song song với EF NF EF CF ⇒= ,= CD NC MN CN Cộng hai đẳng thức, ta được: NF + FC 1 + = = + EF 1⇒ = CN EF MN CD CD MN Mà EF , CF tiếp tuyến EF song song MN ⇒ EF = CF , MN = CN ⇒ 1 = + CF CN CD Bài 46 Cho ba điểm A, B, C nằm đường thẳng theo thứ tự Dựng tam giác vng cân ADB đỉnh A tam giác vuông cân BEC đỉnh E nằm phía AB Gọi I , J trung điểm AE DC Chứng minh BIJ tam giác vuông cân LỜI GIẢI Gọi H giao điểm đường tròn ngoại tiếp tam giác ADB với đường thẳng DC , DA vng góc AB ⇒ BD đường kính đường trịn ngoại tiếp tam giác ABD ⇒ ∠BHD= 90° ⇒ BH vng góc CH Giả thiết, ∠BEC= 90° ⇒ B, H , E , C nằm đường trịn đường kính BC ⇒ ∠BEH = ∠BCH , ∠BAH = ∠BDH ⇒ hai tam giác EAB CDB đồng dạng (g.g) Liên hệ tài liệu word toán SĐT (zalo): 039.373.2038 31 Website: tailieumontoan.com ⇒ AE BE = Mà BI , BJ hai trung tuyến tương ứng với hai tam giác DC BC ⇒ BI BE AE = = BJ BC DC ⇒ IE BE = ; ∠BJC = ∠BIE JC BC ⇒ tứ giác IJHB tứ giác nội tiếp ⇒ ∠BIJ = 90°, ∠IJB = ∠IHB = ∠BCE = 45° ⇒ BIJ tam giác vuông cân I Bài 47 Cho hình bình hành ABCD hai đường chéo cắt I Từ A kẻ đường vng góc với BC , CD, DB thứ tự H , E , K Chứng minh bốn điểm H , I , E , K nằm đường tròn LỜI GIẢI Theo giả thiết AH vng góc BC , AE vng góc CD ⇒ bốn điểm A, H , C , E nằm đường trịn đường kính AC I giao điểm BD AC ⇒ I trung điểm AC ⇒ I tâm đường tròn đường kính AC ⇒ ∠HIE = 2∠HAE = (180° − ∠BCD ) AH vng góc BC , AK vng góc BD, AE vng góc CD ⇒ tứ giác AKED, AKHB nội tiếp ⇒ ∠EKD = ∠EAD ∠BKH = ∠BAH ⇒ ∠HKE = 180° − ∠EKD − ∠BKH = 180° − ∠EAD − ∠BAH = 180° − ( 90° − ∠ADC ) − ( 90° − ∠ABC ) = 2∠ADC = (180° − ∠BCD ) ⇒ K I nhìn đoạn HE góc (180° − ∠BCD ) ⇒ K , I , E , H nằm đường trịn Bài 48 Cho hình bình hành ABCD đường phân giác góc ∠BAD cắt cạnh BC CD M N Gọi O tâm đường tròn ngoại tiếp tam giác CMN Chứng minh bốn điểm O, B, C , D nằm đường tròn Gọi E giao điểm hai đường tròn ngoại tiếp tam giác CMN tam giác BCD Từ suy 90° ∠AEC = LỜI GIẢI Liên hệ tài liệu word toán SĐT (zalo): 039.373.2038 32 Website: tailieumontoan.com O tâm đường tròn ngoại tiếp tam giác CMN , đường phân giác góc ∠BAD cắt BC M CD N ⇒ ∠BAM = ∠MAD, ∠BAM = ∠MAD (so le) ⇒ ∠BAM = ∠BMA ⇒ BA = BM = CD ⇒ CM =CN ⇒ MN vng góc OC ⇒ ∠OCD = ∠OCB + ∠BCD = 90° + ∠BAD ∠BMO = 180° − ∠OMC = 180° − ∠OCM = 90° + ∠BAD ⇒ tam giác BMO tam giác DCO (c.g.c) ⇒ ∠OBC = ∠ODC ⇒ bốn điểm O, C , D, B nằm đường tròn Từ chứng minh suy= OB OD = , OC OE ⇒ EB = CD ⇒ EC song song BD Mặt khác BA =∠ BE , ABD = ∠DBE ⇒ A, E đối xứng qua BD ⇒ AE vng góc BD ⇒ AE vng EC Bài 49 Cho tam giác ABC , đường cao AH Dựng đường trịn đường kính AB, D điểm đường trịn Đường thẳng DH cắt đường trịn đường kính AC E Gọi M , N trung điểm BC DE Chứng minh AMN tam giác vuông LỜI GIẢI Theo giả thiết AB đường kính đường trịn D điểm đường trịn ⇒ ∠ADB= 90° DH cắt đường trịn đường kính AC E ⇒ ∠AEC= 90° Ta có ∠ABH = ∠ADH , ∠ACH = ∠AEH ⇒ tam giác ABC tam giác ADE đồng dạng (g.g) ⇒ AB AD = BC DE AM , AN hai trung tuyến tương ứng ⇒ AB AD = ∠ABM = ∠ADN BM DN ⇒ tam giác ABM tam giác ADN đồng dạng (c.g.c) ⇒ ∠AMB = ∠AND ⇒ tứ giác ANHM tứ giác nội tiếp ⇒ ∠ANM = ∠AHM = 90° ⇒ tam giác AMN vuông N Bài 50 Cho tứ giác ABCD AB CD cắt E , AD, BC cắt F Chứng minh tứ giác Liên hệ tài liệu word toán SĐT (zalo): 039.373.2038 33 Website: tailieumontoan.com ABCD nội tiếp phân giác góc ∠AED ∠CFD vng góc với LỜI GIẢI Gọi I giao điểm hai đường phân giác góc ∠AED ∠CFD 1 ∠CFD + ∠CFE + ∠AED + ∠AEF 2 ⇒ ∠IFE + ∠IEF = = ( ∠CFD + ∠AED ) + ( ∠CFE + ∠AEF ) = ( ∠CFD + ∠AED ) + 180° − ∠ABC (180° − ∠C − ∠D + 180° − ∠A − ∠D ) + 180° − ∠B = = 360° − ∠B − ∠D − = ∠C + ∠A − ( ∠C + ∠A) 1 ( ∠C + ∠A) = ( ∠C + ∠A) 2 Tứ giác ABCD nội tiếp ⇔ ∠A + ∠C= 180° ⇔ ( ∠A + ∠C )= 90° ⇔ ∠IFE + ∠IEF= 90° ⇔ ∠FIE= 90° Chú ý: Bài tốn có tính chất đặc biệt Đường phân giác EI cắt BC P AD M , đường phân giác FI cắt cạnh AB N CD Q Chứng minh tứ giác MNPQ hình thoi Với chứng minh ta có lời giải khác: ∠FME = ∠D + ∠MED ∠BPM = ∠PBE + ∠BEP Tứ giác ABCD nội tiếp ⇔ ∠D = ∠CPE ME phân giác góc ∠AED ⇔ ∠MED = ∠MEA ⇔ ∠FMP = ∠FPM ⇔ FM = FP FQ phân giác góc ∠CFD ⇔ PQ vng góc với EM ⇔ IM =IN ⇔ NM = NP, QM =QP ⇔ MNPQ hình thoi Bài 51 Cho tam giác nhọn ABC nội tiếp đường tròn ( O ) Đường phân giác góc ∠BAC cắt cạnh BC D Từ D kẻ đường vng góc với AO cắt cạnh AB H Chứng minh DH = DC Liên hệ tài liệu word toán SĐT (zalo): 039.373.2038 34 Website: tailieumontoan.com LỜI GIẢI ∠OAB= 90° − ∠AOB Gọi giao điểm AO với HD I ⇒ ∠IAD = ∠BAD − ∠BAI = 1 ∠BAC − 90° − ∠AOB 2 ⇒ ∠ADI= 90° − ∠IAD= 180° − ⇒ ∠ADI = 180° − ( ∠BAC + ∠AOB ) 1 ∠ABC + ∠BAC ( ∠BAC + 2∠ACB ) = 2 Mặt khác ∠ADC góc ngồi tam giác ADB ⇒ ∠ADC = ∠ABC + ∠BAC ⇒ ∠ADI = ∠ADC ⇒ hai tam giác ADH tam giác ADC (g.c.g) ⇒ DH = DC Bài 52 Cho tam giác ABC , đường cao AH , phân giác CD ( H ∈ AB, D ∈ AB ) Gọi K hình chiếu D AC Đường thẳng DK cắt AH M Đường tròn qua D, M , C cắt cạnh BC E ( E khác B ) Chứng minh tam giác DEN tam giác cân LỜI GIẢI Bốn điểm D, E , C , M nằm đường tròn ⇒ ∠DME = ∠DCE = ∠ACB AK vng góc BC ⇒ ∠MAC = ∠HAC = 90° − ∠ACB DH vng góc AC ⇒ ∠AMK = ∠DMN = ∠ACB ⇒ ∠DME = ∠EMH = ∠ACB ⇒ ME phân giác góc ∠DMH D, E , C , M nằm đường tròn ) ⇒ ∠MDC = ∠MEC (cùng chắn cung MC DK vuông góc AC ⇒ ∠KDC + ∠DCK= 90° ⇒ ∠MDC + ∠ACB= 90° Liên hệ tài liệu word toán SĐT (zalo): 039.373.2038 35 Website: tailieumontoan.com ⇒ ∠MEC + ∠ECD= 90° ⇒ ME vng góc với CD ⇒ ME đường trung trực DN ⇒ ED = EN ⇒ tam giác DEN cân E Bài 53 Cho tam giác ABC , I , O tâm đường tròn nội, ngoại tiếp tam giác ABC đồng thời thỏa mãn AB + AC = BC Chứng minh tam giác AOI vuông LỜI GIẢI Gọi I tâm đường tròn nội tiếp tam giác ABC ⇒ AI AB AC = = ID BD CD AI AB + AC AB + AC ⇒= = = ID BD + DC BC ⇒= AB BD,= AC 2CD,= AD 3ID (1) AI cắt đường tròn ngoại tiếp tam giác ABC E ⇒ tam giác ABE tam giác BDE đồng dạng (g.g) ⇒ AB AE BE = = =2 BD BE DE ⇒ AE AE BE 2.2 =⇒ AE = DE ⇒ AD = 3DE (2) = = DE BE DE Từ (1) (2) suy ID = DE ⇒ AI = IE ⇒ OI vng góc AE ⇒ tam giác AIO vng I Bài 54 Cho tam giác ABC , gọi I , O tâm đường nội ngoại tiếp tam giác ABC , M trung điểm BC 1 Đường thẳng AM vng góc với OI Chứng minh = + BC AB AC LỜI GIẢI Gọi bán kính đường trịn nội ngoại tiếp tam giác ABC r , R Đặt BC = a, CA = b, AB = c= ,p a+b+c Theo giả thiết AM vng góc với OI Gọi giao điểm AM OI H ⇒ AI = IH + AH , OM = OH + HM , AO =+ OH AH , IM = IH + IM ⇒ AI + OM = AO + IM (1) Hạ ID vng góc AB ⇒ AD =p − a ⇒ tam giác ADI vuông ⇒ AI = AD + DI Liên hệ tài liệu word toán SĐT (zalo): 039.373.2038 36 Website: tailieumontoan.com ⇒ AI =( p − a ) + r 2 Theo giả thiết, MB = MC ⇒ OM = OB − BM = R2 − a2 2 a IM = r + − ( p − b ) = r + ( b − c ) 2 Thay vào (1) ta được: 1 1 2 ⇒ ( p − a ) + r + R2 − a2 = R2 + r + (b − c ) ⇒ = + 4 BC AB AC Bài 55 Cho tam giác ABC , đường phân giác AD Chứng minh rẳng 1 + = 2 BD CD AD tam giác ABC vuông A LỜI GIẢI AD kéo dài cắt đường tròn ngoại tiếp E Gọi I trung điểm BC ⇒ EI vng góc BC ⇒ ∠BAE = ∠BCE ⇒ tam giác ADB đồng dạng với tam giác CDE ⇒ AD CD = ⇔ AD.DE = BD.DC DB DE 1 CD + BD CD + BD = + = = AD BD CD BD CD AD DE ⇔ 2DE =BD + CD Mặt khác BD + CD = ( BI − DI ) + ( BI + ID ) = ( BI + DI ) 2 ⇔ DE =BI + DI Ta có DE = DI + EI ⇔ BI = EI ⇔ BI = EI ⇔ EI = BI = CI ⇔ tam giác BEC vng E ⇔ BC đường kính ⇔ AB vng góc AC Bài 56 Cho hai điểm A, B đường tròn tâm ( O ) Dựng tiếp tuyến B , lấy điểm C thỏa mãn AB = AC đường phân giác góc ∠ABC cắt đường thẳng AC đường tròn ( O ) Chứng minh ∠ABC > 72° Liên hệ tài liệu word toán SĐT (zalo): 039.373.2038 37 Website: tailieumontoan.com LỜI GIẢI Đường phân giác góc ∠ABC cắt AC D, D đường tròn ⇒ BD cắt đường trịn I ⇒ I điểm cung AB Theo giả thiết AB = AC ⇒ ∠ABC = ∠ACB ⇒ ∠ABC = 2∠IBA = 2∠IAB ∠CAB + 2∠ABC= 180° ⇒ ∠CAB + 4∠IAB= 180° (1) BC tiếp tuyến đường tròn ( O ) ⇒ ∠OBA + ∠ABC= 90° ⇒ ∠OBA + 2∠IAB= 90° (2) Từ (1) (2) suy ∠CAB = 2∠OBA (3) Mặt khác ∠IAB > ∠CAB nên từ (3) suy ∠IAB > 2∠OBA ⇒ ∠IAB > ∠OBA (4) Kết hợp (2) (4) ta được: ∠IAB + 2∠IAB > 90° ⇒ 5∠IAB > 180° ⇒ ∠IAB > 36° ⇒ ∠ABC > 72° Nhận xét: Điều kiện điểm D phải nằm đường tròn khơng thể thiếu Bài 57 Cho đường trịn tâm ( O ) dây AB đường kính C điểm cung nhỏ AB Hình chiếu C AB tiếp tuyến A, B với đường tròn ( O ) D, E , F Chứng minh CD = CE.CF LỜI GIẢI Theo giả thiết AE tiếp tuyến ( O ) AC ), CE ⊥ AE CD ⊥ AB ⇒ ∠EAC = ∠ABC (chắn cung ⇒ hai tam giác AEC BDC đồng dạng (g.g) ⇒ AC CE = BC CD Tương tự (1) CF BC = CD AC Từ (1) (2) ta có: (2) CD CE = ⇒ CD = CE.CF ⇒ CD = CF CD CE.CF Bài 58 Cho đường tròn ( O ) điểm P ngồi đường trịn Từ P kẻ tiếp tuyến PA, PB với đường tròn (O ) M điểm AB , qua M kẻ đường thẳng vng góc với OM cắt PA, PB C D Chứng minh M trung điểm CD Liên hệ tài liệu word toán SĐT (zalo): 039.373.2038 38 Website: tailieumontoan.com LỜI GIẢI Theo giả thiết OA vng góc AC , OB vng góc BD OM vng góc CD ⇒ OMAC OMDB tứ giác nội tiếp ⇒ ∠AOC = ∠AMC , ∠BMD = ∠BOD ∠AMC = ∠BMD (đối đỉnh) ⇒ ∠AOC = ∠BOD Tam giác OAC tam giác OBD hai tam giác vng có: OB ∠AOC = ∠BOD, OA = ⇒ hai tam giác ⇒ OC = OD ⇒ tam giác OCD tam giác cân O Mà OM vng góc CD ⇒ MC = MD Bài 59 Cho hai đường tròn ( O1 ) ( O2 ) cắt A B Đường thẳng O1 A cắt ( O2 ) C , đường thẳng O2 A cắt ( O1 ) D , đường thẳng qua B song song với AD cắt ( O1 ) E Chứng minh DE song song với AC O2C vng góc với CD (Kỳ thu Nữ sinh tài TQ 2014) LỜI GIẢI Tứ giác ABED nội tiếp ⇒ ∠O2 AB = ∠BED ⇒ ∠ACB = ∠AO2 B = ∠AO2O1 = 90° − ∠O2 AB= 90° − ∠BED ∠ABC= ∠AO2C= 90° − ∠O2 AC = 90° − ∠O1 AD Theo giả thiết, BE song song AD DE song song O1 A ⇒ ∠O1 AD = ∠BED ⇒ ∠ABC = 90° − ∠BED ⇒ ∠ABC = ∠ACB ⇒ tam giác ABC cân A ⇒ AB = AC Tứ giác O1 DCO2 có ∠CAO2 = ∠DAO1 (đối đỉnh) Mà tam giác ADO1 tam giác ACO2 cân ⇒ ∠AO1 D = ∠AO2C ⇒ ∠CO1 D = ∠DO2C ⇒ tứ giác O1 DCO2 nội tiếp Mặt khác ∠AO2O1= 90° − ∠BED ∠O1 DA = ∠O1 AD = ∠BED Liên hệ tài liệu word toán SĐT (zalo): 039.373.2038 39 Website: tailieumontoan.com ⇒ ∠DO2O1 + ∠O1 DO2= ( 90° − ∠BED ) + ∠BED= 90° ⇒ ∠DO1O2= 90° ⇒ ∠DCO2= 90° ⇒ O2C vng góc với CD Bài 60 Cho tam giác nhọn ABC ( AB > AC ) nội tiếp đường tròn ( O ) , M trung điểm BC Gọi P điểm nằm tam giác AMC thỏa mãn ∠BAM = ∠PAC Gọi O1 , O2 tâm đường tròn ngoại tiếp tam giác ABP ACP Chứng minh đường thẳng AO qua trung điểm O1O2 LỜI GIẢI Kéo dài AM , AP cắt đường tròn ( O ) E D Theo giả thiết ∠BAM = ∠PAC ∠BAE = ∠DAC = CE ⇒ ED // BC ⇒ BE Ta có ∠CAD =∠COD , ∠CAD = ∠CAP = ∠PO2C ⇒ ∠DOC = ∠PO2C ⇒ hai tam giác OCD O2 PC hai tam giác đồng dạng ⇒ DC OC OC = = CP O2 P O2C Từ ∠OCD = ∠O2CD suy ⇒ OCD + ∠OCP = ∠O2CP + ∠OCP ⇒ ∠DCP = ∠OCO2 ⇒ tam giác DCP OCO2 đồng dạng (c.g.c) ⇒ DC.OO DP DC = ⇒ DP = OO2 OC OC Tương tự, hai tam giác O1 BP OBD đồng dạng ⇒ hai tam giác PBD O1 BO đồng dạng ⇒ DB.OO1 DC.OO2 DB.OO1 DP DB = ⇒ DP = ⇒ = OO1 OB OB OC OB ⇒ DC OO1 EB OO1 = ⇒ = DB OO2 EC OO2 Từ OO1 vng góc AB OO2 vng góc AC ⇒ ∠BEA = ∠O1OA, ∠AEC = ∠AOO2 ⇒ ∠BEC = ∠O1OO2 ⇒ hai tam giác BEC O1OO2 đồng dạng (c.g.c) Liên hệ tài liệu word toán SĐT (zalo): 039.373.2038 40 Website: tailieumontoan.com Mà EM trung tuyến tam giác BEC ⇒ AO chứa trung tuyến tam giác O1OO2 ⇒ AO qua trung điểm O1O2 Bài 61 Cho tam giác nhọn ABC ( AB < AC ) , D E hai điểm cạnh BC thỏa mãn BD = CE ( D nằm B E ), ∠PAB = ∠EAC Chứng minh ∠PBA = ∠PCA LỜI GIẢI * Cách Dựng hình bình hành ABQP ⇒ ∠PAB = ∠PQB, ∠ABP = ∠QPB, AB = PQ Qua P, B, Q dựng đường tròn cắt PD F ∠PAB = ∠PQB = ∠BFP ⇒ ∠DFB = ∠EAC AE song song PD ⇒ ∠AEC = ∠EDB ⇒ ∠DBF = ∠ECA Theo giả thiết BD = CE ⇒ hai tam giác AEC FDB (g.c.g) ⇒ AC song song BF AC = BF ⇒ tứ giác ABFC hình bình hành ⇒ AB = CF , AB song song CF ⇒ tứ giác PQFC hình bình hành ⇒ PC song song QF ⇒ ∠ACP = ∠BFQ = ∠BPQ ⇒ ∠PBA = ∠PCA * Cách Dựng hình bình hành PBQC Gọi I giao điểm PQ BC ⇒ IB = IC IP = IQ Từ giả thiết BD = CE ⇒ ID = IE ⇒ PDQE hình bình hành ⇒ QE song song PD Cũng theo giả thiết PD song song AE ⇒ A, E , Q thẳng hàng Dựng hình bình hành ACQK ⇒ CQ song song AK , CQ = AK ⇒ ∠QAC = ∠AQK (1) ACQK PBQC hình bình hành ⇒ AK song song BP, AK = BP ⇒ AKBP hình bình hành ⇒ ∠PAB = ∠ABK (2) Từ (1) (2), kết hợp giả thiết ∠PAB = ∠EAC suy ra: Liên hệ tài liệu word toán SĐT (zalo): 039.373.2038 41 Website: tailieumontoan.com ∠AQK = ∠ABK ⇒ tứ giác AKBQ nội tiếp ⇒ ∠PBA = ∠BAK = ∠BQK = ∠PCA Bài 62 Cho tam giác ABC.D E cạnh AB AC cho B, C , D, E nằm đường tròn Gọi P giao điểm BE , CD H hình chiếu vng góc P AC M N trung điểm AP BC Chứng minh tam giác ACD đồng dạng với tam giác MNH (Iran 2015) LỜI GIẢI Gọi I J trung điểm PC AC ⇒ IJ song song AP IJ = AP Theo giả thiết, MA = MP ⇒ MI song song AC.PH vng góc AC ⇒ HM = AP ⇒ tứ giác MHJI hình thang cân MJ song song PC ⇒ MICJ hình bình hành ∠IMJ = ∠JCI = ∠ECD Tứ giác BDEC nội tiếp ) ⇒ ∠DBE = ∠DCE (chắn cung DE NJ song song AB, IN song song BE ⇒ ∠DBE = ∠JNI ⇒ năm điểm M , E , J , I , N nằm đường tròn ∠ACD = ∠AJM = ∠HNM ) ∠MHN = ∠MIN (chắn cung MN Mặt khác ∠DBE = ∠DCE ⇒ ∠ADC = ∠AEB.IN song song BE ⇒ ∠AEB = ∠MIN ⇒ ∠MIN = ∠AEB ⇒ ∠ADC = ∠MHN ⇒ tam giác ACD đồng dạng với tam giác MNH (g.g) Bài 63 Cho tam giác ABC ( AC > AB ) nội tiếp đường tròn O D điểm cạnh BC thỏa mãn ∠BAD = ∠CAO Đường thẳng AD cắt đường tròn ( O ) E Gọi I , J , M trung điểm BE , AC , OD Liên hệ tài liệu word toán SĐT (zalo): 039.373.2038 42 Website: tailieumontoan.com Chứng minh I , M , J thẳng hàng LỜI GIẢI AO cắt đường tròn ( O ) K ⇒ ∠AKC= 90° Xét tam giác AKC tam giác ABD có: ∠BAD = ∠CAK (giả thiết) ∠ABD = ∠ABC = ∠AKC (cùng chắn cung AC ) ⇒ ∠ADB = ∠ACK = 90° ⇒ AD vng góc BC ⇒ DI vng góc AC AJ = JC ⇒ OJ vng góc AC ⇒ OJ song song DI Tương tự OI song song DJ ⇒ tứ giác IDJO hình bình hành ⇒ IJ qua trung điểm DO ⇒ I , M , J thẳng hàng Bài 64 Cho tam giác ABC nội tiếp đường tròn Tiếp tuyến A,B,C cắt cạnh BC,CA,AB tương ứng P,Q,R Chứng minh P,Q,R thẳng hàng (Trục Lemoine) LỜI GIẢI Theo giả thiết PA tiếp tuyến đường tròn BAP BCA tam giác PAB đồng dạng với tam giác PCA(g.g) PB AB2 PA AB PB AB PC AC2 PC CA PA CA Tương tự: QC BC2 RA CA , QA BA RB CB2 Nhân ba đẳng thức ta được: PB QC RA AB2 BC2 CA PC QA RB AC2 BA CB2 Áp dụng định lí Menelaus P,Q,R thẳng hàng Bài 65 Cho tam giác cân ABC(AB AC) Gọi M trung điểm BC, P điểm cung nhỏ AM đường tròn ngoại tiếp tam giác ABM, N điểm thỏa mãn NMP 90 NP BP Chứng minh MNB CNM không phụ thuộc vào vị trí P Liên hệ tài liệu word toán SĐT (zalo): 039.373.2038 (Canada) 43 Website: tailieumontoan.com LỜI GIẢI Gọi I trung điểm BN NP BP tam giác PBN cân P PI vuông góc BN IN IB IM song song NC CNM NMI Cũng theo giả thiết NMP 90 tứ giác NIMP nằm đường tròn đường kính PN MNI MPI CNM NMI IPN BPI Từ suy ra: MNB CNM MPI BPI MPB MAB Vậy hiệu MNB CNM khơng phụ thuộc vào vị trí P Bài 66 Cho tam giác ABC B C 90 .D điểm cạnh BC cho AD AC Đường tròn nội tiếp tam giác ABC tiếp xúc cạnh BC,CA E F Gọi J tâm đường tròn nội tiếp tam giác ABD Chứng minh EF qua trung điểm CJ (IMO Shortlist 2006) LỜI GIẢI Gọi P giao điểm EF CJ Từ J kẻ đường thẳng song song với EF cắt cạnh BC M Đặt BCA 2α Theo giả thiết AC AD , E F tiếp điểm đường tròn nội tiếp tam giác ABC CE CF CEF CFE 90 α Vì DJ phân giác góc ADB BDJ 180 ADC 90 α Do JM / /EF DMJ CEF 90 α JDM JMD tam giác JDM cân J JD JM Gọi N tiếp điểm đường tròn (J) với cạnh BC JN MD MN ND Áp dụng cơng thức tính độ dài từ đỉnh tới điểm tiếp xúc cạnh tam giác: DM 2DN AD BD AB CM CD DM CD (AD BD AB) CD AD DB AB BC CA AB 2CE CJ 2CP PJ PC Liên hệ tài liệu word toán SĐT (zalo): 039.373.2038 44 Website: tailieumontoan.com Bài 67 Cho tam giác cân ABC(AB AC), D trung điểm cạnh AC, đường phân giác góc BAC cắt cung BDC E Đường thẳng BD cắt cung AEB F , đường thẳng AF BE cắt I CI cắt BD K Chứng minh I tâm đường tròn nội tiếp tam giác ABK (IMO Shortlist 2011) LỜI GIẢI Gọi G giao điểm cung BDC với cạnh AB M trung điểm BC A,E,M thẳng hàng ABC cân ED EG GBE EBD BI phân giác góc ABK DFA 180 BFA 180 BEA MEB 1 BEC BDC DFA BEM 2 Và FAD AFD DAF tam giác cân AD DF Áp dụng định lí Menelaus với ADF, cát tuyến IKC Mà AD DC, BI phân giác góc ABD Suy CA KD IF CD KF IA IF BF IA BA KD BF KD BF BF KF KF BA KF AD AD KD BD BF FD BF 1 AD AD AD KF KF KD DF AD 1 KD KD KD KD Tam giác BDA tam giác ADK đồng dạng (c.g.c) DAK DBA Áp dụng tính chất góc ngồi tam giác ABF IAB AFD ABD DAF DAK KAI AI phân giác BAK I tâm đường tròn nội tiếp tam giác ABK Bài 68 Cho tam giác ABC nội tiếp đường tròn (O), AD phân giác tam giác, M điểm thay đổi AD, P Q hình chiếu M AB AC,I trung điểm BC,H hình chiếu I PQ Chứng minh MH qua điểm cố định M thay đổi AD Liên hệ tài liệu word toán SĐT (zalo): 039.373.2038 45 Website: tailieumontoan.com LỜI GIẢI AD cắt đường tròn (O) E EC EB EI đường thẳng nằm đường kính đường trịn (O) cố định, đường kính cắt (O) F F điểm cố định MF cắt PQ J Từ giả thiết suy PAM QAM, MP vuông góc AB,MQ vng góc AC AM vng góc PQ Gọi K giao điểm AM PQ EF đường kính (O) ECF 90 Mà EFC EAC (chắn cung CE ) tam giác AQM tam giác FCE đồng dạng ( g.g) FI AK (1) IE KM Mặt khác EF đường kính (O) nên AE vng góc AF AF song song PQ Từ (1) (2) suy AK FJ (2) KM JM FI FJ Theo định lí Thales đảo ta IJ song song AD IE JM IJ vng góc PQ J H MH qua F điểm cố định Bài 69 Cho tam giác ABC nội tiếp đường tròn (O)(AB AC) Gọi I tâm đường tròn nội tiếp tam giác, M trung điểm cạnh BC,N điểm cung BAC Chứng minh IMB INA LỜI GIẢI Kéo dài AI cắt đường tròn (O) D D,M,N thẳng hàng hay DM qua tâm O NAD NBD 90 I tâm đường tròn nội tiếp ABC DB DI Mặt khác DNB tam giác vuông, BM ND BD DM.DN BD DN DI DN DM BD DM DI tam giác IDM tam giác NDI đồng dạng (c.g.c) IMD NID IMB IMD BMD IMD 90 Trong tam giác ANI : INA NID IAN IMD 90 Liên hệ tài liệu word toán SĐT (zalo): 039.373.2038 46 Website: tailieumontoan.com IMB INA Bài 70 Cho đường tròn (O) dây AB khơng đường kính C điểm AB,D điểm cung nhỏ AB đường tròn (O),OD cắt AB E Đường thẳng OC cắt đường tròn (O') ngoại tiếp tam giác OAB F, EF cắt đường tròn (K) G, GD cắt đường tròn (O') H Chứng minh CFD CDO OH qua trung điểm CD LỜI GIẢI Theo giả thiết O,A,F,B thuộc đường tròn ngoại tiếp tam giác OAB AFO ABO OAB tam giác OAC tam giác OFA đồng dạng (g.g) OA OC OC OD OF OA OD OF tam giác COD tam giác DOF đồng dạng (c.g.c) OFD ODC hay CFD CDO Kéo dài DO cắt đường tròn (O) K Bốn điểm A,B,D,K thuộc đường tròn (O) EA.EB ED.EK bốn điểm A,B,F,G thuộc đường tròn (O') EA.EB EF.EG EF.EG ED.EK tứ giác FKGD nội tiếp DKF DGF.(1) Mặt khác tứ giác FOGH nội tiếp đường tròn (O') FOH FGH.(2) Gọi I trung điểm DF OI song song với KF DOI DKF Kết hợp (1) (2) suy FOH IOD Mà tam giác OCD tam giác ODF đồng dạng, ID IF OH qua trung điểm CD Bài 71 Cho tứ giác ABCD nội tiếp đường tròn AC BD cắt P,AD BC cắt Q thỏa mãn PQ vng góc với AC E trung điểm AB Chứng minh PE vng góc với (Pakistan 2007) LỜI GIẢI Gọi K giao điểm đường tròn ngoại tiếp tam giác AQC đường tròn ngoại tiếp tam giác QBD ABCD nội tiếp đường tròn, giao điểm đường tròn (ABCD) đường tròn (AQC) A C , giao điểm (ABCD) đường tròn (QBD) B D Liên hệ tài liệu word toán SĐT (zalo): 039.373.2038 47 Website: tailieumontoan.com Q,P,K thẳng hàng DQK DBK ADB ACB QDB ACQ tam giác QAC tam giác QBD đồng dạng (g.g) QAC QBD Theo giả thiết ta có QP vng góc với AC, 90 AQP QAP PBK QBD KBQ 90 hay KB vng góc với BC Gọi H trực tâm tam giác QAC AH vng góc với QC,QP vng góc với AC H nằm BK Theo tính chất trực tâm tam giác PH PK Theo giả thiết, EA EB PE song song với KB PE vng góc với BC Bài 72 Cho tam giác nhọn ABC Tiếp tuyến B C với đường tròn ngoại tiếp tam giác ABC cắt P Gọi D điểm đối xứng B qua AC, E điểm đối xứng C qua AB,O tâm đường tròn ngoại tiếp tam giác PDE Chứng minh AO vng góc với BC LỜI GIẢI *Cách Theo giả thiết tiếp tuyến B C cắt P PB PC D điểm đối xứng B qua AC, E điểm đối xứng C qua AB CD CB BCD 2ACB BE BC CBE 2ABC PBC PCB BAC PCD PCB BCD BAC 2ACB, PBE 360 PBC CBE 360 BAC 2ABC ABC BCA CAB BAC 2ABC BAC 2ACB PBE PCD tam giác PBE tam giác PCD (c.g.c) PE PD tam giác PDE cân P PDE PED hai tam giác cân PBC,PED đồng dạng (g.g) PBC PED Gọi K giao điểm PB đường tròn ngoại tiếp tam giác PDE BKD PED PBC BC song song với KD BAC BKD Theo giả thiết CB CD AC vng góc với BD AB AD Liên hệ tài liệu word toán SĐT (zalo): 039.373.2038 48 Website: tailieumontoan.com BAC CAD BAD 2BAC 2BKD A tâm đường tròn ngoại tiếp tam giác DKB O tâm đường tròn ngoại tiếp tam giác PDK O A nằm trung trực DK AO vuông góc DK Mà DK song song BC AO vng góc với BC *Cách Goi J tâm đường trịn ngoại tiếp tam giác PBC Hồn tồn cách ta chứng minh tam giác PCD tam giác PBE CPD BPE, PD PE tam giác PDE cân P Mà PB tiếp tuyến với đường tròn ngoại tiếp tam giác ABC tam giác PBC tam giác cân BPC BPD DPC BPD BCP DPE tam giác PBC tam giác PDE đồng dạng ( g.g) BE = BC AB vng góc với CE AE AC EAC 2BAC Tam giác JBP cân J, BJP 2BAC tam giác AEC tam giác JBP đồng dạng (g.g) O tâm đường tròn ngoại tiếp tam giác PDE OP OE POE 2BAC tam giác OPE đồng dạng tam giác JBP ba tam giác AEC,OEP,JBP đồng dạng AEC OEP AEO CEP AE AC OE EP tam giác AEO tam giác CEP đồng dạng (c.g.c) AOE CPE Mà OEP JBP JPC nên AO song song JP Mà JP vng góc BC nên AO vng góc BC *Cách Trước hết ta chứng minh toán tổng quát: Cho tam giác ABC , P điểm ngồi đường trịn ngoại tiếp tam giác ABC kẻ cát tuyến PB PC cắt đường tròn ngoại tiếp tam giác ABC N M Gọi D E điểm đối xứng B C qua AM,AN K tâm đường tròn ngoại tiếp tam giác PDE Chứng minh AK vng góc với MN Liên hệ tài liệu word toán SĐT (zalo): 039.373.2038 49 Website: tailieumontoan.com Bài 73 Cho tam giác nhọn ABC nội tiếp đường tròn (O) Đường cao BD CE cắt H Kẻ MN vng góc với BH,N thuộc AC Chứng minh ON song song BC LỜI GIẢI Theo giả thiết, BMN 90 BD vng góc với AC BDC BDN 90 tứ giác BMDN nội tiếp Theo giả thiết, H trực tâm M trung điểm AH MA MH MD MBN MDA DAM DBC MBD MBN DBN DBC DBN NBC O tâm đường tròn ngoại tiếp tam giác ABC tam giác OBC cân O ABH ABD OBC ABM OBC MBD ABD MBD OBC NBC OBN BM BO cặp đẳng giác góc B tam giác ABN Mà có AH AO cặp đường đẳng giác xuất phát đỉnh A M O hai điểm đẳng giác tam giác ABN BNO ANM DNM Tứ giác BMDN nội tiếp DNM MBD ONB MBD NBC ON song song với BC Bài 74 Cho tam giác ABC Đường tròn qua B C cắt cạnh AB D AC E,CD BE cắt P Gọi M,N,G,F trung điểm AP,BC,AC,CP H hình chiếu P AC Chứng minh hai tam giác ACD MHN đồng dạng LỜI GIẢI Theo giả thiết M,G,F trung điểm PA,AC,CP PH vng góc với AC Theo tính đường tròn điểm (đường tròn Euler) với tam giác PAC suy tứ giác MHGF nội tiếp Đường tròn qua B C cắt cạnh AB D AC E tứ giác BDEC nội tiếp DCE DBE G,F,M trung điểm AC,PC,PA MG song song PC,MF song song với AC tứ giác MGCF hình bình hành FMG GCD ECD EBD Liên hệ tài liệu word toán SĐT (zalo): 039.373.2038 50 Website: tailieumontoan.com N trung điểm BC NF song song với BP,NG song song với BA FNG PBA EBD FNG FMG tứ giác MGFN nội tiếp điểm M,G,H,E,F,N thuộc đường tròn ACD AGM MNH MHN MFN AEB ACB CBE ADE EDC ADC tam giác ACD tam giác MHN đồng dạng (g.g) Bài 75 Cho tứ giác ABCD Đường tròn nội tiếp ABC tiếp xúc với AB,BC N P Đường tròn nội tiếp BCD tiếp xúc với BC,CD Q H Đường tròn nội tiếp CDA tiếp xúc với CD,DA K R Đường tròn nội tiếp DAB tiếp xúc với DA,AB S M Chứng minh tứ giác ABCD ngoại tiếp đường tròn MN HK PQ RS LỜI GIẢI Do đường tròn nội tiếp ABC tiếp xúc với AB,BC N P , đường tròn nội tiếp BCD tiếp xúc với BC,CD Q H , đường tròn nội tiếp CDA tiếp xúc với CD,DA K R , đường tròn nội tiếp DAB tiếp xúc với DA,AB S M AM AS, BN BP, CQ CH, DK DR Tứ giác ABCD ngoại tiếp đường tròn AB CD BC DA AM MN NB CH HK KD BP PQ QC DR RS SA MN HK PQ RS Bài 76 Cho A,B,C,D nằm đường thẳng d theo thứ tự cho đoạn BC nhỏ AB CD Dựng tam giác AEB, BPC, CQD phía với đường thẳng d Biết góc EPQ 120 Chứng minh 1 AB CD BC LỜI GIẢI Liên hệ tài liệu word toán SĐT (zalo): 039.373.2038 51 Website: tailieumontoan.com Kéo dài EP,QP cắt đường thẳng d M N Theo giả thiết, EPQ 120 MPN 120 (đối đỉnh) Tam giác BPC tam giác BPC 60 BPN CPM 60 PBC góc ngồi tam giác PBN BPN BNP PBC 60 PNB CPM Mặt khác EBA PCB 60 EB song song PC BEP CPM PNB PEB tứ giác ENBP nội tiếp Tương tự, tứ giác QMCP nội tiếp ENP PBC 60 PCQ PMQ EN song song QM ANE CMQ CPN AEN NEB NBE BEP AEB BEP PNB Mà AE BP nên hai tam giác NEA PNC (g.c.g) AN CP Hai tam giác PNC MQC đồng dạng (g.g) CM CQ CQ.CP CD.BC CM CP CN CN AB Mặt khác QPM tam giác CQ CP CM CD BC CM BC CD.BC AB Chia hai vế cho CD.BC ta 1 AB CD BC Bài 77 Cho hình thang ABCD nội tiếp đường trịn (O) đường kính AB,E giao điểm hai đường chéo AB CD Đường trịn tâm B bán kính BE cắt (O) P Q ( P phía với C bờ AB ) Đường thẳng qua E vng góc với BD cắt CD M Chứng minh PM vng góc với DQ Liên hệ tài liệu word tốn SĐT (zalo): 039.373.2038 (2014 Braztkos) 52 Website: tailieumontoan.com LỜI GIẢI Đường thẳng ME cắt AB N Theo giả thiết AB / /CD , đường tròn tâm B bán kính BE cắt đường trịn (O) P Q PQ AB PQ CD AB đường kính PA PB, PQ cắt AB I CD J PI AB Theo hệ thức tam giác vng, ta có: PB2 BI.BA BI.2OB.(1) ABCD hình thang cân EA EB EO AB Tam giác NEB vuông E BE BO.BN.(2) Mặt khác, BP BE Kết hợp với (1) (2) suy BI.2OB BO.BN BN 2BI Mà MN / /AD nên BNMC hình thang cân JM JC Tam giác DPQ nội tiếp đường tròn (O), DC PQ JM JC M trực tâm tam giác DPQ PM DQ Bài 78 Cho tam giác ABC , đường thẳng MN song song với cạnh BC (M cạnh AB, N cạnh AC), đường thẳng MC cắt BN P Đường tròn ngoại tiếp tam giác MPB giao với đường tròn ngoại tiếp tam giác NPC Q (P khác Q) Chứng minh góc BAQ góc CAP LỜI GIẢI Tứ giác MPQB nội tiếp BMQ BPQ AMQ 180 BMQ 180 BPQ Tứ giác NPQC nội tiếp BPQ NCQ AMQ 180 NCQ Tứ giác AMQC nội tiếp BAQ MCQ Tương tự, NBQ NAQ.(1) Theo giả thiết BMPQ nội tiếp CPQ ABQ Liên hệ tài liệu word toán SĐT (zalo): 039.373.2038 53 Website: tailieumontoan.com hai tam giác ABQ CPQ đồng dạng (g.g) MN / /BC AB BQ (2) CP PQ AM MN MP AM MP AB AM (3) AB BC PC AB PC PC MP Kết hợp (2) (3) suy BQ AM PQ MP Mà BQP AMP hai tam giác BQP AMP đồng dạng (c.g.c) PBQ MAP Kết hợp với (1) suy MAP NAQ BAQ CAP Bài 79 Cho tam giác ABC,D trung điểm BC,E cạnh BC thỏa mãn BAE CAD Đường tròn ngoại tiếp tam giác ABE cắt cạnh AC M , đường tròn ngoại tiếp tam giác ACE cắt cạnh AB N Chứng minh MN song song với BC LỜI GIẢI Trước hết chứng minh BE AB2 : CE AC2 SABE BE SABE AB.AE , SACD CD SACD AC.AD BE AB.AE (1) CD AC.AD Tương tự: BD AB.AD (2) CE AC.AE Nhân (1) với (2) ta được: BE AB2 CE AC2 BE CE c b a.b ac CE , BE CE b2 c2 b2 c2 b2 Theo giả thiết, AMEB nội tiếp CM.CA CE.CB CM CE.CB ab a a2 CA CA c2 b2 b2 b2 c2 BN BE.BC ac a a2 Tương tự, BN.BA BE.BC AB AB2 c b2 c2 b2 c2 MN song song BC Bài 80 Cho R S đường trịn (O),RS khơng phải đường kính T đường thẳng RS cho S trung điểm RT,J điểm cung nhỏ RS Qua R dựng tiếp tuyến với đường tròn (O) cắt đường tròn ngoại tiếp tam giác JST hai điểm, gọi A điểm gần R,AJ cắt đường tròn (O) K Chứng minh KT tiếp tuyến đường tròn ngoại tiếp tam giác JST Liên hệ tài liệu word toán SĐT (zalo): 039.373.2038 54 Website: tailieumontoan.com (IMO 2017) LỜI GIẢI Tứ giác RJSK nội tiếp đường tròn (O) JKR JSR Tứ giác AJST nội tiếp JSR JAT RKJ RKA JAT RK song song AT Gọi B giao điểm RK AS tam giác RSB tam giác TSA có RS ST, BRS ATS,RSB=TSA (đối đỉnh) hai tam giác (g.c.g) SB SA tứ giác RATB hình bình hành ART RTB RA tiếp tuyến đường tròn (O) ARS=RKS RTB=RKS tứ giác STBK nội tiếp KTS=KBS=SAT KT tiếp tuyến đường tròn ngoại tiếp tam giác JST Bài 81 Cho đường tròn () điểm O đường trịn Dựng đường trịn (O) cắt đường trịn () P Q C điểm đường tròn () , đường thẳng CP,CQ cắt đường tròn tâm (O) A B Chứng minh AB PQ (Sharygin Geometry Olympiad 2008) LỜI GIẢI Từ P kẻ đường thẳng vng góc với OP cắt đường trịn () D OD đường kính đường trịn () DP DQ tiếp tuyến với đường tròn (O) 1 QAP QOP, AQB AOB 2 BQP BAP (chắn cung BP đường tròn (O) ) DP tiếp tuyến đường tròn (O) AQP APx DPC.(1) DQ tiếp tuyến đường tròn (O) DQB QAB.(2) Mặt khác, D,Q,P,C nằm đường tròn () DPC DQC DQB.(3) Từ (1), (2), (3) QAB AQB QAP AQB QOP AOB Liên hệ tài liệu word toán SĐT (zalo): 039.373.2038 55 Website: tailieumontoan.com Tam giác OPQ tam giác OAB hai tam giác cân có cạnh bên góc đỉnh hai tam giác PQ AB Bài 82 Cho tam giác nhọn ABC , đường cao BD CE Dựng hai đường tròn qua A E đồng thời tiếp xúc với cạnh BC P Q Chứng minh A,D,P,Q nằm đường tròn LỜI GIẢI Gọi CH đường cao tam giác kẻ từ A BD,CE đường cao tam giác nhọn ABC tứ giác ACHE ADHB tứ giác nội tiếp Mặt khác, đường tròn qua A E đồng thời tiếp xúc với cạnh BC P Q Theo tính chất tiếp tuyến, ta có: BP BE.BA,BQ BE.BA BP BQ CP.CQ CB BPCB BQ CB BPCB BP CB2 BP CB2 BE.BA CB2 BC.BH BC BC BH BC.HC CD.CA Tứ giác ADQP nội tiếp, hay A,D,P,Q nằm đường tròn Bài 83 Cho tứ giác lồi ABCD Chứng minh đường tròn điểm tam giác ABC,BCD,CDA,DAB qua điểm LỜI GIẢI Trước hết nhắc lại đường tròn điểm tam giác ABC : H trực tâm tam giác ABC , đường tròn qua ba trung điểm cạnh BC , CA, AB, qua ba trung điểm AH , BH , CH chân ba đường cao kẻ từ đỉnh xuống cạnh đối diện gọi đường tròn Euler tam giác ABC Gọi M,N,P,Q thứ tự trung điểm AB,BC,CD,DA E,F trung điểm hai đường chéo AC BD Liên hệ tài liệu word toán SĐT (zalo): 039.373.2038 56 Website: tailieumontoan.com Q,E,P trung điểm AB,AC,DC QE song song cạnh CD PQ song song cạnh AC tứ giác QECP hình bình hành Tương tự, tứ giác MNCE FNCP hình bình hành Giả sử đường trịn Euler tam giác ABC tam giác ACD cắt E điểm thứ hai K tứ giác ENMK tứ giác QPEK nội tiếp Từ ta có: NKP NEK EKP EMN EQP ECN ECP NCP NFP tứ giác PFKN nội tiếp K thuộc đường tròn Euler tam giác BCD ba đường tròn Euler tam giác ABC,BCD,CDA qua điểm K đường tròn điểm tam giác ABC,BCD,CDA,DAB qua điểm Bài 84 Cho tam giác ABC nội tiếp đường tròn (O) (AC AB) Tiếp tuyến A cắt BC D Đường thẳng vng góc với BC B cắt trung trực AB E Đường thẳng vng góc với BC C cắt trung trực AC F Chứng minh D,E,F thẳng hàng LỜI GIẢI Theo giả thiết, AD tiếp tuyến đường tròn (O) ACD DAB tam giác ADB tam giác CDA đồng dạng (g.g) AD AB DB DA DB.DC CD CA DA AB2 AD DB.DC DB (1) CB2 CD CD DC Kẻ AH vng góc với BC Gọi M,N giao điểm OE với AB OF với AC MA MB, NA NC EBM BAH tam giác BME tam giác AHB đồng dạng BE AB AB.BM AB2 BE BM AH AH 2AH AC2 BE AB2 Tương tự CF 2AH CF AC2 Từ (1) suy DB BE hai tam giác DBE DCF đồng dạng (c.g.c) DC CF BDE CDF D,E,F thẳng hàng Liên hệ tài liệu word toán SĐT (zalo): 039.373.2038 57 Website: tailieumontoan.com Bài 85 Cho tam giác nhọn ABC BD,CE đường cao cắt H M trung điểm cạnh BC Đường tròn qua B,H,C cắt đường tròn qua D,E,M P Q ( P,B phía với AH ).Chứng minh DE,PH,MQ cắt điểm đường tròn ngoại tiếp tam giác ABC LỜI GIẢI Từ giả thiết suy tứ giác AEHD BPHC nội tiếp Gọi N giao điểm ED MQ NDB 180 EDB 180 EDH 180 EAH 90 ABC, NPB 180 HCB 90 ABC NDB NPB tứ giác BPDN nội tiếp Mặt khác PCE PCH PBH PBD PND PNE tứ giác PCNE nội tiếp đường tròn AH cắt cạnh BC F theo tính chất đường trịn điểm Euler F thuộc đường tròn D,E,M tứ giác BEDC,BPDN,AEFC,PCNE nội tiếp BNC PNC PNB PEC PDB PEF+FEC-PDF+BDF =FEC+BDF=FAC+FAB =BAC N thuộc đường tròn ngoại tiếp tam giác ABC Bài 86 Cho tam giác nhọn ABC , nội tiếp đường tròn (O) , trực tâm H Trung trực AH cắt AC,AB P Q Chứng minh OA phân giác góc POQ (Sharygin 2017) LỜI GIẢI Kéo dài AO cắt đường tròn (O) D ACD 90 Kéo dài AH cắt BC E ABE ADC tam giác EBA tam giác CDA đồng dạng (g.g) EAB CAD hay HAB OAC PQ trung trực AH tam giác APH AQH tam giác cân tương ứng P,Q hai tam giác AQH AOC đồng dạng (g.g) Liên hệ tài liệu word toán SĐT (zalo): 039.373.2038 58 Website: tailieumontoan.com AQ AH AO AC Từ HAB OAC OAQ CAH hai tam giác OAQ CAH đồng dạng (c.g.c) AOQ ACH Mặt khác, H trực tâm tam giác ABC ACH 90 BAC AOQ 90 BAC Chứng minh tương tự, ta có: OAP HBA 90 BAC AOQ AOP OA phân giác góc POQ Bài 87 Cho tam giác ABC nội tiếp đường tròn (O),D điểm tùy ý BC Đường thẳng qua D cắt AB E AC kéo dài F Đường tròn qua B,D,E cắt đường tròn (O) M Đường thẳng MF,MD cắt đường tròn (O) N P Chứng minh PN AB LỜI GIẢI Đường tròn qua B,D,E đường tròn qua A,B,C cắt M Theo tính chất tứ giác tồn phần AEDC , điểm M điểm Miquel đường tròn qua C,D,F qua M tứ giác CDMF nội tiếp FMC FDC Vì M,N,B,C thuộc đường tròn (O) FMC NBC FDC NBC BN song song FD Tứ giác CDMF nội tiếp DMC DFC Mặt khác DMC PMC PAC PAC DFC AP song song với AD AP song song với BN tứ giác BNAP hình thang cân hai đường chéo PN AB Bài 88 Cho đường trịn (O) điểm M nằm ngồi (O) Từ M kẻ tiếp tuyến MA,MB cát tuyến MCD (MC < MD) với đường tròn (O) Chứng minh AC BC DA DB Liên hệ tài liệu word toán SĐT (zalo): 039.373.2038 59 Website: tailieumontoan.com LỜI GIẢI Theo giả thiết MA,MB tiếp tuyến với đường tròn (O) ADC MAC tam giác MAC tam giác MDA đồng dạng (g.g) MA AC MD DA Tương tự MB BC MD DB Mà MA MB AC BC DA DB Chú ý: Tứ giác ADBC nội tiếp thỏa mãn AC BC có tên tứ giác điều hịa DA DB Bài 89 Cho đường tròn (O) dây AD Gọi I điểm đối xứng A qua D Kẻ tiếp tuyến IB với đường tròn (O) Từ A dựng tiếp tuyến với (O) cắt IB C CD cắt đường tròn (O) E Chứng minh EB song song AD LỜI GIẢI CA,CB tiếp tuyến (O) tứ giác AEBD tứ giác điều hòa DA EA DB EB Mặt khác AD DI DI EA DB EB ADBE nội tiếp BDI BEA hai tam giác BDI,BEA đồng dạng (c.g.c) CBE EAB BID BE song song AD Bài 90 Cho tam giác ABC M điểm tam giác AM,BM,CM cắt BC,CA,AB D,E,F N điểm BC cho AM vuông góc với MN Gọi P Q đối xứng M qua DE DF Chứng minh P,Q,N thẳng hàng Liên hệ tài liệu word toán SĐT (zalo): 039.373.2038 60 Website: tailieumontoan.com LỜI GIẢI Theo giả thiết, P Q đối xứng với M qua DE DF DP DM DQ Q,M,P nằm đường trịn tâm D bán kính DM Mặt khác AM vng góc MN NM tiếp tuyến đường tròn (D) Gọi K điểm đối xứng M qua BC K thuộc đường tròn (D) NK tiếp tuyến (D) MQKP tứ giác điều hòa N,P,Q thẳng hàng Bài 91 Cho tứ giác điều hòa ABCD Chứng minh đường phân giác góc BAD, BCD đường thẳng BD đồng quy LỜI GIẢI Giả sử đường phân giác góc BAD cắt BD E Theo tính chất đường phân giác ta có: EB AB ED AD ABCD tứ giác điều hòa AB CB CB BE AD CD CD ED CE phân giác góc BCD (đpcm) Bài 92 Cho tứ giác ABCD nội tiếp đường tròn (O) AB CD cắt P,AD cắt BC Q,AC cắt BD M Chứng minh O trực tâm tam giác MPQ (Định lí Brocard) (Điểm Brocard công bố Henri Brocard, sĩ quan quân đội Pháp, năm 1825) LỜI GIẢI Từ P kẻ tiếp tuyến PE,PF với đường tròn (O) Gọi I,K giao điểm EF với AB,CD tứ giác BEAF tứ giác điều hòa PIAB PKDC Liên hệ tài liệu word toán SĐT (zalo): 039.373.2038 61 Website: tailieumontoan.com Theo giả thiết, BC,AD,EF đồng quy Q thuộc EF Mặt khác từ PIAB PKDC AC,BD,IK đồng quy M thuộc EF Mà PO vng góc EF PO vng góc MQ Chứng minh tương tự, OQ vng góc MP O trực tâm MPQ Bài 93 Cho tứ giác ABCD nội tiếp đường trịn Đường phân giác góc BAD, BCD gặp cạnh BD Đường thẳng qua C song song với AD cắt đường thẳng qua A trung điểm BD P Chứng minh tam giác PCD cân (Moldova 2014) LỜI GIẢI Theo giả thiết đường phân giác góc BAD, BCD cắt E thuộc BD Theo tính chất đường phân giác ta có: AB BE CB AD DE CD tứ giác ABCD tứ giác điều hòa tiếp tuyến A C cắt BD Gọi M trung điểm BD T giao điểm hai tiếp tuyến Từ suy AC đường đối trung tam giác ABD CDT CDB CAB MAD CP song song AD DAP APC, PCD ADC TC tiếp tuyến ACT ADC (cùng chắn cung AC ) ACT PCD DCT PCA tam giác DCT tam giác PCA đồng dạng (g.g) CT CD CA CP tam giác TAC tam giác DCP đồng dạng (c.g.c) Mà tam giác TAC cân T tam giác DCP cân D Bài 94 Cho tứ giác ABCD nội tiếp đường tròn (O) AB,CD cắt E AD,BC cắt F AC,BD cắt G Đường tròn ngoại tiếp hai tam giác ADE DCF cắt H Đường phân giác góc AHB cắt AB I, đường phân giác góc DHC cắt DC J Liên hệ tài liệu word toán SĐT (zalo): 039.373.2038 62 Website: tailieumontoan.com Chứng minh I,G,J thẳng hàng LỜI GIẢI Theo giả thiết H điểm Miquel tứ giác toàn phần ABCD EHCB nằm đường tròn ADH AEH 180 , BCH AEH 180 ADH BCH Mà DAH DEH CEH HBC tam giác ADH tam giác BCH đồng dạng (g.g) AH AD DH (1) BH BC CH Tam giác AGD tam giác BGC đồng dạng (g.g) AG AD (2) BG BC Giả thiết HI phân giác AHB,HJ phân giác DHC AH AI DH DJ , (3) BH BI CH CJ Từ (1), (2) (3) suy AG AI DJ BG BI CJ GI phân giác AGB,GJ phân giác góc DGC Mặt khác AGB DGC (đối đỉnh) I,G,J thẳng hàng Bài 95 Cho sáu điểm A, B, C, D, E, F nằm đường tròn Gọi K, M, N theo thứ tự giao điểm cặp đường thẳng (AB, DE), (BC, EF), (CD, FA) Chứng minh ba điểm K, M, N thẳng hàng (Định lí Pascal) LỜI GIẢI Liên hệ tài liệu word toán SĐT (zalo): 039.373.2038 63 Website: tailieumontoan.com Giả sử DE cắt BC P, cắt AF Q, BC cắt AF I Áp dụng định lí Menelaus cho tam giác IPQ với cát tuyến ABK ta được: KP AQ BI KP AI BP 1 1 KQ AI BP KQ AQ BI Tương tự, ta chứng minh được: NQ CP DQ 2 NI CI DP MI FI EQ 3 MP FQ EP Áp dụng hệ thức đường tròn: IA.IF IB.IC, PC.PB PD.PE QF.QA QE.QD.4 Từ (1), (2), (3) (4) Þ KP NQ MI AI BP CP DQ FI EQ KQ NI MP AQ BI CI DP FQ EP KP NQ MI KQ NI MP Theo định lí đảo Menelaus K,M,N thẳng hàng Chú ý: định lí Pascal khơng với điểm đường tròn mà cho đường conic Bài 96 Cho tam giác ABC, D AB E AC cho B, D, E, C nằm đường tròn, BE CD cắt P Gọi H hình chiếu P AC M, N thứ tự trung điểm AP BC Chứng minh hai tam giác MHN ADC đồng dạng LỜI GIẢI Gọi I,J thứ tự trung điểm AC PC IJ song song AP,NI song song AB,NJ song song BP JNC EBC, INC ABC INJ BNJ BNI B Liên hệ tài liệu word toán SĐT (zalo): 039.373.2038 64 Website: tailieumontoan.com 180JNC 180INC INC JNC ABC EBC ABE DCE Ta có MJ song song AC,IJ song song AP DCE PJM JMI INJ IMJ MIJN nội tiếp Mặt khác M,I,J nằm đường tròn Euler tam giác APC , theo giả thiết PH vng góc AC M,H,I,J,N nằm đường trịn Euler HMN NIC CAD, HNM HIM ACD tam giác MNH tam giác ADC đồng dạng (g.g) Bài 97 Cho tam giác ABC nội tiếp đường tròn (O) AC = 2AB Tiếp tuyến A, C cắt M Chứng minh MB qua điểm cung BAC LỜI GIẢI Gọi I giao điểm MB với đường tròn (O) Tiếp tuyến A, C cắt M MCI MBI hai tam giác MCI,MBC đồng dạng (g.g) MC CI MB BC Tương tự, MA AI MB BA Do MA MC CI AI BC BA CI.BA AI.BC Áp dụng định lí Ptolemy với tứ giác AICB ta IB.AC AI.BC AB.CI IB.AC 2AB.CI Theo giả thiết AC = 2AB IC IB I điểm cung BAC Bài 98 Cho tứ giác ABCD nội tiếp đường tròn (O), đồng thời thỏa mãn AB CD Gọi P giao điểm AD CB AD BC, Q giao điểm AB CD E, F, H, G trung điểm AB, BC, CD, DA Đường phân giác góc APB cắt EG S, đường phân giác góc AQD cắt FH T Chứng minh ST song song với BD Liên hệ tài liệu word toán SĐT (zalo): 039.373.2038 (Japan) 65 Website: tailieumontoan.com LỜI GIẢI Theo giả thiết E G trung điểm AB CD PE,PG trung tuyến tam giác PBA tam giác PDC Hai tam giác có CPD chung Mà tứ giác ABCD nội tiếp đường tròn (O) BAD PCD hai tam giác PBA PDC đồng dạng (g.g) cạnh tương ứng đường tương ứng tỉ lệ với CD PG AB PE Mặt khác PS phân giác góc APB CD PG GS AB PE SE Tương tự: BC QF FT AD QH TH Kết hợp giả thiết CD BC GS FT ta thu SE TH AB AD Theo định lí Thales đảo ta có EH song song với FG TS,GF,EH song song với Mà FG đường trung bình tam giác CBD FG song song với BD ST song song với BD Bài 99 Cho đường tròn tâm (O) hai dây AB, CD khơng cắt P điểm đường trịn, PC, PD cắt AB M N Chứng minh AM.BN khơng phụ thuộc vào vị trí điểm P MN (Định lí Haruki) LỜI GIẢI Đường thẳng AB cắt đường tròn ngoại tiếp tam giác PMD E Ta có BAD BCD CPD CBD Mặt khác, CPD MPD MED AED AED CBD hai tam giác AED CBD đồng dạng (g.g) AE AD AD.BC AE CB CD CD Vì vế phải khơng đổi nên AE không đổi BE không đổi Theo hệ thức đường tròn AN.NB PN.ND PN.ND MN.NE AN.NB MN.NE Liên hệ tài liệu word toán SĐT (zalo): 039.373.2038 66 Website: tailieumontoan.com AM MN NB MN NB BE AM.NB MN.BE BE AM.BN MN AM.BN khơng phụ thuộc vào vị trí P MN Bài tốn Con bướm Cho đường tròn dây PQ I trung điểm PQ qua I kẻ hai cát tuyến AB CD AD BC cắt PQ M N Chứng minh IM = IN LỜI GIẢI Áp dụng định lí Haruki ta có: PM.IQ PI.NQ MI IN Theo giả thiết IP IQ PM NQ PM MI NQ NI MI IN MI NI PI IQ IM IN MI IN Bài 100 Cho tam giác ABC đường tròn nội tiếp tâm I tiếp xúc cạnh BC D Đường tròn bàng tiếp tâm J tiếp xúc cạnh BC F, DI cắt đường tròn (I) E Gọi H hình chiếu A BC Chứng minh A, E, F thẳng hàng Từ suy IF qua trung điểm AH LỜI GIẢI Gọi N tiếp điểm (I) với cạnh AC, K tiếp điểm (J) với AC IN,JK vng góc với AC IN JK song song AI IN , IN IE, JF JK AJ JK AI IN IE AJ JK JF Theo định lí Thales đảo ta có A, E, F thẳng hàng H hình chiếu A BC AH vng góc với BC AH DE song song với Theo định lí Thales, AM EI 1 MH ID Liên hệ tài liệu word toán SĐT (zalo): 039.373.2038 67 Website: tailieumontoan.com AM AH hay IF qua trung điểm AH Bài 101 Cho hai đường tròn k1 , k2 tiếp xúc với đường tròn k theo thứ tự A B, đường tròn k1 , k2 cắt C D Chứng minh đường phân giác góc ∠CAD ∠CBD cắt CD LỜI GIẢI Theo giả thiết ta suy tam giác ACD nhận đường tròn k1 đường tròn ngoại tiếp, tam giác CBD nhận đường tròn k2 đường tròn ngoại tiếp Từ A B kẻ tiếp tuyến với đường tròn k cắt E Đường tròn k1 , k2 tiếp xúc với đường k A B ⇒ E nằm đường thẳng CD Theo tính chất tiếp tuyến ⇒ ∠EAC = ∠ADC Từ suy tam giác EAC tam giác EDA đồng dạng (g.g) ⇒ AC AE = AD DE Gọi AP phân giác ∠CAD (D thuộc CD) ⇒ Tương tự, AC CP AE (1) = = AD DP DE BE BC = DE BD Mà EA EB tiếp tuyến đường tròn k ⇒ EA = EB ⇒ AE BC = (2) DE BD CP AE BC Kết hợp (1) (2) ta được: = = DP DE BD ⇒ BP phân giác góc ∠CBD Bài 102 Cho tam giác ABC nội tiếp đường trịn (O) P điểm cung BC khơng chứa A D H hình chiếu P AD BC Gọi M N trung điểm AC, DH Chứng minh PN vng góc với NM LỜI GIẢI Gọi K hình chiếu P AC Theo tính chất đường thẳng Simson ⇒ D, H , K thẳng hàng Ta có tứ giác ADPK nội tiếp ⇒ ∠KDP = ∠KAP Tứ giác DBPH nội tiếp ⇒ ∠DPH = ∠DBH = ∠APC ⇒ tam giác DHP tam giác ACP đồng dạng (g.g) Theo giả thiết = MA MC = , ND NH ⇒ tam giác PDN tam giác PAM đồng dạng (c.g.c) ⇒ ∠DNP = ∠AMP ⇒ ∠PNK = ∠PMK Liên hệ tài liệu word toán SĐT (zalo): 039.373.2038 68 Website: tailieumontoan.com ⇒ tứ giác MNPK nội tiếp ⇒ ∠PNM = ∠PKM = 900 ⇒ PN vng góc với NM Bài 103 Cho tam giác ABC , D trung điểm cạnh BC Đường tròn qua D tiếp xúc với AB B, đường tròn qua D tiếp xúc AC C cắt M Gọi N điểm đối xứng M qua cạnh BC Chứng minh A, N , D thẳng hàng LỜI GIẢI Gọi (O1 ) đường tròn qua D tiếp xúc với AB B ⇒ O1 B ⊥ AB O1 nằm trung trực BD Tương tự xác định đường tròn O2 O1 O2 cắt M ⇒ ∠BMD = ∠ABC , ∠DMC = ∠ACB (do AB, AC tiếp tuyến) ⇒ ∠BMC = ∠BMD + ∠DMC = ∠ABC + ∠ACB = 1800 − ∠BAC ⇒ tứ giác ABMC nội tiếp đường tròn hay M thuộc đường tròn ngoại tiếp tam giác ABC ⇒ ∠AMC = ∠ABC = ∠ABD = ∠BMD, ∠MAC = ∠MBC = ∠MBD ⇒ hai tam giác AMC , BMD đồng dạng (g.g) ⇒ MC AC AC = = ⇒ hai tam giác ACD CMD đồng dạng (c.g.c) MD BD CD ⇒ ∠ADC = ∠MDC ⇒ ∠ADB = ∠MDB ⇒ DA, DM đối xứng qua cạnh BC ⇒ N thuộc AD ⇒ A, N , D thẳng hàng Bài 104 Cho tam giác ABC nội tiếp đường tròn (O) Tiếp tuyến B C với đường tròn (O) cắt P Gọi M trung điểm BC , D E hình chiếu P AD AC Chứng minh AM vng góc với DE LỜI GIẢI Theo giả thiết, tiếp tuyến B C cắt P ⇒ PD = PE , ∠BCP = ∠BAC Theo giả thiết, MB = MC ⇒ PM vng góc với BC Kết hợp với giả thiết PE ⊥ AC suy tứ giác PMCE tứ giác nội tiếp ⇒ ∠MCP = ∠MEP ⇒ ∠MEP = ∠BAC ⇒ ∠BAC + ∠AEM = ∠BAC + (900 − ∠MEP) = ∠BAC + (900 − ∠BAC ) = 900 ME cắt AB K ⇒ EK vng góc với AB Liên hệ tài liệu word toán SĐT (zalo): 039.373.2038 69 Website: tailieumontoan.com Tương tự DG vng góc với AE ⇒ M trực tâm tam giác ADE ⇒ AM vng góc với DE Bài 105 Cho tam giác ABC nội tiếp đường tròn (O) Tiếp tuyến B C với đường tròn (O) cắt P Đường trịn đường kính AO cắt đường tròn ngoại tiếp tam giác OBC Q ( Q khác O) Chứng minh A, P, Q thẳng hàng LỜI GIẢI Từ giả thiết suy đường tròn đường kính OA tiếp xúc với đường trịn (O) OQ vng góc với AQ ⇒ ∠AQO =900 Tiếp tuyến B C với đường tròn (O) cắt P ⇒ OP vng góc BP OC vng góc CP ⇒ ∠OBP = 900 , ∠OCP = 900 ⇒ tứ giác OBPC nội tiếp đường kính OP ⇒ OQ vng góc QP ⇒ ∠OQP =900 ⇒ ∠AQO + ∠OQP =1800 ⇒ A, Q, P thẳng hàng Bài 106 Cho tam giác ABC đường tròn nội tiếp tâm ( I ) tiếp xúc với cạnh BC , CA, AB D, E , F Chứng minh đường tròn ( AEF ) tiếp xúc với đường tròn ( BIC ) LỜI GIẢI * Cách Theo giả thiết ta có IE ⊥ AC IF ⊥ AB ⇒ I thuộc đường tròn ( AEF ) có đường kính AI Qua I kẻ đường thẳng vng góc với AI cắt AB, AC M N ⇒ MN tiếp tuyến đường tròn ( AEF ) Theo tính chất đường trịn nội tiếp ⇒ ∠AIB = 900 + ∠C ⇒ ∠MIB = ∠AIB − ∠AIM = ∠C Mặt khác, I tâm đường tròn nội tiếp tam giác ABC ⇒ ∠ICB = ∠C ⇒ ∠MIB = ∠ICB ⇒ MN tiếp tuyến đường tròn ( IBC ) ⇒ đường tròn ( AEF ) tiếp xúc với đường tròn ( BIC ) I * Cách Liên hệ tài liệu word toán SĐT (zalo): 039.373.2038 70 Website: tailieumontoan.com Các tứ giác AEIF , BDIF , CDIE tứ giác nội tiếp 1 ⇒ ∠BIF = ∠BDF = 900 − ∠ABC = (∠BAC + ∠ACB) 2 = ∠IAF + ∠ICD = ∠IEF + ∠ICD ⇒ đường tròn ( AEF ) tiếp xúc với đường tròn ( BIC ) I Bài 107 Cho tam giác ABC , P điểm tam giác Gọi D hình chiếu P cạnh BC Đường trịn tâm P, bán kính PD cắt cạnh AC , AB E F Đường tròn qua ( BDF ) giao với đường tròn (CDE ) I (I khác D) Chứng minh đường tròn ( AEF ) tiếp xúc với đường tròn ( BIC ) LỜI GIẢI Theo giả thiết đường trịn tâm P, bán kính PD ⇒ BC tiếp tuyến đường tròn ⇒ ∠FDB = ∠FED Từ I kẻ tiếp tuyến Ix với đường tròn ( BIC ) ⇒ ∠xIB = ∠ICB Theo tính chất tứ giác nội tiếp BDIF ⇒ ∠BIF = ∠BDF = ∠DEF = ∠IEF + ∠IED = ∠IEF + ∠ICB Mặt khác ∠BIF = ∠BIx + ∠xIF = ∠IEF = ∠xIF ⇒ Ix tiếp tuyến đường tròn ( IEF ) ⇒ đường tròn ( IEF ) đường trịn ( IBC ) tiếp xúc Ta có ∠EIF = 3600 − ∠DIF − ∠DIE = 3600 − (1800 − ∠B ) − (1800 − ∠C ) = ∠B + ∠C = 1800 − ∠A ⇒ tứ giác AFIE nội tiếp hay A thuộc đường tròn ( IEF ) ⇒ đường tròn ( AEF ) tiếp xúc với đường tròn ( BIC ) Bài 108 Cho tam giác ABC nội tiếp đường trịn (O) M điểm cung BC chứa A, P điểm AM, D hình chiếu P BC Đường trịn tâm P bán kính PD cắt AC, AB E F Chứng minh đường tròn ( AEF ) tiếp xúc đường trịn tâm M bán kính MB LỜI GIẢI Gọi I giao điểm đường tròn ( BFD) (CED) (I khác D) ⇒ ∠FIE = ∠FID + ∠DIE = ∠FBD + ∠DCE = 1800 − ∠BAC ⇒ tứ giác AFIE nội tiếp đường tròn Mặt khác M điểm cung BC chứa A ⇒ AM phân giác ngồi góc ∠BAC Liên hệ tài liệu word toán SĐT (zalo): 039.373.2038 71 Website: tailieumontoan.com Mà PE = PF ⇒ M điểm cung FAE đường tròn ngoại tiếp tam giác AEF ⇒ bốn điểm F , A, P, E nằm đường trịn Ta có PD = PE = PF 1 ⇒ ∠PDF = 900 − ∠DPF ∠PDE= 900 − ∠DPE 2 1 1800 − ∠FAE = ∠FDE = ∠FDP + ∠PDE = ∠1800 − ∠FPE = 2 ⇒ ∠BIC = ∠BIF + ∠FIE + ∠EIC = ∠FDB + 1800 − ∠FAE + ∠EDC = 1800 − ∠FAE + 1800 − ∠FDE 1 = 1800 − ∠FAE + ∠FAE = 1800 − ∠FAE 2 (do tứ giác BFDI CEDI nội tiếp) ⇒ I thuộc đường trịn tâm M bán kính MB Xét tiếp tuyến xIy với đường trịn tâm M bán kính MB, ta có: ∠xIE = ∠xIC + ∠CIE = ∠CBI + ∠EDC = ∠IFD + ∠CDE = ∠IFD + ∠EFD = ∠EFI ⇒ đường tròn ( AEF ) tiếp xúc đường trịn tâm M bán kính MB Bài 109 Cho tam giác ABC nội tiếp đường tròn (O) Một tiếp tuyến với đường tròn ngoại tiếp tam giác BOC cắt AB E AC F Gọi D điểm đối xứng A qua EF Chứng minh đường tròn ngoại tiếp tam giác DEF tiếp xúc với đường tròn (O) (Serbia 2016) LỜI GIẢI Gọi I tiếp điểm EF với đường tròn ( BOC ), P (P khác I ) giao điểm đường tròn ( BEI ) (CFI ) ⇒ ∠BPC = ∠BPI + ∠IPC = ∠AEI + ∠AFI = 1800 − ∠EAF (do tứ giác BEIP tứ giác CFIP nội tiếp) ⇒ tứ giác ABPC nội tiếp ⇒ điểm P thuộc đường tròn (O) ∠EPF = ∠EPI + ∠IPF = ∠EBI + ∠ICF = = ∠ABC − ∠IBC + ∠ACB − ∠ICB = (1800 − ∠BAC ) − (1800 − ∠BIC ) = ∠BIC − ∠BAC = ∠BOC − ∠BAC = ∠BAC = ∠EDF ⇒ tứ giác EFPD nội tiếp đường tròn Qua P kẻ tiếp tuyến Px với đường tròn (O) ⇒ ∠EPx = ∠BPx + ∠BPE = ∠BCP + ∠BIE = ∠BCP + ∠ICB Liên hệ tài liệu word toán SĐT (zalo): 039.373.2038 72 Website: tailieumontoan.com = ∠ICP = ∠IFP = ∠EFP ⇒ đường tròn ngoại tiếp tam giác DEF tiếp xúc với đường tròn (O) Bài 110 Cho tam giác ABC , P điểm tam giác Tiếp tuyến P đường tròn ( PBC ) cắt cạnh AB D cạnh AC E Đường tròn ( BDP) đường tròn (CEP) cắt Q (P khác Q) Chứng minh đường tròn ( DEQ) tiếp xúc với đường tròn cố định LỜI GIẢI Tứ giác BDPQ nội tiếp đường tròn ⇒ ∠ADE = ∠BQP Tứ giác CEPQ nội tiếp đường tròn ⇒ ∠AED = ∠PQC ⇒ ∠BQC = ∠BQP + ∠PQC = ∠ADE + ∠AED = 1800 − ∠DAE ⇒ tứ giác ABQC nội tiếp đường tròn hay Q thuộc đường tròn ngoại tiếp tam giác ABC Qua Q kẻ tiếp tuyến Qx với đường tròn ngoại tiếp tam giác ABC ⇒ ∠BQx = ∠BCQ Tứ giác BDPQ nội tiếp ⇒ ∠DQP = ∠DBP Tứ giác CEPQ nội tiếp ⇒ ∠PCQ = ∠PEQ Theo giả thiết DE tiếp tuyến P ⇒ ∠DPB = ∠PCB ⇒ ∠xQD = ∠xQB + ∠BQD = ∠BCQ + ∠PCB = ∠PCQ = ∠PEQ = ∠DEQ ⇒ Qx tiếp tuyến đường tròn ( DEQ) ⇒ đường tròn ( DEQ) tiếp xúc với đường tròn ngoại tiếp tam giác ABC ⇒ đường tròn ( DEQ) tiếp xúc với đường tròn cố định Bài 111 Cho tam giác nhọn ABC ( AB > AC ) Gọi Γ đường tròn ngoại tiếp tam giác, H trực tâm tam giác, F hình chiếu vng góc A BC , M trung điểm BC , Q K Γ thỏa mãn ∠HQA = 900 ∠HKQ = 900 ( A, B, C , K , Q Γ theo thứ tự đó) Chứng minh đường trịn ( HKQ) đường tròn ( MFK ) tiếp xúc (Ukraine) LỜI GIẢI Gọi (O) tâm đường tròn ngoại tiếp tam giác ABC Kéo dài AO cắt đường tròn Γ D, AH kéo dài cắt đường tròn Γ E ⇒ AD đường kính ⇒ ∠AQD =900 Mà ∠AQH =900 ⇒ D, H , Q thẳng hàng Liên hệ tài liệu word toán SĐT (zalo): 039.373.2038 73 Website: tailieumontoan.com Giả sử QD cắt BC G, tam giác ABC nhọn ⇒ H F đối xứng qua BC ⇒ FH = FE ⇒ HC =CE , ∠AED =900 ⇒ DE song song BC ⇒ BD = CE ⇒ tam giác GBD tam giác GCH ⇒ GB = GC ⇒ G ≡ M ⇒ Q, H , M , D thẳng hàng Kẻ đường kính QP ∠PKQ = 900 Theo giả thiết ∠HKQ = 900 ⇒ P, H , K thẳng hàng Xét đường tròn ngoại tiếp tam giác KHE : 900 − ∠QOE = 900 − ∠QBE ∠PKE = ∠PQE = ∠OQE = 1 + sñ BD ) = + sñ CE ) = ∠QMC = ∠EMC = (sñ QC (sñ QC sñ QE = ∠QBE 2 Mặt khác, ∠MHE = 900 − ∠QMC = 900 − ∠QBE ⇒ ∠PKE = ∠MHE ⇒ DQ tiếp tuyến đường tròn ngoại tiếp tam giác KHE H Đường vng góc với HQ cắt BC I ⇒ IK = IH = IE ⇒ I tâm đường tròn ngoại tiếp tam giác KHE DQ tiếp tuyến ⇒ IH vng góc QD ⇒ IH 2= IF IM= IK ⇒ IK tiếp tuyến đường tròn ngoại tiếp tam giác MF IH vng góc HQ ⇒ ∠HKQ = 900 ⇒ IH tiếp tuyến đường tròn ngoại tiếp tam giác HKQ ⇒ IK tiếp tuyến đường tròn ngoại tiếp tam giác HKQ ⇒ đường tròn HKQ đường tròn MFK tiếp xúc Nhận xét: Bài ban tổ chức xếp vào loại khó, qua kết làm thí sinh phổ điểm cho thấy điểm thấp Để giải này, ta tìm đường thẳng tiếp xúc với đường tròn để làm cầu nối chứng minh hai đường tròn tiếp xúc Bài 112 Cho tam giác ABC nội tiếp đường tròn tâm O Đường tròn (Ω) tâm A cắt cạnh BC D E ( B, D, E , C theo thứ tự đó), đồng thời đường trịn (Ω) cắt đường trịn O F G ( F , B, C , G theo thứ tự đó) Đường trịn qua F , B, D cắt AB K , đường tròn qua E , C , G cắt cạnh AC L, đường thẳng FK GL cắt X Chứng minh X nằm đường thẳng AO (Hy Lạp) LỜI GIẢI Đường tròn Ω cắt cạnh BC D, E đường tròn (O) F , G ⇒ F , D, E , G nằm đường tròn ⇒ tứ giác FDEG nội tiếp ⇒ AF = AG ∠GEC = ∠GFD (vì ∠GFD + ∠GED =1800 ) Liên hệ tài liệu word toán SĐT (zalo): 039.373.2038 74 Website: tailieumontoan.com ⇒ AF = AG Giả sử đường thẳng FG cắt AC I 1 ) = ) = (sñ (sñ sñ AC = AF + sñ GC AG + sñ GC ⇒ ∠AIF = ∠ABC (1) 2 Tứ giác BFKD ECGL nội tiếp đường tròn ∠GEC ⇒ ∠ABC = ∠KFD ∠GLC = Xét tam giác LIG kết hợp (1) ta có ∠LGF = ∠LGI = ∠LIF − ∠GLC = ∠ABC − ∠GEC = ∠ABC − ∠GFD ⇒ ∠LGF = ∠ABC − (∠KFD − ∠KFG ) = ∠ABC − ∠KBD + ∠KFG = ∠KFG ⇒ tam giác XFG cân X Mà tam giác AFG cân A ⇒ A, X , O thẳng hàng ⇒ X ∈ AO Bài 113 Cho tam giác ABC nội tiếp đường tròn (O), trực tâm H , trung tuyến AM Đường thẳng AH AM cắt đường tròn (O) K N Đường thẳng qua H vng góc với AM cắt BC KN P Q, KN cắt BC R Chứng minh ( PQR) tiếp xúc đường tròn (O) LỜI GIẢI Đường thẳng AO cắt đường tròn (O) E Gọi D hình chiếu H AM ⇒ ∠AKE =900 ⇒ KE song song với BC H K đối xứng qua BC ⇒ H , M , E thẳng hàng Đường thẳng HM cắt đường tròn (O) I ⇒ ∠AIH =900 HP cắt AM D ⇒ ∠ANK = ∠AEK (chắn cung AK ), ∠KAE = ∠KIE ⇒ ∠ADH = 900 ⇒ ∠DQN = ∠HQK = 900 − ∠ANK Liên hệ tài liệu word toán SĐT (zalo): 039.373.2038 75 Website: tailieumontoan.com = 900 − ∠AEK = ∠KAE = ∠KIE = ∠KIH ⇒ tứ giác QIHK nội tiếp đường tròn ⇒ ∠IQK = ∠IHA (1) Từ giả thiết suy H trực tâm tam giác APM Mà ∠AIM = 900 ⇒ A, I , Q thẳng hàng ⇒ ∠IHA = ∠APM ⇒ ∠IQK = ∠IPM ⇒ ∠IQR = ∠IPR ⇒ bốn điểm I , P, Q, R nằm đường tròn Qua I dựng tiếp tuyến Ix với đường tròn (O) ⇒ ∠xIK = ∠IAK ∠QIx = ∠QIK − ∠xIK = ∠QHK − ∠IAK = ∠APH = ∠IRQ ⇒ Ix tiếp tuyến đường tròn ( PQR) ⇒ đường tròn ( PQR) tiếp xúc đường tròn (O) Bài 114 Cho tam giác ABC khơng cân nội tiếp đường trịn (O) Đường phân giác góc A cắt BC D Từ B kẻ đường thẳng vng góc với AD cắt đường trịn ( ABD) E Chứng minh A, O, E thẳng hàng LỜI GIẢI Khơng tính tổng qt, giả sử AC > AB ∠ACB AO cắt đường tròn (O) K ⇒ ∠ABK =900 ∠AKB = ⇒ ∠BAO = 900 − ∠AKB = 900 − ∠ACB ⇒ ∠BAE = ∠BAD + ∠DAE = ∠BAD + ∠DBE = ∠BAC + ∠DBE 1 =∠BAC + (900 − ∠ADB) =∠BAC + 900 − ∠BAC − ∠ACB 2 = 900 − ∠ACB ⇒ ∠BAO = ∠BAE ⇒ A, O, E thẳng hàng Bài 115 Cho tam giác ABC Đường tròn (O) qua B C cắt AC D AB E Trên BD lấy điểm P cho BP = AC , CE lấy điểm Q cho CQ = AB Chứng minh tâm đường tròn ngoại tiếp tam giác APQ nằm đường tròn (O) LỜI GIẢI Đường tròn qua B, C cắt AC D cắt AB E ⇒ tứ giác BEDC nội tiếp ⇒ ∠EBD = ∠ECD Liên hệ tài liệu word toán SĐT (zalo): 039.373.2038 76 Website: tailieumontoan.com Cũng theo giả thiết, BP = AC CQ = AB ⇒ tam giác PBA tam giác ACQ (c.g.c) ⇒ AP = AQ, ∠BAP = ∠CQA, ∠APB = ∠QAC ⇒ ∠AEQ = 1800 − ∠BAC − ∠ACE = 1800 − ∠BAC − ∠ABD = ∠ADB Mặt khác ∠ADB = ∠APD + ∠PAD = ∠QAC + ∠PAC = ∠QAP Gọi I tâm đường tròn ngoại tiếp tam giác APQ ⇒ ∠AIQ = 2∠APQ = 1800 − ∠QAP ⇒ ∠AIQ = 1800 − ∠AEQ ⇒ tứ giác AIQE nội tiếp Tương tự: tứ giác AIDP nội tiếp ⇒ ∠EID = 3600 − ∠AIE − ∠AID = 3600 − ∠AQE − ∠AID = ∠AQC + ∠APB = ∠AQC + ∠QAC = 1800 − ∠DCE ⇒ ∠EID + ∠DCE = 1800 ⇒ tứ giác EIDC nội tiếp ⇒ I thuộc đường tròn ( BEDC ) Bài 116 Cho tứ giác ABCD ngoại tiếp đường tròn ( I ) Gọi E , F , P tiếp điểm tiếp xúc đường tròn ( I ) với cạnh AD, BC , CD Trên tia FE lấy điểm K cho DK = DE Chứng minh CI qua trung điểm KF LỜI GIẢI Theo giả thiết ta có DE = DP DK = DE ⇒ tam giác DEF , DKP, DKE cân D ⇒ ∠= EPD ∠= KPD (1800 − ∠PDE ), (1800 − ∠PDK ) Suy ra: ∠EPK = ∠EPD − ∠KPD = 1 (1800 − ∠PDE ) − (1800 − ∠PDK )= ∠EDK (1) 2 Ta có ∠FPE = ∠DKE (đối đỉnh), ∠DEK = ∠AEF (cùng chắn cung EF ), ∠AEF = ∠DKE ( DE = DK ) Kết hợp với (1) suy ra: ⇒ ∠FPK = ∠FPE + ∠EPK = ∠DEK + ∠EPK = 1 = 900 (1800 − EDK ) + ∠EDK 2 ⇒ PF vng góc với PK Mặt khác CP, CF tiếp tuyến với đường trịn ( I ) ⇒ CI vng góc với PF qua trung điểm PF CI cắt KF M Liên hệ tài liệu word toán SĐT (zalo): 039.373.2038 77 Website: tailieumontoan.com ⇒ CM song song với PK ⇒ M trung điểm KF Bài 117 Cho hai đường tròn (O1 ) ( O2 ) cắt P Q Tiếp tuyến chung xy cắt đường tròn (O1 ) (O2 ) A B Một đường qua A B, cắt đường tròn (O1 ) ( O2 ) D C Chứng minh CP DP = CQ DQ (China 2016) LỜI GIẢI Gọi giao điểm đường tròn (O1 ) ( O2 ) với DC M N Theo giả thiết đường tròn qua A B cắt (O1 ) ( O2 ) D C ⇒ tứ giác ABCD nội tiếp ⇒ ∠BCD = ∠DAx CD cắt (O1 ) ( O2 ) M N ⇒ tứ giác DMQA nội tiếp ⇒ ∠AMD = ∠DAx ⇒ ∠BCD = ∠AMD ⇒ BC song song với AM Tương tự AD song song với BN Suy ∠DAM = ∠NBC (góc có cạnh tương ứng song song) Mặt khác ∠DAM = ∠DQM ∠NQC = ∠NBC ⇒ ∠DQM = ∠CQN ⇒ QM , QN đường đẳng giác tam giác DQC ⇒ MD ND QD = MC NC QC Tứ giác DQPM CQPN nội tiếp ⇒ ∠DQM = ∠DPM , ∠CBN = ∠CQN ⇒ ∠DPM = ∠CPN ⇒ PM , PN đường đẳng giác tam giác PDC ⇒ MD ND PD QD PD CP DP = ⇒ = ⇒ = MC NC PC QC PC CQ DQ Bài 118 Cho tam giác ABC nội tiếp đường tròn (O) I tâm đường tròn nội tiếp tam giác Đường thẳng AI cắt BC D cắt đường tròn (O) E Gọi K L tâm đường tròn nội tiếp tam giác BDE CDE , P điểm đối xứng I qua KL Chứng minh PB vng góc với PC LỜI GIẢI AI phân giác góc ∠BAC ⇒ ∠BAE = ∠BAC K tâm đường tròn nội tiếp tam giác BDE 1 ⇒ ∠BKD = 900 + ∠BED = 900 + ∠ACB 2 Liên hệ tài liệu word toán SĐT (zalo): 039.373.2038 78 Website: tailieumontoan.com Mặt khác ∠AIB= 900 + ∠ACB ⇒ ∠BKD = ∠AIB ⇒ tứ giác BIDK nội tiếp Tương tự, tứ giác IDLC nội tiếp ⇒ ∠DIK = ∠DBK = ∠BAC Mà EB = EI ⇒ KI = KB I P đối xứng qua KL ⇒ KI = KP ⇒ KI = KB = KP Tương tự, LI = LC = LP ⇒ ∠KPL = ∠KIL ⇒ ∠BPC = ∠BPK + ∠KPL + ∠LPC = ∠KBP + ∠KIL + ∠LCP (1) Ta có ∠KIL = ∠KID + ∠DIL = ∠KBD + ∠DCL (2) Từ (1) (2) suy ra: ⇒ ∠BPC = ∠PBC + ∠PCB ⇒ 1800 = ∠BPC + ∠PBC + ∠PCB = 2∠BPC ⇒ ∠BPC =900 ⇒ PB vng góc với PC Bài 119 Cho tam giác BCF vuông B A điểm tia CF cho FA = FB D điểm thỏa mãn DA = DC AC phân giác ∠DAB E điểm thỏa mãn EA = ED AD phân giác góc ∠EAC M trung điểm CF , X điểm thỏa mãn cho AMXE hình bình hành Chứng minh BD, FX , ME đồng qui (IMO 2016 LỜI GIẢI Theo giả thiết FB = FA ⇒ ∠ABF = ∠BAF , DA = DC ⇒ ∠DAC = ∠DCA, EA = ED ⇒ ∠DAE = ∠ADE ⇒ ∠BAC = ∠CAD = ∠DAE ⇒ ∠CAD = ∠ADE ⇒ AC song song với DE Cũng theo giả thiết, AMXE hình bình hành ⇒ XE song song với AM ⇒ X , D, E thẳng hàng Tam giác ABF tam giác ACD đồng dạng (g.g) ⇒ AB AF = ⇒ tam giác BAC FAD đồng dạng (c.g.c) AC AD ⇒ ∠AFD = ∠ABC = 900 + ∠ABF ⇒ ∠BFD = 3600 − ∠BFA − ∠AFD = 3600 − (1800 − 2∠ABF ) − (900 + ∠ABF ) Liên hệ tài liệu word toán SĐT (zalo): 039.373.2038 79 Website: tailieumontoan.com = 900 + ∠ABF = ∠AFD ⇒ tam giác DFB tam giác DFA (c.g.c) ⇒ DB = DA ⇒ D tâm đường tròn ngoại tiếp tam giác ABC ⇒ ∠BDC = 2∠BAC = ∠BFC ⇒ tứ giác BFDC nội tiếp 2∠FAB = 2∠ABF = 2∠BAC ∠ABD = ∠OAB = = 2∠ADE = 1800 − ∠AED ⇒ tứ giác ABDE nội tiếp ⇒ ∠BED = ∠BAD = ∠BFC ∠FAE ⇒ EA = EF ⇒ B, F , E thẳng hàng, ∠EFA = Theo giả thiết AMXE hình bình hành ⇒ AE = MX ⇒ MX = EF ⇒ MXEF hình thang cân M trung điểm CF ⇒ ∠AMB = 1800 − 2∠BFC = ∠ADB BM ⇒ MBAD nội tiếp, hay đa giác MBAED thuộc đường tròn ⇒ AE = ⇒ MX = MB ⇒ X thuộc đường tròn ( BCDF ) ⇒ BD, XF , ME đường thẳng qua giao điểm hai ba đường tròn ( BMDEA), ( BCXDF ), ( MXEF ) Vậy BD, XF , ME đồng qui Bài 120 Cho hình thang ABCD ( AB // CD AB < CD) ngoại tiếp đường tròn tâm (O ') Đường tròn tâm ( I ) nội tiếp tam giác BCD, tiếp xúc cạnh BD, DC N M Chứng minh M , N , O ' thẳng hàng LỜI GIẢI CI cắt MN O CI phân giác góc ∠BCD BI phân giác ∠CBD ⇒ ∠ION = ∠ICM + ∠OMC = ∠BCD + (1800 − ∠DMN ) 1 900 + (∠BCD + ∠BDC ) =∠BCD + 1800 − (900 − ∠MDN ) = 2 = 1800 − ∠DBC = 1800 − ∠NBI ⇒ tứ giác NBIO nội tiếp N tiếp điểm cạnh BD với đường tròn nội tiếp tâm I ⇒ ∠INB = 900 ⇒ ∠COB = 900 ⇒ ∠OBC = 900 − ∠BCO = 900 − ∠BCD Liên hệ tài liệu word toán SĐT (zalo): 039.373.2038 80 Website: tailieumontoan.com 1 = 900 − (1800 − ∠ABC )= ∠ABC 2 ⇒ BO phân giác góc ∠ABC ⇒ O tâm đường trịn nội tiếp hình thang ABCD ⇒ O trùng O ' Bài 121 Cho tam giác vuông ABC vuông A D cạnh AC cho BC = AD Đường tròn ngoại tiếp tam giác BDC cắt đường tròn qua A, C trung điểm M BC P Chứng minh CP phân giác góc ∠ACB LỜI GIẢI Gọi M trung điểm BC ⇒ AD = AM = BM Tứ giác BPDC nội tiếp ⇒ ∠ADP = ∠PBC Tứ giác APMC nội tiếp ⇒ ∠PAD = ∠PBM ⇒ tam giác APD tam giác PBM (g.c.g) ⇒ AP = PM ⇒ ∠ACP = ∠PCM ⇒ CP phân giác góc ∠ACB Bài 122 Cho tam giác nhọn ABC nội tiếp đường tròn (O) H trực tâm tam giác, M trung điểm 900 AH , N điểm cạnh AC cho góc ∠BMN = Chứng minh ON song song với BC LỜI GIẢI Gọi chân đường vng góc A B cạnh đối diện D E Theo giả thiết ∠BMN =900 ⇒ tứ giác BMEN nội tiếp đường trịn đường kính BN ⇒ ∠MBN = ∠AEM Tam giác AEH vuông E MH = MA ⇒ ∠AEM = ∠MAE ⇒ ∠MBN = ∠AEM = ∠EAM = ∠EBC ⇒ ∠MBE = ∠MBN − ∠EBN = ∠EBC − ∠EBN = ∠NBC Tam giác OBC cân O, ∠BOC = 2∠BAC ⇒ ∠OBN = ∠OBC − ∠NBC = 900 − ∠BAC − ∠MBE = ∠ABE − ∠MBE = ∠ABM Mặt khác AD AO đường đối trung ⇒ O, M điểm đẳng giác tam giác ABN ⇒ ∠BNO = ∠ANM , ∠ENM = ∠EBM (chắn cung ME ) ⇒ ∠BNO = ∠EBM , ∠ONB = ∠NBC ⇒ ON song song BC Bài 123 Cho tam giác cân ABC , AB = AC , D hình chiếu C AB Đường trịn ( B, BD) Liên hệ tài liệu word toán SĐT (zalo): 039.373.2038 81 Website: tailieumontoan.com đường tròn (C , CD) cắt E khác D Đường thẳng AC cắt đường tròn (C , CD) P Q Đường thẳng QD cắt đường tròn ( B, BD) M 1) Tính góc ∠QDE 2) Chứng minh P, M , E thẳng hàng 3) Chứng minh BM song song với CE LỜI GIẢI 1) Ta = có BD BE , CD CE CD ⊥ AB = ⇒ CE ⊥ BE ⇒ ∠QDE = ∠ECQ = (1800 − ∠ACB − ∠BCE ) = (1800 − ∠ACB − ∠BCD) = (1800 − 900= ) 450 2) Cũng từ giả thiết suy ∠MBE = 2∠MDE = 2∠QDE = 900 ⇒ ∠BEM = ∠BME = (1800 − 900 ) = 450 , ∠CEM = ∠CEB − ∠BEM = 900 − 450 = 450 Tam giác CPE cân C ⇒ ∠CEP = ∠CPE = 450 ⇒ ∠CPE = ∠QPE = ∠QDE = 450 Mặt khác D, P, Q, E thuộc đường tròn (C , CD) ⇒ E , M , P thẳng hàng 3) ∠BME = 450 ⇒ BM song song với CE ∠MEC = Bài 124 Cho hai đường tròn (O1 ) (O2 ) cắt A B P điểm đường tròn (O1 ), Q điểm đường tròn (O2 ) cho AP = AQ Đường thẳng PQ cắt đường tròn (O1 ) M , đường tròn (O2 ) N Gọi C điểm cung PB khơng chứa A, D điểm cung BQ không chứa A, E giao điểm MC ND Chứng minh AE vng góc với CD LỜI GIẢI Ta có ∠CAD = ∠CAB + ∠BAD = 1 ∠PAB + ∠BAQ = ∠PAQ 2 Theo giả thiết, AP = AQ Liên hệ tài liệu word toán SĐT (zalo): 039.373.2038 82 Website: tailieumontoan.com ⇒ tam giác APQ cân A ⇒ ∠PAQ = 1800 − 2∠AQP ⇒ ∠CAD = 900 − ∠AQP = 900 − ∠AQN Mặt khác ∠AQN = ∠ADN ⇒ ∠CAD + ∠AQN = 900 ⇒ CM vng góc với AD Tương tự, DN vng góc với AC ⇒ E trực tâm tam giác ACD ⇒ AE vng góc với CD Bài 125 Cho tứ giác ABCD nội tiếp đường tròn ( AB < CD) AD BC cắt E , AC BD cắt F Gọi M , N , I trung điểm EF , FD, FC H , K hình chiếu vng góc F BC , AD Chứng minh đường tròn ngoại tiếp tam giác MKN tam giác MHI cắt trung điểm CD LỜI GIẢI Gọi P trung điểm DE Xét tam giác DEF , theo giả thiết, M , N trung điểm EF , DF FK vng góc với DE nên theo tính chất đường trịn Euler ta có M , N , P, K nằm đường tròn ⇒ ∠PMN = ∠PDF = ∠ADB = ∠ACB (1) Gọi Q trung điểm CD ⇒ PQ song song với CE , NQ song song với AC ⇒ ∠ACB = ∠NQP (2) Từ (1) (2) suy ∠PMN = ∠PQN ⇒ tứ giác PMQN nội tiếp ⇒ điểm M , N , P, K , Q thuộc đường trịn Đường trịn đường trịn ngoại tiếp tam giác KMN Tương tự M , H , I , Q nằm đường Đường tròn ngoại tiếp tam giác MHI qua Q trung điểm cạnh CD Bài 126 Cho tam giác ABC nội tiếp đường tròn (O) Gọi D, E , F thứ tự trung điểm cạnh BC , CA, AB, H hình chiếu cạnh BC Đường thẳng DF , HF cắt nửa đường trịn đường kính AB phía ngồi tam giác ABC thứ tự P M Đường thẳng DE , HE cắt nửa đường trịn đường kính AC phía ngồi tam giác ABC thứ tự Q N 1) Chứng minh PQ MN cắt A 2) Đường thẳng PM QN cắt K Chứng minh DK cắt PQ điểm đường trịn (O) Liên hệ tài liệu word tốn SĐT (zalo): 039.373.2038 83 Website: tailieumontoan.com LỜI GIẢI 1) Theo giả thiết AB, AC đường kính đường trịn, HF cắt nửa đường tròn M ⇒ ∠AMB =900 Kết hợp AH vng góc với BC ⇒ tứ giác AMBH tứ giác nội tiếp MH AB cắt trung điểm AB ⇒ AMBH hình chữ nhật ⇒ AM song song với BH Tương tự AN song song với HC ⇒ M , A, N thẳng hàng F trung điểm AB ⇒ F tâm nửa đường trịn đường kính AB ⇒∠ = PAB 1 ∠ = PFB (1800 − ∠= PFA) (1800 − ∠BFD) 2 Cũng theo giả thiết D trung điểm BC ⇒ ∠BFD = ∠BAC ⇒ ∠PAB = Tương tự ∠= QAC (1800 − ∠BAC ) (1800 − ∠CAB) ⇒ ∠PAB + ∠BAC + ∠QAC =1800 ⇒ P, A, Q thẳng hàng ⇒ MN PQ cắt A 2) Gọi I điểm cung BAC Ta có ∠IAH = ∠IAC + ∠CAH = (∠ABC + ∠ACB) + (900 − ∠ACB) = 1800 − ∠BAC − ∠ACB, ∠PAH = ∠PAB + ∠BAH = (1800 − ∠BAC ) + (900 − ∠ABC ) = 1800 − ∠ABC − ∠BAC = ∠BAC + ∠ACB ⇒ ∠IAH + ∠PAH = 1800 ⇒ P, A, I thẳng hàng ⇒ I thuộc PQ Liên hệ tài liệu word toán SĐT (zalo): 039.373.2038 84 Website: tailieumontoan.com Tứ giác ABPM nội tiếp ⇒ ∠KMA = ∠ABP =− ∠BAC 900 ∠PAB = Tương tự, ∠KNA = ∠QCA = ∠BAC ⇒ ∠KMA = ∠KNA ⇒ tam giác KMN cân K ⇒ tam giác KBC cân K ⇒ KD vng góc với BC ⇒ I thuộc giao điểm PQ với đường tròn (O) Bài 127 Cho tam giác vuông ABC vuông A, đường cao AH D điểm tam giác AHC cho AH qua trung điểm BD CD cắt AH E Qua E kẻ đường thẳng tiếp xúc với nửa đường trịn đường kính DC phía cạnh AC M Chứng minh BD AM cắt điểm nửa đường tròn đường kính CD (Dự tuyển IMO 2015 – Iran 2016) LỜI GIẢI * Cách Gọi N giao điểm BD AM EM tiếp tuyến với nửa đường trịn đường kính DC ⇒ ∠EMD = ∠MCD ⇒ tam giác EMC tam giác EDM đồng dạng (g.g) ⇒ DM EM ED = = ⇒ EM = ED.EC MC EC EM ⇒ DM EM ED.EC ED = = = (1) MC EC EC EC Tam giác ABC vuông A, AH vng góc BC AB BH = (2) BH BC , AC = CH CB ⇒ ⇒ AB = AC CH 2 Gọi K hình chiếu D BC ⇒ AH song song với DK , AH qua trung điểm BD theo định lí Thales ⇒ BH = HK ⇒ ED HK BH = = (3) EC HC HC Từ (1), (2) (3) suy DM AB DM AB = ⇒ = 2 MC AC MC AC Theo giả thiết M điểm nửa đường tròn đường kính CD ⇒ ∠DMC =900 ⇒ tam giác ABC tam giác MDC đồng dạng (c.g.c) ⇒ ∠ACB = ∠MCD ⇒ ∠ACM = ∠BCD Mà MC CD = AC CB Liên hệ tài liệu word toán SĐT (zalo): 039.373.2038 85 Website: tailieumontoan.com ⇒ tam giác ACM tam giác BCD đồng dạng (c.g.c) ⇒ ∠MAC = ∠DBC ⇒ ∠NAC = ∠NBC ⇒ tứ giác ABCN nội tiếp ⇒ ∠BNC = 900 ⇒ ∠DNC = 900 ⇒ tứ giác DMNC nội tiếp * Cách Gọi E giao điểm BD nửa đường trịn đường kính CD, G giao điểm AE với nửa đường trịn đường kính CD, I giao điểm AC DG, K giao điểm AC với nửa đường trịn đường kính CD Ta có ∠BEC = ∠DEC = 900 , ∠BAC =900 ⇒ BAEC tứ giác nội tiếp DC đường kính ⇒ ∠DGC =900 ⇒ ∠GDC = 900 − ∠GCD = 900 − ∠GED = 900 − ∠AEB = 900 − ∠ACB = ∠ABC = ∠HAC ⇒ tứ giác APDI nội tiếp K giao điểm BC với đường trịn đường kính CD ⇒ ∠DKC =900 ⇒ DK vng góc với BC HK AH vng góc BC ⇒ DK song song với AH Theo giả thiết, AH qua trung điểm BD ⇒ HB = ∠APD = ∠PDK (so le trong) Tứ giác APDI nội tiếp ⇒ ∠ADP = ∠AIP, ∠AIG = ∠APD ⇒ ∠AIG = ∠PDK ⇒ ∠PIG = ∠PIA + ∠AIG = ∠ADP + ∠PDK = ∠ADK (1) Mặt khác ∠ACG = ∠AIG − 900 = ∠PDK − 900 = ∠BCD ⇒ tam giác GCI đồng dạng với tam giác KCD (g.g) ⇒ IG IC = (2) DK DC Từ ∠IAD = ∠IPD ⇒ tam giác PIC đồng dạng với tam giác ADC ⇒ IC PI = (3) DC AD Từ (2) (3) suy IG PI = DK AD Kết hợp với (1) ta được: hai tam giác ADK , PIG đồng dạng (c.g.c) ⇒ ∠PGD = ∠PGI = ∠AKD = ∠KAH = ∠BAH = ∠ACB = ∠GCD ⇒ PG tiếp tuyến đường trịn đường kính CD ⇒ G ≡ Q Liên hệ tài liệu word toán SĐT (zalo): 039.373.2038 86 Website: tailieumontoan.com ⇒ BD, AM cắt đường trịn đường kính CD Bài 128 Cho tam giác nhọn ABC , trực tâm H Dựng hình bình hành ABGH Kéo dài GH lấy I cho AC qua trung điểm HI Đường tròn qua G, C , I cắt AC J Chứng minh AH = IJ (Dự tuyển IMO 2015) LỜI GIẢI * Cách Theo giả thiết ABGH hình bình hành ⇒ AH song song BG H trực tâm tam giác ABC ⇒ AH vng góc với BC ⇒ ∠GBC =900 AB song song GH CH vng góc AB ⇒ ∠GHC = 900 ⇒ G, B, H , C nằm đường trịn đường kính GC Gọi E giao điểm đường tròn (GBHC ) với cạnh CA, M giao điểm GH với AC Theo hệ thức đường tròn ⇒ ME.MC = MH MG = MG.MI = MC.MJ ⇒ ME = MJ Theo giả thiết = MI MH ⇒ tứ giác HEIJ hình bình hành ⇒ HE = IJ H trực tâm ⇒ ∠HAE = ∠HBC Tứ giác BHEC nội tiếp ⇒ ∠HBC = ∠HEA ⇒ ∠HAE = ∠HEA ⇒ tam giác AHE cân H ⇒ HA = HE ⇒ HA = IJ * Cách Theo giả thiết ABGH hình bình hành ⇒ AH song song BG H trực tâm tam giác ABC ⇒ AH vng góc với BC ⇒ ∠GBC =900 AB song song GH CH vng góc AB ⇒ ∠GHC = 900 ⇒ G, B, H , C nằm đường tròn ⇒ ∠HBC = ∠HGC = ∠IGC Từ I kẻ đường thẳng song song với AH cắt AC K ⇒ ∠HAK = ∠JKI Do H trực tâm ⇒ ∠HBC = ∠HAC Tứ giác JICG nội tiếp Liên hệ tài liệu word toán SĐT (zalo): 039.373.2038 87 Website: tailieumontoan.com ⇒ ∠IJC = ∠IGC ⇒ ∠IKJ = ∠IJK ⇒ tam giác IJK cân I ⇒ IJ = IK Theo giả thiết, AC qua trung điểm HI ⇒ tam giác MAH tam giác MKI (g.c.g) ⇒ AH = IK ⇒ IJ = AH Bài 129 Cho tam giác ABC Trên AC lấy điểm E AB lấy điểm F cho CE = BF Đường tròn qua A, E , B đường tròn qua A, F , C cắt D khác A Chứng minh AD phân giác góc BAC LỜI GIẢI Đường tròn qua A, F , C cắt cạnh BC P BP.BC ⇒ BF BA = Đường tròn qua A, E , B cắt BC Q ⇒ CE.CA = CQ.CB Chia hai đẳng thức, kết hợp với CE = BF suy BA BP = CA CQ AD giao với BC I , theo hệ thức đường trịn, ta có IA.ID = IB.IQ IA.ID = IP.IC ⇒ IB.IQ = IP.IC ⇒ IB IP IB − IP BP IB AB = = = ⇒ = IC IQ IC − IQ CQ IC AC ⇒ AD phân giác góc ∠BAC Bài 130 Cho tam giác ABC Đường tròn nội tiếp tam giác ABC tiếp xúc cạnh BC , CA, AB D, E , F AD cắt đường tròn nội tiếp I Đường thẳng DE , DF cắt đường thẳng qua A song song với BC M N Chứng minh ∠EIF = ∠MIN LỜI GIẢI Theo giả thiết, BC , AB tiếp xúc đường tròn nội tiếp tam giác ABC D F ⇒ ∠FID = ∠FDB = ∠BFD MN song song với cạnh BC ⇒ ∠BDN = ∠AND (so le trong) Mà ∠BFD = ∠NFA (đối đỉnh) Liên hệ tài liệu word toán SĐT (zalo): 039.373.2038 88 Website: tailieumontoan.com ⇒ ∠ANF = ∠NFA = ∠FID Từ chứng minh suy ∠FID = ∠ANF ⇒ tứ giác ANFI nội tiếp ⇒ ∠NFA = ∠NIA ⇒ ∠NIA = ∠FID Tương tự ∠AME = ∠EID ⇒ tứ giác AMEI nội tiếp ⇒ ∠AIM = ∠EID ⇒ ∠EIF = ∠EID + ∠DIE = ∠NIA + ∠AIM = ∠NIM Bài 131 Cho tam giác ABC vng A Đường trịn nội tiếp tam giác ABC tiếp xúc với AC , AB M N Đường thẳng MN cắt BC E Tiếp tuyến với đường tròn ngoại tiếp tam giác ABC cắt BC D AD MN cắt F Chứng minh tam giác DEF cân LỜI GIẢI Theo giả thiết, AD tiếp tuyến đường tròn ngoại tiếp tam giác ABC ⇒ ∠DAB = ∠ACB Góc ∠ADE góc ngồi tam giác ADB ⇒ ∠ADE = ∠ABD + ∠DAB = 1800 − ∠ABC + ∠ACB = 1800 − ∠ABC + 900 − ∠ABC = 2700 − 2∠ABC (1) Gọi I tâm đường tròn nội tiếp tam giác ABC ⇒ AMIN hình vng ⇒ ∠AMN =450 Tam giác AFM ta có: ∠EFD = ∠AFM = 1800 − ∠FAM − ∠AMF 450 − ∠ACB = 1800 − ∠FAB − 900 − 45= Kết hợp với (1), ta được: ∠FED = 1800 − ∠EDF − ∠EFD = 1800 − 2700 + 2∠ABC − 450 + ∠ACB = 2∠ABC + ∠ACB − 1350 = 2(900 − ∠ACB) + ∠ACB − 135 = 45.0 − ∠ACB ⇒ ∠DEF = ∠DFE ⇒ tam giác DEF cân D Bài 132 Cho tam giác ABC vuông A, AB = Ba đường trịn có bán kính tiếp xúc với cạnh AC (như hình vẽ) Tính độ dài AC Liên hệ tài liệu word toán SĐT (zalo): 039.373.2038 89 Website: tailieumontoan.com LỜI GIẢI Trước hết ta chứng minh tính chất: Hai đường trịn (O1 , R1 ) (O2 , R2 ) rời nhau, tiếp tuyến chung chung cắt C , A B tiếp điểm tiếp tuyến chung ngoài: Chứng minh AC.CB = R1.R2 Thật vậy, ta có CO1 CO2 hai đường phân giác tạo hai tiếp tuyến ⇒ CO1 CO2 vng góc với ⇒ ∠O1CA = ∠CO2 B ⇒ hai tam giác vuông AO1C BCO2 đồng dạng (g.g) ⇒ AO1 AC = ⇒ AC.CB = R1.R2 BC BO2 Trở lại tốn, tâm đường trịn nằm đường thẳng song song với AC cách AC khoảng 1, AB = 4, MA = ⇒ BM = 3, BN = BM Đặt DN =⇒ x DP = x Theo định lí Pythagoras với tam giác vng ABD : AB + AD = BD ⇒ 162 + (1 + x) = (3 + x) ⇒ x = ⇒ AD = Áp dụng toán ta DP.DQ = R1.R= 1.1 = ⇒ DQ = ⇒ AQ = 2 Tương tự ta có: 225 289 16 + ( + EQ) =( + EQ) ⇒ EQ =4 ⇒ BE =16 + = 2 4 Liên hệ tài liệu word toán SĐT (zalo): 039.373.2038 90 Website: tailieumontoan.com Tiếp tục cách làm trên, ta có: EQ.EI =⇒ EI = 1 31 ⇒ AI = AD + DQ + QE + EI = + + + = 4 ⇒ 16 + ( 31 33 + CI ) = ( + CI ) ⇒ CI = 4 ⇒ AC = AI + IC = 15, 75 Nhận xét: Ta có tốn sau: Cho tam giác ABC đường tròn nội tiếp tam giác tiếp xúc cạnh BC D Chứng minh BD.DC = r 2h (trong h đường cao kẻ từ A r bán kính đường trịn nội tiếp) h − 2r Chứng minh: p = (a + b + c) ⇒ BD = p − b, CD = p − c Mặt khác S ABC =pr = ah = p ( p − a )( p − b)( p − c) ⇒ ( p − b)( p= − c) S2 prah rah rh r 2h = = = = p ( p − a ) p ( p − a ) 2( p − a ) 2( h − 1) h − 2r 2r Bài tốn tổng qt: BA0 = Các đường trịn bán kính tiếp xúc với cạnh A0 An điểm D1 , D2 , D3 BA1 , BA2 , BA3 , BAn tiếp xúc với đường trịn bán kinh Tính độ dài A0 An Lời giải: Áp dụng bổ đề ta có: A0 D1 1,= D1 A1 2,= A1 D2 = 1 D2 A2 4,= A2 D3 Dn An 2n ,= = = A0 A= ( A0 D1 + A1 D2 + + An −1 Dn ) + ( D1 A1 + D2 A2 + Dn An ) n 1 = (1 + + + n−1 ) + (2 + + 2n ) 2 = (2 − 1 ) + (2n +1 − 2) = 2n +1 − n −1 n −1 2 Liên hệ tài liệu word toán SĐT (zalo): 039.373.2038 91 Website: tailieumontoan.com Bài 133 Cho ba điểm A, B, C thẳng hàng theo thứ tự thỏa mãn AB = BC Dựng đường tròn đường kính AC, AB, BC Xác định tâm O bán kính đường trịn tiếp xúc với đường trịn đường kính AC, AB, BC LỜI GIẢI Khơng tính tổng quát, đặt BC = gọi tâm đường trịn đường kính AC, BC, AB O, O1 , O2 tâm đường tròn cần xác định O3 Gọi bán kính đường trịn tâm (O) r ⇒ O1O3 = + r , O2 O3 = + r , OO3 = − r , O1O2 = Theo định lí Stewart , ta có ; O2 O32 O1O + O1O32 OO2 = O2 O1 O3 O + O2 O1 OO2 OO1 ⇒ ( + r ) + (1 + r ) = ( − r ) + 3.1.2 2 Giải phương trình ta 28r = 24 ⇒ r = 20 13 Từ xác định tâm O3 giao điểm hai đường tròn O2 ; O1 ; 7 Bài 134 Cho tứ giác ABCD, AB CD không song song với Đường tròn ( C1 ) qua A, B tiếp xúc với CD P, đường tròn ( C2 ) qua C D tiếp xúc với AB Q Hai đường tròn ( C1 ) ( C2 ) giao E F Chứng minh EF qua trung điểm PQ AD BC song song LỜI GIẢI Gọi giao điểm EF PQ K Đường thẳng PQ cắt đường tròn ( C1 ) ( C2 ) M N, AB CD cắt O Theo giả thiết đường tròn ( C1 ) ( C2 ) tiếp xúc với AB CD P Q, theo hệ thức đường tròn ⇒ OP = OA.OB OQ = OC.OD Cũng theo hệ thức đường tròn : KP.KM = KE.KF KE.KF = KQ.KN ⇒ KP.KM= KQ.KN ⇒ KP ( KQ + QM )= KQ ( KP + KN ) ⇒ KP.QM = KQ.PN Ta có KP = KQ ⇔ QM = PN ⇔ PQ.QM = PQ.PN ⇔ QA.QB = PC.PD Liên hệ tài liệu word toán SĐT (zalo): 039.373.2038 92 Website: tailieumontoan.com ⇔ ( OQ − OA )( OB − OQ ) = ( OC − OP )( OP − OD ) ⇔ OQ.OB − OQ − OA.OB + OA.OQ= OC.OP − OC.OD − OP + OP.OD ⇔ OQ.OB − OC.OD − OA.OB + OA.OQ = OC.OP − OC.OD − OA.OB + OP.OD ⇔ OQ ( OA + OB= ) OP ( OC + OD ) ⇔ OQ ( OA + OB )= OP ( OC + OD ) 2 ⇔ OC.OD ( OA + OB = = ) OA.OB ( OC + OD ) ⇔ ( OA.OC − OB.OD ) ⇔ 2 OA OD = ⇔ AD song song với BC OB OC Bài 135 Cho đường tròn (O) , tiếp xúc với cạnh AB, AC tam giác ABC Q điểm góc ∠BAC Trên AQ lấy điểm P cho OP vng góc với AQ Đường trịn ngoại tiếp tam giác BPQ CPQ cắt OP M N Chứng minh OM = ON LỜI GIẢI Kéo dài AB cắt đường tròn ngoại tiếp tam giác BPQ D, kéo dài AC cắt đường tròn ngoại tiếp tam giác CPQ E Ta có AB AD = AP AQ AC AE = AP AQ ⇒ AB AD = AC AE Theo giả thiết, AB, AC tiếp xúc với đường tròn (O) ⇒ AB = AC ⇒ AD = AE ⇒ tam giác ADE cân A Vì OB, OC bán kính đường trịn (O) nên ∠BAO = ∠CAO ⇒ AO vng góc với DE Gọi giao điểm AO với DE H ⇒ HD = HE Theo giả thiết ∠ABO = ∠ACO = ∠APO = 90° ⇒ điểm A, B, P, O, C nằm đường trịn đường kính AO Mà tứ giác BPQD, CPQE nội tiếp ⇒ ∠PQD = 180° − ∠PBD = ∠PBA = 180° − ∠ACP = ∠PCE = 180° − ∠PQE ⇒ Q, D, E thẳng hàng hay Q thuộc DE Tứ giác PMDQ nội tiếp, MP vng góc PQ ⇒ ∠MDQ= 90° ⇒ MD vng góc với DE; Tương tự NE vng góc DE ⇒ MD, AH , NE song song với Mà HD =HE ⇒ OM =ON Bài 136.Cho tam giác nhọn ABC nội tiếp đường tròn ( O )( BC > CA > AB ) , H trực tâm tam giác ABC Liên hệ tài liệu word toán SĐT (zalo): 039.373.2038 93 Website: tailieumontoan.com Đường phân giác góc ∠ABC cắt đường tròn ngoại tiếp tam giác BHC M P trực tâm tam giác BMC 1) Chứng minh A, P, C, B nằm đường tròn 2) Đường thẳng qua H song song với AO cắt BC E, F cạnh BC cho BE = CF Chứng minh A, O, F thẳng hàng 3) Gọi N tâm đường tròn ngoại tiếp tam giác ABM Chứng minh PN = PO (Đề thi học sinh giỏi lớp Hà Nội năm 2017) LỜI GIẢI 1) Theo giả thiết, H trực tâm tam giác ABC ⇒ ∠BHC = 180° − ∠BAC (*) M giao điểm đường phân giác góc ∠ABC đường tròn ngoại tiếp tam giác BHC ⇒ tứ giác BHMC nội tiếp ⇒ ∠BMC = ∠BHC Mặt khác P trực tâm tam giác BMC ⇒ ∠BMC = 180° − ∠BPC Thay vào (*) ⇒ ∠BAC = ∠BPC Vậy A, P, C, B nằm đường tròn 2) Từ câu 1) ta suy P thuộc đường tròn ngoại tiếp tam giác ABC ⇒ O tâm đường tròn ngoại tiếp tứ giác APCB Kéo dài AO cắt đường tròn (O) K cắt BC G ⇒ AK đường kính ⇒ KB vng góc với AB, H trực tâm ⇒ CH vng góc với AB ⇒ KB song song với CH Tương tự KC song song với BH ⇒ tứ giác BHCK hình bình hành ⇒ BH = CK ∠BHE = ∠CKG Theo giả thiết, AO song song HE ⇒ ∠HEB = ∠OGB = ∠CGK ⇒ hai tam giác HBE KCG (g.c.g) ⇒ BE = CG ⇒ G ≡ F ⇒ A, O, F thẳng hàng 3) Tứ giác BHCK hình bình hành ⇒ tam giác BHC tam giác BKC Gọi I tâm đường tròn ngoại tiếp tam giác BHC Tứ giác ABKC nội tiếp đường tròn (O) ⇒ O tâm đường tròn ngoại tiếp tam giác BKC ⇒ IB = IC = OM = OC Theo giả thiết, N tâm đường tròn ngoại tiếp tam giác ABM ⇒ ON trung trực AB, OI trung trực BC, IN trung trực BM ⇒ ∠ONI = ∠OIN = ∠ABC ⇒ ∠POB = 2∠PCB = ( 90° − ∠MBC ) = 180° − ∠ABC Liên hệ tài liệu word toán SĐT (zalo): 039.373.2038 94 Website: tailieumontoan.com ⇒ Tam giác PON tam giác BOI Mà tam giác BOI cân B ⇒ PN = PO Bài 137.Cho tứ giác ABCD nội tiếp đường tròn AC BD cắt E M N trung điểm AB CD Gọi G, K hình chiếu E BC AD Chứng minh MN vng góc với KG LỜI GIẢI Gọi P, Q thứ tự trung điểm ED E Theo giả thiết, N trung điểm CD ⇒ PN song song với CE NQ song song với ED ⇒ tứ giác EPNQ hình bình hành ⇒ ∠EPN = ∠EQN K hình chiếu E AD ⇒ EK vng góc AD ⇒ PE = PD = PK Mà EP = QN ⇒ QN = KP EG vng góc BC ⇒ QG = QE = QC ⇒ ∠GQE = 2∠GCE = 2∠BCA PN = GQ ⇒ ∠KPN = ∠KPE + ∠EPN = 2∠ADE + ∠EPN = 2∠ADB + ∠EPN = 2∠ACB + ∠EQN = ∠GQN NG ⇒ tam giác KPN tam giác NQG ( c.g c ) ⇒ NK = Tương tự chứng minh MK = MG ⇒ tam giác KMN tam giác GMN ( c.c.c ) ⇒ MN trục đối xứng KG ⇒ MN vuông góc với KG Bài 138.Cho tam giác ABC khơng cân nội tiếp đường trịn (O) Đường phân giác góc ∠BAC cắt cạnh BC D Qua B kẻ đường vuông góc với AD cắt đường trịn ngoại tiếp tam giác ABD E Chứng minh A, O, E thẳng hàng LỜI GIẢI Khơng tính tổng qt giả sử AB < AC Ta có OA= OB ⇒ ∠BAO= 90° − ∠AOB= 90° − ∠ACB Theo giả thiết, E thuộc đường tròn ngoại tiếp tam giác ABD ⇒ ∠EBD = ∠EAD BE vng góc với AD ⇒ ∠ADB= 90° − ∠DBE AD phân giác góc ∠BAC ⇒ ∠ADB = ∠DAC + ∠DCA Liên hệ tài liệu word toán SĐT (zalo): 039.373.2038 95 Website: tailieumontoan.com = ∠BAC + ∠ACB ⇒ ∠BAE = ∠BAD + ∠DAE = ∠BAD + ∠DBE = ∠BAD + ( 90° − ∠ADB ) = 1 ∠BAC + 90° − BAC − ∠ACB 2 = 90° − ∠ACB ⇒ ∠BAE = ∠BAO ⇒ A, O, E thẳng hàng Bài 139 Cho tam giác ABC nội tiếp đường tròn (O) H trực tâm Đường phân giác góc ∠BAC cắt cạnh BC D, M trung điểm BC, MO cắt đường tròn ngoại tiếp tam giác BHC N (N tam giác ABC).Chứng minh ∠ADO = ∠HAN (USA JMO 2017) Bổ đề Tam giác ABC nội tiếp đường tròn (O) , trực tâm H, đường cao kẻ từ A cắt đường tròn ngoại tiếp tam giác ABC K Chứng minh H K đối xứng với qua cạnh BC, từ suy cung BHC cung BKC đối xứng với qua BC LỜI GIẢI Gọi BE, CF đường cao tam giác ⇒ tứ giác AFHE, BFEC nội tiếp ⇒ ∠AFE = ∠AHE = ∠BHD ∠AFE = ∠ACB = ∠AKB ⇒ ∠BHD = ∠BKD AD vng góc với BC ⇒ tam giác DHB DKB (g.c.g) ⇒ DK = DH , hay H K đối xứng qua cạnh BC ⇒ cung BHC cung BKC đối xứng qua BC * Cách Theo giả thiết, M trung điểm BC ⇒ đường thẳng MO cắt đường tròn (O) P Q ⇒ PQ đường kính đường trịn (O) ⇒ PQ vng góc với BC H trực tâm tam giác ABC ⇒ AH vng góc BC ⇒ AH song song PQ ⇒ ∠HAN = ∠ANQ(*) Theo bổ đề ta có ∠DNP = ∠DPN Tam giác OAP cân O ⇒ ∠OAP = ∠OPA ⇒ ∠DNP = ∠OAD ⇒ tứ giác ANOD nội tiếp Liên hệ tài liệu word toán SĐT (zalo): 039.373.2038 96 Website: tailieumontoan.com ⇒ ∠ADO = ∠ANQ Kết hợp (*) ⇒ ∠HAN = ∠ADO *Cách Tam giác ODP: ∠ADO = ∠DOP + ∠OPD = ∠DOP + ∠HAD Ta có: ∠HAN = ∠HAD + ∠DAN Tứ giác AQMD nội tiếp ⇒ PD.PA = PM PQ Mặt khác theo bổ đề ⇒ MP = MN OP = OQ ⇒ PM PQ = PN PO = PN PM ⇒ PD.PA = PO.PN ⇒ tứ giác ANOD nội tiếp ⇒ ∠DOP = ∠DAN ⇒ ∠HAN = ∠ADO Bài 140 Cho tam giác ABC ( AB < AC ) Đường phân giác góc ∠BAC cắt đường trịn ngoại tiếp tam giác D E giao điểm đường phân giác ngồi góc ∠BAC với trung trực AC Chứng minh trung điểm đoạn thẳng AB nằm đường tròn ngoại tiếp tam giác ADE (APMO 2017) LỜI GIẢI * Cách Gọi I, N, M trung điểm BC , CA, AB ⇒ DI vng góc với BC EN vng góc CA Ta có ∠DBC = ∠DAC , AE phân giác ngồi góc ∠BAC ⇒ AE vng góc với AD ⇒ ∠AEN = ∠DAC ⇒ ∠DBC = ∠AEN ⇒ tam giác IBD tam giác NEA đồng dạng (g.g) ⇒ ID NA = BI EN Mà= MN BI = , NA IM ⇒ ID IM = MN EN Mặt khác ∠MIB = ∠ANM ⇒ ∠DIM = ∠DIB + ∠BIM = ∠ENA + ∠ANM = ∠ENM ⇒ tam giác DIM tam giác MNE đồng dạng (c.g.c) ⇒ ∠EMN = ∠MDI = 90° − ∠NMD Liên hệ tài liệu word toán SĐT (zalo): 039.373.2038 97 Website: tailieumontoan.com ⇒ ∠EMN + ∠NMD= 90° ⇒ ∠DME= ∠DAE= 90° ⇒ M thuộc đường tròn ngoại tiếp tam giác ADE * Cách Gọi M trung điểm AB, N điểm đối xứng D qua M ⇒ AN = DB = DC Theo giả thiết EO trung trực AC ⇒ EA = ED Mặt khác, ∠NAE = 360° − ∠NAB − ∠EAB = 360° − ∠ABD − ∠EAB = 360° − ∠ABD − ( ∠BAC + ∠EAC ) = 180° + ∠ACD − ∠BAC − ∠EAC = ∠ACD + ∠ACE = ∠DCE ⇒ tam giác NAB tam giác DCE (c.g.c) ⇒ EN = ED ⇒ tam giác END cân E ⇒ EM vng góc MD ⇒ M thuộc đường trịn đường kính DE Bài 141 Cho tứ giác ABCD nội tiếp AB CD cắt Q BC, AD cắt P Gọi M, N trung điểm BD AC Chứng minh ∠PMQ + ∠PNQ = 180° LỜI GIẢI Theo giả thiết tứ giác ABCD nội tiếp đường tròn ⇒ ∠BAC = ∠BDC ⇒ tam giác AQC tam giác DQB đồng dạng(g.g) N, M trung điểm AC BD ⇒ tam giác QNC tam giác QMB đồng dạng (c.g.c) ⇒ ∠QNC = ∠QMB Tương tự ∠PNC = ∠PMD ⇒ ∠PMQ + ∠PNQ = ∠PMQ + ∠PNC + ∠CNQ = ∠PMQ + ∠PMQ + ∠QMB = ∠BMD = 180° Bài 142 Cho tam giác ABC, trực tâm H Đường cao BD CE cắt đường tròn ngoại tiếp tam giác ABC M N Đường thẳng MN cắt BC P Gọi (O) đường tròn ngoại tiếp tam giác BHC Chứng minh AO vng góc với PH LỜI GIẢI Theo giả thiết H trực tâm tam giác ⇒ AH = AM = AN ⇒ A tâm Liên hệ tài liệu word toán SĐT (zalo): 039.373.2038 98 Website: tailieumontoan.com đường tròn qua M, N, H Giả sử PH cắt đường tròn (O) K Theo hệ thức đường tròn ta có : PH PK = PB.PC Mà PN PM = PB.PC ⇒ PH PK = PN PM ⇒ N , H , K , M thuộc đường tròn tâm A ⇒ AO vng góc HK hay AO vng góc với PH Bài 143.Cho tam giác nhọn ABC Đường cao BD, CE trực tâm H Đường tròn ngoại tiếp tam giác BCH cắt đường tròn qua D, E trung điểm BC P Q Chứng minh DE, PH, MQ đồng qui đường tròn ngoại tiếp tam giác ABC LỜI GIẢI Gọi M trung điểm BC, I giao điểm ED PH Ta có ∠IDB = 180° − ∠EDB = 180° − ∠EDH = 180° − ∠EAH = 90° + ∠ABC , ∠IPB = ∠HPB = 180° − ∠HCB = 90° + ∠ACB ⇒ ∠IDB = ∠IPB ⇒ B, P, D, I nằm đường tròn hay tứ giác BPDI nội tiếp ⇒ ∠PBD = ∠PID Theo giả thiết, đường tròn ngoại tiếp tam giác BCH cắt đường tròn qua D, E M P ⇒ tứ giác BPHC nội tiếp ⇒ ∠PCE = ∠PCH = ∠PBH = ∠PBD = ∠PID = ∠PIE ⇒ tứ giác EICP nội tiếp Gọi K chân đường vng góc kẻ từ A xuống BC.Theo định lí Euler đường trịn điểm, ta có: E, D, M, K nẳm đường tròn ⇒ tứ giác BCDE, PCIE, ACKE , ABKD nội tiếp ⇒ ∠PIC = ∠PEC , ∠PEK = ∠PDK ⇒ ∠BIC = ∠PIC − ∠PIB = ∠PEC − ∠PDB = ∠PEK + KEC − ( ∠PDK − ∠BDK ) = ∠PEK + ∠KEC − ∠PDK + ∠BDK ⇒ ∠KEC + ∠BDK = ∠KAC + ∠BAK = ∠BAC ⇒ tứ giác BAIC nội tiếp hay I thuộc đường tròn ngoại tiếp tam giác ABC Vậy DE, PH, MQ đồng qui đường tròn ngoại tiếp tam giác ABC Bài 144 Cho tam giác vng ABC vng A Đường phân giác góc C cắt AB D Đường tròn qua B, D Liên hệ tài liệu word toán SĐT (zalo): 039.373.2038 99 Website: tailieumontoan.com , C cắt cạnh AC E Chứng minh AE + AC = BC LỜI GIẢI Từ D hạ DH vng góc với BC Theo giả thiết , ∠ACD = ∠DCB ⇒ DA = DH AC = CH Tam giác ADE tam giác HDB tam giác vng, có DA = DH ⇒ hai tam giác ⇒ AE = BH ⇒ AE + AC = BH + CH = BC Bài 145 Cho tam giác ABC.Trên đường kéo dài đường cao BD CE lấy điểm P Q cho ∠PAQ = 90° Gọi H hình chiếu A PQ Chứng minh BH vng góc HC LỜI GIẢI Theo giả thiết, BD vng góc AC CE vng góc với AB ⇒ B, E , D, C nằm đường trịn đường kính BC H hình chiếu A PQ ⇒ ∠AHP = ∠AHQ = 90° ⇒ A, H , E , P nằm đường trịn đường kính AP A, H, D, Q nằm đường trịn đường kính AQ ⇒ ∠PAE = ∠PHE , ∠QAD = ∠QHD ⇒ ∠DHE = 180° − ∠PHE − ∠QHD = 180° − ∠PAE − ∠QAD = 180° − ( 90° − ∠BAC )= 90° + ∠BAC= 90° + ( 90° − ∠ACE ) = 180° − ∠ACE ⇒ tứ giác EHDC nội tiếp ⇒ H thuộc đường tròn ( BEDC ) ⇒ ∠BHC= 90° Bài 146 Cho tam giác ABC D E cạnh AB AC thỏa mãn tâm đường tròn nội tiếp tam giác ABC nằm đường tròn ngoại tiếp tam giác AND Chứng minh BD + CE = BC LỜI GIẢI Gọi I tâm đường tròn nội tiếp tam giác ABC M, N, P tiếp điểm với cạnh BC, CA, AB ⇒= BP BM , CM = CN ⇒ BP + CN = BM + CM = BC Theo giả thiết A, D, I, E nằm đường tròn ⇒ ∠AEI = ∠IDB ⇒ tam giác INE IPD hai tam giác vuông ⇒ NE = PD Liên hệ tài liệu word toán SĐT (zalo): 039.373.2038 100 Website: tailieumontoan.com ⇒ BD + CE =( BP + PN ) + ( CN − NE ) =BP + CN =BC Bài 147 Cho tam giác ABC D AB, E AC cho BD = CE Gọi M, N trung điểm BC DE Chứng minh đường phân giác góc ∠BAC song song với MN LỜI GIẢI Theo giả thiết, BD = CE ⇒ đường tròn ngoại tiếp tam giác ADE qua điểm P cung cung BAC Kẻ đường phân giác góc ∠BAC cắt đường tròn ngoại tiếp tam giác ABC I, cắt đường tròn ngoại tiếp tam giác ADE Q ⇒ IB = IC ⇒ P, M , I thẳng hàng PI đường kính đường trịn ngoại tiếp tam giác ABC Mà QD = QE ⇒ ∠DPQ = ∠QPE ⇒ P, N ,Q thẳng hàng PQ đường kính đường trịn ngoại tiếp tam giác ADE ⇒ tam giác PDQ tam giác PBI hai tam giác vng đồng dạng (vì DN vng góc với PQ PI vng góc với BC) ⇒ PN PM = NQ MI Theo định lí Thales ta có MN song song với IQ hay đường phân giác góc ∠BAC song song với MN Bài 148 Cho tam giác ABC nội tiếp đường tròn (O) Tiếp tuyến B C với đường tròn (O) cắt P Gọi D E hình chiếu P AB AC Chứng minh trực tâm tam giác ADE nằm BC LỜI GIẢI Gọi H trung điểm BC Theo giả thiết, PB, PC tiếp tuyến đường tròn (O) ⇒ PB = PC ⇒ tam giác PBC cân P ⇒ PH vng góc với BC Theo giả thiết PE vng góc với AC ⇒ tứ giác HCEP nội tiếp ⇒ ∠HEP = ∠HCP PC tiếp tuyến đường tròn (O) ⇒ ∠BAC = ∠HCP ⇒ ∠BAC = ∠HEP ⇒ ∠HEA + ∠BAC= ( 90° − ∠HEP ) + ∠BAC= 90° ⇒ EH vuông góc với AB Tương tự DH vng góc với AC ⇒ H trực tâm tam giác ADE Bài 149 Cho tam giác ABC vuông C Gọi G trọng tâm tam giác Trên AG lấy điểm P cho Liên hệ tài liệu word toán SĐT (zalo): 039.373.2038 101 Website: tailieumontoan.com ∠CPA = ∠CAB , BG lấy điểm Q cho ∠CQB = ∠ABC Chứng minh đường tròn ngoại tiếp tam giác BPG AQG gặp AB (Canada 2012) LỜI GIẢI * Cách Gọi M trung điểm AB ∠BCA = 90° ⇒ MB = MA = MC ⇒ ∠ACM = ∠BAC ⇒ ∠ACG = ∠ACM = ∠BAC = ∠CPA Tam giác APC tam giác ACG đồng dạng (g.g) ⇒ AC AP = ⇒ AC = AP AG AG AC Gọi H hình chiếu C AB Theo tính chất tam giác vng ta có AC = AH AB ⇒ AP AG = AH AB ⇒ tứ giác BPGH nội tiếp ⇒ tứ giác AQGH nội tiếp ⇒ đường tròn ngoại tiếp tam giác BPG AQG gặp AB * Cách Trên AB lấy điểm D cho CA = CD ⇒ tam giác CAD cân C ⇒ ∠CDA = ∠A = ∠CPA ⇒ tứ giác CPDA nội tiếp ⇒ ∠GPD = ∠APD = ∠ACD = 180° − 2∠CAD M trung điểm AB ⇒ MC = MB ⇒ ∠CMD = ∠CMB = 2∠A = 180° − ∠GPD ⇒ tứ giác GPDM nội tiếp ⇒ AM AD = AG AP CH vng góc với AD ⇒ HA = HD ⇒ AM AD = AB.2 AH =AH AB =AG AP ⇒ tứ giác BPGH nội tiếp Bài 150 Cho tam giác nhọn ABC có trực tâm H M, N chân đường cao kẻ từ đỉnh B C D điểm cạnh BC Gọi ω1 đường tròn qua C, D, M ω2 đường tròn qua B, D, N DQ đường kính đường trịn ω1 , DP đường kính đường trịn ω2 Liên hệ tài liệu word toán SĐT (zalo): 039.373.2038 102 Website: tailieumontoan.com Chứng minh P, Q, H thẳng hàng (IMO 2013) LỜI GIẢI Đường tròn ω1 qua ( D, C , M ) đường tròn ω2 qua ( D, B, N ) cắt D K Đường cao AH cắt cạnh BC E Tứ giác BDKN nội tiếp ⇒ ∠B + ∠DKN = 180° Tương tự ∠C + ∠DKM= 180° ⇒ ∠DKN + ∠DKM = 360° − ∠B − ∠C = 180° + ∠A Mặt khác ∠NKM = 360° − ∠DKN − ∠DKM ⇒ ∠NKM = 180° − ∠A A 180° ⇒ ∠NKM + ∠= ⇒ ANKM nội tiếp BM vng góc AC, CN vng góc AB ⇒ ANHM nội tiếp ⇒ A, N, H, K, M nằm đường tròn ⇒ ∠AKN = ∠AHN , ∠AHN = ∠B ⇒ ∠AKN = ∠B ⇒ ∠DKN + ∠AKN = 180° ⇒ A, K , D thẳng hàng Các tứ giác CMKD, BNKD, BNHE nội tiếp AC AK AD, AK AD AN AB, AM AC AN AB, AH AE AN AB ⇒ AM = = = = ⇒ AK AD = AH AE ⇒ tứ giác EHKD nội tiếp Mà AE vng góc BC ⇒ ∠HKD= 90° ⇒ P, H , Q thẳng hàng Bài 151 Cho tam giác ABC, A1 , B1 , C1 tiếp điểm đường tròn bàng tiếp với cạnh BC, CA, AB Chứng minh tâm đường tròn ngoại tiếp tam giác A1 B1C1 nằm đường tròn ngoại tiếp tam giác ABC ABC tam giác vng (IMO 2013) LỜI GIẢI Gọi O tâm đường tròn ngoại tiếp tam giác A1 B1C1 Theo giả thiết, O nằm đường tròn ngoại tiếp tam giác ABC Liên hệ tài liệu word toán SĐT (zalo): 039.373.2038 103 Website: tailieumontoan.com Theo giả thiết, A1 , B1 , C1 tiếp điểm đường tròn bàng tiếp ⇒ BC = CB1 , AB = BA1 , CA = AC1 1 (bạn đọc tự chứng minh) Giả sử AC ≥ AB Gọi I điểm ⇒ IB = cung BAC IC ⇒ ∠IBA = ∠ABC − ∠IBC = ∠B − 90° − ∠A = 90° − ∠A − ∠C = ∠ICB − ∠ACB = ∠ICA (do ∠A =∠BIC ) ⇒ tam giác IBC1 ICB1 (c.g.c) ⇒ IC1 = IB1 ⇒ I ≡ O Trên OB lấy điểm M OC kéo dài lấy điểm N cho BM = CN = OA ⇒ ∠OBA1 = ∠OBC = ∠OAC , AB1 = BA1 Tứ giác OABC nội tiếp ⇒ ∠NCA1 = ∠OAB Mà CA1 = AC1 nên tam giác CNA1 , AOC1 Mà tam giác BMA1 , AOB1 ⇒ NA1 = OC1 = OA1 = OB1 = MA1 ⇒ ∠CA1 N = ∠AC1O = 180° − ∠BC1O = 180° − ∠OB1C = ∠AB1O = ∠BA1 M ⇒ M , A1 , N thẳng hàng ⇒ NA1= OA1= MA1 ⇒ ∠MON= 90° ⇒ ∠BAC= 90° ⇒ tam giác ABC vuông A Bài 152 Cho tam giác ABC Đường phân giác AD cắt đường tròn ngoại tiếp tam giác ABC E M trung điểm BC EM cắt đường tròn ngoại tiếp tam giác ABC K Đường thẳng KD cắt đường tròn ngoại tiếp tam giác ABC F Chứng minh góc ∠BAF = ∠CAM LỜI GIẢI Liên hệ tài liệu word toán SĐT (zalo): 039.373.2038 104 Website: tailieumontoan.com Theo AD phân giác góc ∠BAC ⇒ EB = EC M trung điểm BC ⇒ EM vng góc với BC ⇒ EK đường kính đường tròn ngoại tiếp tam giác ABC ⇒ ∠KAE= 90° ∠KFE = 90° ⇒ tứ giác AKMD nội tiếp ⇒ ∠MKD = ∠MAD = ∠MAE Mặt khác, ∠FKE = ∠FAE ⇒ ∠MAE = ∠FAE ⇒ ∠BAF = ∠BAE − ∠FAE = ∠EAC − ∠EAM = ∠MAC ⇒ ∠BAF = ∠CAM Bài 153 Cho đường tròn (O) dây cung BC cố định Điểm A chuyển động đường tròn (O) cho tam giác ABC nhọn Tiếp tuyến B C cắt P Gọi D E hình chiếu P AB AC Chứng minh DE qua điểm cố định A thay đổi LỜI GIẢI Gọi M trung điểm BC ⇒ PM vng góc với BC PD vng góc với AB PE vng góc với AC ⇒ tứ giác BMPD MCEP nội tiếp ∠ACB ⇒ ∠DMP = ∠DBP ∠MPE = PB tiếp tuyến đường (O) ⇒ ∠CAB = ∠CBP Mặt khác ∠DBP = 180° − ∠CBP − ∠ABC = 180° − ∠BAC − ∠ABC = ∠ACB ⇒ ∠DMP = ∠MPE ⇒ DM song song với PE Tương tự DP song song với ME ⇒ tứ giác DMEP hình bình hành ⇒ MP DE cắt trung điểm đường Mà MP cố định ⇒ I cố định Vậy DE qua trung điểm MP Bài 154 Cho đường trịn (O) bán kính R điểm M ngồi đường trịn.Kẻ tiếp tuyến MA, MB C điểm thuộc đoạn AB I K trung điểm MA MC AK cắt đường tròn (O) điểm thứ hai D 1) Chứng minh KO − KM = R2 2) Chứng minh tứ giác BCDM nội tiếp 3) Gọi E giao thứ hai MD với đường tròn (O), N trung điểm KE KE cắt đường tròn (O) điểm Liên hệ tài liệu word toán SĐT (zalo): 039.373.2038 105 Website: tailieumontoan.com thứ hai F Chứng minh I, A, N, F nằm đường tròn (Đề thi Chuyên Đại học SP HN 2017) LỜI GIẢI 1) Bổ đề : Cho đường tròn (O) Kẻ tiếp tuyến MA, MB Gọi I, J trung điểm MA, MB N điểm đoạn IJ Kẻ tiếp tuyến NC Chứng minh NM = NC AB vng góc OM, IJ đường trung bình tam giác MAB ⇒ IJ song song với AB ⇒ NM = ND Tam giác OAM vuông ⇒ OC = OA2 = OD.OM ⇒ OC tiếp tuyến đường trịn (MDC) NM NC vng góc với OC ⇒ NC = ⇒ NO = NC + OC Quay trở lại toán, K trung điểm MC nên K vai trò N bổ đề ⇒ KO − KM = R2 2) Từ chứng minh ⇒ KC = KM = KO − R = KD.KA ⇒ KC KA = KD KC ⇒ tam giác KCD tam giác KAC đồng dạng ⇒ ∠KCD = ∠KAC ⇒ ∠MCD = ∠BAD = ∠DBM ⇒ tứ giác DCBM nội tiếp 3) Từ ∠AEM = ∠MAK = ∠EMK ⇒ AE song song với KM Gọi P trung điểm AK ⇒ PN song song với DE Theo hệ thức lượng đường trịn, ta có: = KF KE KD= KA, KE 2= KN , KD KP ⇒ KF KN = KP.KA ⇒ tứ giác ANFP nội tiếp ⇒ ∠PAF = ∠PNF = ∠MEK = ∠FMK ⇒ ∠KAF = ∠KMF ⇒ tứ giác MKFA nội tiếp ⇒ ∠AFN = ∠AMK = ∠AIN ⇒ tứ giác IANF nội tiếp, hay bốn điểm I, A, N, F nằm đường tròn Liên hệ tài liệu word toán SĐT (zalo): 039.373.2038 106 Website: tailieumontoan.com Bài 155 Cho tam giác ABC ( ∠B > ∠C ) Trên AC lấy điểm P Q cho ∠PBA = ∠QBA = ∠BCA Q nằm đoạn AC D điểm BQ cho PB = PD AD cắt đường tròn ngoại tiếp tam giác ABC E Chứng minh QB = QE (IMO Shortlist 2013) LỜI GIẢI Theo giả thiết , ∠PBA = ∠QBA = ∠BCA ⇒ tam giác PBA tam giác PCB đồng dạng (g.g) ⇒ PB PA = ⇒ PB = PA.PC PC PB ⇒ PB tiếp tuyến đường tròn ngoại tiếp tam giác ABC (Bạn đọc tự chứng minh) Tương tự, PD tiếp tuyến đường tròn qua A, D, C ⇒ ∠PDA = ∠DCA Mặt khác, tam giác ABQ tam giác ACB có ∠QBA = ∠BCA ⇒ hai tam giác đồng dạng ( g g ) ⇒ ∠AQB = ∠B = ∠AEC ⇒ tứ giác DQCE nội tiếp đường tròn ⇒ ∠DEQ = ∠DCQ ⇒ ∠BEQ = ∠BEA + ∠AEQ = ∠AEQ + ∠DCQ = ∠C + ∠DCA, ∠QDE = ∠ADB = ∠ADP + ∠PDB = ∠DCA + ∠PBD = ∠DCA + 2∠C (1) Mặt khác, ∠QDE = 180° − ∠DQE − ∠DEQ ( ) từ (1) (2) ta thu ⇒ ∠DCA + 2∠C = 180° − ∠DQE − ∠DEQ ⇒ ∠DQE= 180° − 2∠C − ∠DCA − ∠DEQ= 180° − 2∠C − 2∠DCA ⇒ ∠QBE = 180° − ∠BQE − ∠QEB = 180° − (180° − 2∠C − 2∠DCA ) − ( ∠DCA + ∠C ) = ∠DCA + ∠C ⇒ ∠BEQ = ∠QBE ⇒ QB = QE Bài 156 Cho tam giác ABC vuông A, đường cao AH M ∈ AH , P ∈ BM cho= CP CA, Q ∈ CM cho BQ = BA CP cắt BQ E Chứng minh EP = EQ (IMO 2012) LỜI GIẢI Liên hệ tài liệu word toán SĐT (zalo): 039.373.2038 107 Website: tailieumontoan.com * Cách BP CQ cắt đường tròn ngoại tiếp tam giác ABC G D Đường thẳng BD CG cắt I , ∠A = 90° ⇒ BC đường kính đường trịn ngoại tiếp tam giác ABC ⇒ ∠BDC = ∠BGC = 90° ⇒ M trực tâm △IBC ⇒ I nằm đường thẳng AH Ta có BQ = BA2 = BH BC ⇒ BQ BC = BH BQ ⇒ tam giác BQH đồng dạng với tam giác BCQ (c.g.c) ⇒ ∠BQH = ∠BCQ Mặt khác ∠BCD = ∠BIH ⇒ ∠BIH = ∠BQH ⇒ B, Q, I , H nằm đường trịn Mà IH vng góc với BC ⇒ BI đường kính đường trịn ngoại tiếp tứ giác BQIH 2 BI − BQ BI − BA BI − BH BC ⇒ BQI = 90° ⇒ QI= = = Mặt khác, BH BC = BD.BI ⇒ QI = BI − BH BC = BI − BD.BI = BI DI Tương tự, PI = CI GI Ta lại có: ID.IB= IG.IC ⇒ QI= PI Hai tam giác EQI EPI vuông, có QI = PI cạnh huyền EI chung ⇒ hai tam giác ⇒ EP = EQ * Cách 2 Ta có BQ = BA = BH BC ⇒ BQ tiếp tuyến đường tròn (QHC) ⇒ ∠BQH = ∠QCB Trên AH lấy điểm I cho HM HI = HC.HB ⇒ HB HM = HI HC Mà IH vng góc với BC ⇒ hai tam giác IBH CMH đồng dạng (c.g.c) ⇒ ∠BIH = ∠MCH ⇒ ∠BQH = ∠BIH Liên hệ tài liệu word toán SĐT (zalo): 039.373.2038 108 Website: tailieumontoan.com ⇒ tứ giác QBIH nội tiếp ⇒ BQ vng góc QI ⇒ QI = BI − BQ = BI − BA2 2 Từ CP = CA = CH CB Tương tự CP ⊥ IP ⇒ IP = CI − CP = CI − CA2 Mà AH vng góc với BC ⇒ BI − BA2 = CI − CA2 ⇒ ( BI − BH ) − ( BA2 + BH ) = ( CI − CH ) − ( CA2 − CH ) = EPI ⇒ tam giác BQI tam giác EPI ⇒ EP = ⇒ QI = PI , BQI EQ Bài 157 Cho tam giác cân ABC ( AB = AC , ∠A < 60° ) Trên cạnh AC lấy điểm D cho ∠DBC = ∠A E giao điểm đường trung trực BD đường thẳng qua A song song với BC Lấy điểm P tia đối tia AC cho PA = AC Chứng minh đường thẳng qua E vng góc với AC, đường thẳng qua P vng góc với AB đường thẳng BD đồng qui LỜI GIẢI Theo giả thiết, ∠DBC =∠A ⇒ tứ giác EACB hình bình hành => AC song song với BE Gọi M trung điểm AP ⇒ AM = AC = AB => MBC tam giác vuông B => MB vng góc AE => tứ giác MABE hình thoi ⇒ ME = MP = MA ⇒ PE vng góc với EA Giả sử đường thẳng qua P vng góc với AB cắt BD Q ⇒ ∠QPE = ∠ABC ⇒ ∠QPE = ∠BDC = ∠BDE => tứ giác PQDE nội tiếp ⇒ ∠PEQ = ∠PDQ Mặt khác ∠PDQ = ∠PAE , ∠PAE + ∠APE = 90° ⇒ ∠PEQ + ∠APE= 90° => PA vuông góc EQ => AC vng góc EQ Bài 158 Cho tam giác ABC nội tiếp đường tâm (O) Gọi D, E, F điểm cạnh BC, CA, AB cho DE vng góc với OC, DF vng góc với OB Gọi I tâm đường tròn ngoại tiếp tam giác AEF Chứng minh ID vng góc với BC LỜI GIẢI Liên hệ tài liệu word toán SĐT (zalo): 039.373.2038 109 Website: tailieumontoan.com Từ giả thiết ⇒ ∠OCB = (180° − ∠BOC ) = 90° − ∠BOC= 90° − ∠A Mà OC vng góc DE ⇒ ∠EDC= 90° − ∠OCB ⇒ ∠EDC = ∠A Tương tự, ∠BDF = ∠A ⇒ ∠EDF = 180° − 2∠A I tâm đường tròn ngoại tiếp tam giác AEF ∠EIF = 2∠A ⇒ ∠EDF = 180° − ∠EIF ⇒ ∠EDF + ∠EIF = 180° => I, E, D, F nằm đường tròn ⇒ ∠IEF = ∠IDF ⇒ ∠IEF= (180° − ∠EIF )= 90° − ∠A ⇒ ∠IDF + ∠BDF= 90° => ID vng góc với BC Bài 159 Cho tam giác ABC khơng cân nội tiếp đường trịn (O) P điểm nằm tam giác ABC không trùng O Đường thẳng AP cắt (O) D Gọi DE, AF đường kính (O) Giả sử EP, FP cắt (O) G I Đường thẳng AI cắt DG K Gọi H hình chiếu K lên đường thẳng OP OP cắt EF M Chứng minh A, K, H, D, M thuộc đường tròn LỜI GIẢI Theo giả thiết DE, AF đường kính đường trịn (O), KH ⊥ OP ⇒ ∠KHP = ∠KIP = ∠KGP = 90° => K, H, P, G, I nằm đường trịn đường kính KP ⇒ ∠IHP = ∠IGP − ∠IGE = ∠IDE ⇒ ∠IHO = ∠IDO ⇒ tứ giác IHDO nội tiếp đường tròn ⇒ ∠IHD + ∠IOD = 180° Mặt khác ∠OHI = ∠IGE = ∠IDE (1) => HDOI tứ giác nội tiếp ⇒ ∠DHO = ∠DIO = ∠IDO (2) Từ (1) (2) ⇒ ∠OHI = ∠OHD ⇒ ∠OHI = ∠PKI = 90° − ∠KAD ⇒ ∠OHD = 90° − ∠HKD ⇒ ∠KHD = ∠KAD hay K, H, A, D thuộc đường trịn Ta có AO = OF AP song song EF nên tứ giác PAMF hình hình hành ⇒ ∠DAM = ∠PFM = ∠IGE = ∠IHP = ∠MHD => A, H, D, K, M nằm đường tròn Liên hệ tài liệu word toán SĐT (zalo): 039.373.2038 110 Website: tailieumontoan.com Bài 160 Cho tam giác ABC trực tâm H Gọi P điểm nằm đường tròn ngoại tiếp tam giác ABC E chân đường cao hạ từ B xuống cạnh AC Dựng hình bình hành PAQB PADC QA cắt HD F Chứng minh EF song song với AP LỜI GIẢI Kéo dài AH cắt cạnh BC K ⇒ AK ⊥ BC ⇒ tức giác HKCE nội tiếp ⇒ ∠EHK + ∠ACB = 180° P đường tròn (O) ⇒ ∠APB = ∠ACB Mặt khác, theo giả thiết, AQBP hình bình hành ⇒ ∠AQB = ∠APB ⇒ ∠AQP + ∠AHB = ∠AQB + ∠EHK = 180° => tứ giác AQGH nội tiếp Gọi I giao điểm BH AP ⇒ ∠KAF = ∠HBQ = ∠AIE Tương tự tứ giác ADCH nội tiếp ⇒ ∠AHD = ∠ACD = ∠CAP ⇒ ∠AHD + ∠HAF = ∠CAP + ∠AIE = 90° => tứ giác AEFH nội tiếp ⇒ ∠HFE = ∠HAE = ∠HAC = ∠HDC => EF song song với CD => EF song song với AP Bài 161 Cho tam giác ABC nội tiếp đường tròn (O) Tiếp tuyến (O) B C cắt M, đường thẳng AM cắt (O) N Gọi P, Q giao điểm đường thẳng vng góc NC C với (O) BN AP cắt BC E Chứng minh Q, M, E thẳng hàng LỜI GIẢI Gọi D giao điểm MO PQ, H giao điểm PO MQ, I giao điểm OM với cạnh BC DO vng góc với BC ⇒ ∠BDM = ∠CDM = 90° − ∠PCB CN vuông góc với PQ ⇒ ∠BCN= 90° − ∠PCB ) Mà ∠BCN = ∠BAN (chắn cung BN ⇒ ∠BDM = ∠BAM ⇒ tứ giác AMBD nội tiếp Mà OM vng góc BC ⇒ ∠BMD = ∠CMD ⇒ ∠CMD= 90° − ∠BCM= 90° − ∠BAC ⇒ ∠BQC= ∠NQC= 90° − ∠CNQ= 90° − ∠BAC (do tứ giác ABNC nội tiếp) Liên hệ tài liệu word toán SĐT (zalo): 039.373.2038 111 Website: tailieumontoan.com ⇒ ∠BMD = ∠BQD => tứ giác BMQD nội tiếp ⇒ MB = MC = MQ ⇒ ∠MQC = ∠MCQ = 90° − ∠NCM = 90° − ∠NPC = ∠PNC => tứ giác NCQH nội tiếp Mà NC vng góc PQ => NH vng góc MQ Giả sử PN cắt ME K MB tiếp tuyến (O) ⇒ MB = MN MA Mặt khác MB = MI MO ⇒ MI MO = MN MA ⇒ tứ giác AOIN nội tiếp ⇒ ∠OIA = ∠ONA = ∠KNM ⇒ tứ giác AEKN nội tiếp => PN vuông góc ME => M, Q, E thẳng hàng Bài 162 Cho tam giác nhọn ABC nội tiếp đường tròn (O), H trực tâm Đường trung trực AH cắt AC, AB tương ứng P Q Chứng minh OA phân giác góc ∠POQ (Sharygin 2017) LỜI GIẢI Kéo dài AO cắt đường tròn (O) D ⇒ ∠ABD= 90° Kéo dài AH cắt BC E Vì ∠BDA = ∠ACB ⇒ ∠HAC = ∠OAB = 90° − ∠ACB ⇒ ∠PAO = ∠PAH Đường trung trực AH cắt AC P ⇒ PA = PH => tam giác PAH tam giác OAB đồng dạng ⇒ AH PA = AB OA => tam giác AOP tam giác ABH đồng dạng (c.g.c) ⇒ ∠POA = ∠HBA = 90 − ∠ABC Tương tự, ∠QOA = ∠HCA = 90° − ∠ABC ⇒ ∠QOA = ∠AOP = 90° − ∠ABC Bài 163 Cho đa giác ABCDEF nội tiếp đường tròn (O) thỏa mãn AB = BC = CD = DE K điểm AE cho ∠BKC = ∠KFE , ∠CKD = ∠KFA Chứng minh KC = KF (China TST 2016) LỜI GIẢI Liên hệ tài liệu word toán SĐT (zalo): 039.373.2038 112 Website: tailieumontoan.com Theo giả thiết AB = DE ⇒ BD // AE Cũng theo giả thiết ∠BKC = ∠KFE , ∠CKD = ∠KFA ⇒ ∠BKD = ∠BKC + ∠CKD = ∠KFE + ∠KFA = ∠AFE Theo tính chất cung chứa góc BD song song với AE, đồng thời AB cắt CD S => bán kính đường tròn ngoại tiếp tam giác BDK AEF tỉ lệ với theo tỉ số BD AE Giả sử SK cắt đường tròn ngoại tiếp tam giác AFE H => BK song song AH DK song song HE ⇒ ∠HBK = ∠BHA = ∠DBC ∠HDK = ∠DHE = ∠CBD ⇒ ∠KHA = ∠DKC , ∠KHE = ∠BKC ⇒ H ≡ F O tâm đường tròn ngoại tiếp đa giác ABCDEF ⇒ ∠BOD = ∠AFE = ∠BKD Mà OB = OD C, F đối xứng qua OK ⇒ KC = KF Bài 164 Cho tam giác ABC Đường tròn tâm (O) qua B C cắt AC D AB E, BD lấy điểm P cho BP = AC Trên CE lấy điểm Q cho CQ = AB Chứng minh tâm đường tròn ngoại tiếp tam giác APQ nằm đường tròn tâm (O) LỜI GIẢI Đường tròn qua B, C cắt AC D AB E => tứ giác BEDC nội tiếp ⇒ ∠EBD = ∠ECD Cũng theo giả thiết BP = AC CQ = AB => tam giác BAP tam giác CQA (c.g.c) ⇒ AP = AQ, ∠BAP = ∠CQA, ∠APB = ∠QAC ⇒ ∠AEQ = 180° − ∠BAC − ∠ACE = 180° − ∠BAC − ∠ABD = ∠ADB Mặt khác ∠ADB = ∠APD + ∠PAD = ∠QAC + ∠PAC = ∠QAP Gọi I tâm đường tròn ngoại tiếp tam giác APQ ⇒ ∠AIQ = 2∠APQ = 180° − ∠QAP ⇒ ∠AIQ = 180° − ∠AEQ => tứ giác AIQE nội tiếp đường tròn Tương tự, tứ giác AIDP nội tiếp đường trịn Liên hệ tài liệu word tốn SĐT (zalo): 039.373.2038 113 Website: tailieumontoan.com ⇒ ∠EID = 360° − ∠AIE − ∠AID = 360° − ∠AQE − ∠AOD = ∠AQC + ∠APB = ∠AQC + ∠QAC = 180° − ∠DCE ⇒ ∠EID + ∠DCE = 180° ⇒ tứ giác EIDC nội tiếp => I thuộc đường tròn (BEDC) Bài 165 Cho tam giác ABC Gọi M, N trung điểm AB AC E điểm BC khơng chứa A Đường trịn ngoại tiếp tam giác AME cắt đường trung trực AC P Đường tròn ngoại tiếp tam giác AME cắt đường trung trực AB Q, P Q nằm tam giác ABC Đường thẳng PQ cắt MN K Chứng minh KA = KE (TST France 2014, Shortlist 2013) LỜI GIẢI NP NQ trung trực AC AB => O tâm đường tròn ngoại tiếp tam giác ABC Gọi J tâm đường trịn ngoại tiếp tam giác AME => OJ vng góc AE => OJ trung trực AE Theo giả thiết E điểm cung BC => OE vng góc BC ON vng góc AC ⇒ ∠POE = ∠ACB Ta có ∠ACB = ∠AOM ⇒ ∠AOM = ∠EOP, OA = OE , OM = OP => tam giác OAM tam giác OEP đối xứng qua trung trực AE => hai tam giác OAM OEP Mặt khác MA = MB => OM vng góc AB => OP vng góc PE Liên hệ tài liệu word tốn SĐT (zalo): 039.373.2038 114 Website: tailieumontoan.com Ta có OE vng góc PE AM = EP => MP song song AE Tương tự NQ song song AE ⇒ ∠MAE = ∠PEA, ∠NAE = ∠QEA ⇒ ∠MAN = ∠PEQ => hai tam giác MAN PEQ (c.g.c) ⇒ MN = PQ => tứ giác MNQP hình thang cân => tam giác KMP cân K ⇒ KM = KP => tam giác AMK tam giác EPK (c.g.c) ⇒ KA = KP Bài 166 Cho tam giác ABC D cạnh BC cho đường tròn nội tiếp tam giác ABD tam giác ADC tiếp xúc AD Gọi I, J tâm đường tròn nội tiếp tam giác ABD tam giác ADC O tâm đường tròn ngoại tiếp tam giác AIJ Chứng minh BO đường phân giác góc ∠BAC LỜI GIẢI Gọi E tiếp điểm đường tròn nội tiếp tam giác ABD tam giác ADC AD Q, N, M, P tiếp điểm tam giác ABD tam giác ADC cạnh BA, CA, AB (hình vẽ) Theo tính chất đường trịn nội tiếp ta có DQ = DE = DN , DQ = AD + DC − AC BD + DA − AB DN = 2 ⇒ AB + CD = AC + BD Từ suy cách dựng điểm D Do I, J tâm tâm đường tròn nội tiếp tam giác ABD ADC => ID vng góc với JD Theo giả thiết, AD vng góc với IJ ⇒ ∠JID= 90° − ∠EDI = 90° − ∠ADB Theo tính chất tâm đường trịn nội tiếp ⇒ ∠AID= 90° + ∠ABD 1 ⇒ ∠AIJ = ∠AID − ∠DIJ = 90° + ∠ABD − 90° − ∠ADB 2 = ( ∠B + ∠ADB ) Tương tự ∠AJI = 1 ( ∠C + ∠ADC ) = 90° + ( ∠C − ∠ADB ) 2 Liên hệ tài liệu word toán SĐT (zalo): 039.373.2038 115 Website: tailieumontoan.com Trong tam giác AIJ: ∠JAO= 90° − ( ∠B + ∠ADB ) , ∠JAC = ∠JAD = 90° − ∠AJI = ( ∠ADB − ∠C ) ⇒ ∠OAC = ∠OAJ + ∠JAC =90° − 1 ( ∠B + ∠ADB ) + ( ∠ADB − ∠C ) = ∠A 2 Bài 167 Cho tứ giác ABCD có ∠ABC = ∠CDA = 90° Điểm H chân đường vng góc hạ từ A xuống BD Các điểm S T tương ứng nằm AB, AD cho H nằm tam giác SCT ∠CHS − ∠CSB= 90°, ∠THC − ∠DTC= 90° Chứng minh đường thẳng BD tiếp xúc với đường tròn ngoại tiếp tam giác TSH (IMO 2014) LỜI GIẢI Phân tích cách dựng hình: Để xác định hai điểm S T không dễ, từ giả thiết ta tới cách xác định điểm S T hướng cho lời giải Từ ∠THC − ∠DTC= 90° ⇒ ∠THC = ∠DTC + 90° = 90° − ∠TCD + 90° = 180° − ∠TCD ⇒ ∠THC + ∠TCD = 180° Theo giả thiết ∠ADC = 90° Gọi F điểm đối xứng C qua D ⇒ TF = TC ⇒ ∠TFC = ∠TCD = ∠THC + ∠TFC = 180° ⇒ tứ giác THCF nội tiếp => T giao điểm đường tròn (HCF) AD Tương tự ∠CHS − ∠CSB= 90° Dựng điểm E đối xứng C qua AB => tứ giác HSEC nội tiếp => S giao điểm (HCE) AB Từ cách dựng ⇒ BE = BC DF = DC => BD // EF ⇒ AM ⊥ EF , AE = AF ⇒ ME = MF ⇒ HE = HF Kéo dài HC lấy ddeiemr P cho HP = HE => H tâm đường tròn ngoại tiếp tam giác EPF Tứ giác THCF nội tiếp ⇒ ∠THF = ∠TCF , ∠CHF = ∠CTF Mà tam giác TCF, HPF cân ⇒ ∠TCF = ∠HFP ⇒ ∠THF = ∠HFP ⇒ HT // PF (1) Liên hệ tài liệu word toán SĐT (zalo): 039.373.2038 116 Website: tailieumontoan.com Tương tự HS // PE ⇒ ∠SHT = ∠EPF (2) Từ (1) (2) ⇒ ∠EPF = ∠SHT (3) Ta có ∠THF = ∠CPF ∠HTF = ∠PCF => hai tam giác HTF PCF đồng dạng (g.g) ⇒ Tương tự HT HF = PC PF (4) HS HE (5) = PC PE Từ (4) (5) ⇒ HS PF = HT PE Kết hợp (3) => hai tam giác SHT EPF đồng dạng (g.g) ⇒ ∠TSH = ∠THD ⇒ ∠TSH = ∠EFP, ∠EFP = ∠THD => BD tiếp tuyến đường tròn ngoại tiếp tam giác HST Bài 168 Cho tam giác ABC nội tiếp đường tròn (O) Đường tròn tâm J tiếp xúc với cạnh AC E AB F, đồng thời tiếp xúc với đường tròn (O) P AP cắt EF D Chứng minh tam giác BDF tam giác CDE đồng dạng Trước giải chứng minh Bài toán sau Bài toán Cho tam giác ABC nội tiếp đường tròn (O), đường tròn (J) tiếp xúc với đường tròn (O) P, đồng thời tiếp xúc với cacnhj AC, AB E F a) Chứng minh trung điểm EF tâm đường tròn nội tiếp tam giác ABC b) Kéo dài PE, PF cắt đường tròn (O) M N Chứng minh MN song song EF (Đường tâm (J) có tên đường tròn Mixtilinear) Chứng minh a) Theo giả thiết đường tròn tâm (J) tiếp xúc với cạnh AC, AB E F tiếp xúc với đường tròn ngoại tiếp tam giác ABC P, O tâm đường tròn ngoại tiếp tam giác ABC => O, J, P thẳng hàng Đường thẳng PE cắt đường tròn (O) M => tam giác OPM tam giác JPE tam giác cân ⇒ ∠OPM = ∠OMP = ∠JPE = ∠JEP ⇒ OM song song JE, E tiếp điểm (J) với cạnh AC => JE vng góc với AC => OM vng góc AC => M điểm cung AC => BM phân giác góc ∠ABC ⇒ ∠CPM = ∠ACM ⇒ tam giác CME, tam giác PMC đồng dạng (g.g) Liên hệ tài liệu word toán SĐT (zalo): 039.373.2038 117 Website: tailieumontoan.com ⇒ CM PM = ⇒ CM = PM ME ME MC (1) Đường thẳng BM cắt EF I, xy tiếp tuyến với (O) (J) P ⇒ ∠IBP = ∠MBP = ∠MPy ∠IFP = ∠EFP = ∠EPy = ∠MPy ⇒ ∠IBP = ∠IFP => tứ giác BFIP nội tiếp ⇒ ∠BFP = ∠BIP , AB tiếp xúc đường tròn (J) F ⇒ ∠BFP = ∠FEP ⇒ ∠BIP = ∠FEP ⇒ ∠MIP = 180° − ∠BIP = 180° − ∠FEP = ∠IEM => tam giác IMP tam giác EMI đồng dạng (g.g) ⇒ IM EM = ⇒ MI = PM ME MP MI (2) Từ (1) (2) ⇒ MI = MA = MC => I tâm đường tròn nội tiếp tam giác ABC Từ suy cách dựng đường trịn Mixtilinear: Qua I (I tâm đường tròn nội tiếp tam giác ABC) dựng đường thẳng vng góc với AC cắt AI J => J tâm đường tròn Mixtilinear có bán kính JE b) Tương tự N điểm cung AB hay CN phân giác góc ∠ACB , đường trịn (O) (J) tiếp xúc P => MN song song EF LỜI GIẢI Đường thằng PE, PF cắt đường tròn (O) M N AC AB tiếp xúc đường tròn Mixtilinear ứng với góc ∠A => EF // MN Ta có PF PI đối xứng qua đường phân giác góc ∠EPF (PD đường đối trung tam giác PFE) Áp dụng tính hất đường đối trung DF PF ⇒ = DE PE (1) Theo tính chất đường tròn ⇒ FA.FB = FN FP EA.EC = EM EP Chia hai vế cho nhau, với EA =FA ⇒ Mặt khác EF // MN ⇒ (2) FN PF = EM PE Thay vào đẳng thức (2) ⇒ Từ (1) (3) ⇒ FB FN FP = EC EM EP FB PF = EC PE (3) DF BF = DE CE Mà ∠BFD = ∠CED ⇒ hai tam giác BDF DEC đồng dạng (c.g.c) Liên hệ tài liệu word toán SĐT (zalo): 039.373.2038 118 Website: tailieumontoan.com Bài 169 Cho tam giác ABC nội tiếp đường tròn (O) Đường tròn Mixtilinear tiếp xúc với AB, AC đường tròn (O) E, F, P Gọi Q điểm cung BC khơng chứa A, PQ cắt EF M Đường thẳng MA cắt đường trịn (O) D Chứng minh DI vng góc với AM LỜI GIẢI Theo tính chất đường trịn mixtilinear tiếp xúc với AB, AC đường tròn tâm (O) E F, ta có trung điểm EF tâm đường tròn nội tiếp tam giác ABC Theo giả thiết Q điểm cung BC không chứa A => A, I, Q thẳng hàng PI cắt đường tròn ngoại tiếp tam giác K => K trung điểm cung BAC => KQ đường kính đường trịn ngoại tiếp tam giác ABC ⇒ ∠KAQ= 90° ⇒ KA vng góc với AI => KA song song EF D, A, K, P thuộc đường tròn (O) ⇒ ∠xAK = ∠DPK = ∠DPI Mà KA // EF ⇒ ∠xAK = ∠xMI ⇒ ∠DPI = ∠xMI = ∠DMI ⇒ tứ giác MDIP nội tiếp KQ đường kính đường trịn (O) ⇒ ∠KPQ= 90° ⇒ ∠IPM= 90° => MI đường kính đường trịn MDIP ⇒ ∠MDI= 90° => DI vng góc với MA Bài 170 Cho tam giác ABC nội tiếp đường tròn (O) Trên cạnh AC lấy điểm E Đường tròn tiếp xúc với AC BE N M, đồng thời tiếp xúc với đường (O) P Gọi I tâm đường tròn nội tiếp tam giác BEC Chứng minh PI phân giác góc BPC LỜI GIẢI Theo bổ đề Sawayama-Thebault, gọi J tâm đường tròn nội tiếp tam giác ABC => J nằm MN tam giác EMN cân E Đường phần giác góc ∠EBC cắt CJ I ⇒ ∠BIC= 90° + ∠BEC ⇒ ∠JIB= 180° − ∠BIC= 90° − ∠BEC 1 Ta có ∠EMN= 90° − ∠MEN= 90° − ∠BEC 2 Liên hệ tài liệu word toán SĐT (zalo): 039.373.2038 119 Website: tailieumontoan.com ⇒ ∠JIB = ∠EMN => tứ giác BMJI nội tiếp Mặt khác theo bổ đề Sawayama-Thebault tứ giác BMJP nội tiếp => điểm B, M, J, I, P nằm đường tròn => tứ giác BPIJ nội tiếp 1 ⇒ ∠BPI = 180° − ∠BJI = 180° − 90° + ∠BAC = 90° − ∠BAC 2 Tứ giác ABPC nội tiếp ⇒ ∠BPC = 180° − ∠BAC ⇒ ∠IPC = ∠BPC − ∠BPI 1 = 180° − ∠BAC − 90° − ∠BAC = 90° − ∠BAC 2 => PI phân giác góc BPC Bài 171 Cho tam giác ABC nội tiếp đường tròn (O) H hình chiếu A BC, D E thứ tự trung điểm AC, AB Đường tròn qua D, E đồng thời tiếp xúc với đường (O) P (P khác A) Gọi G trọng tâm tam giác ABC Chứng minh P, H, G thẳng hàng LỜI GIẢI Nếu AB = AC ⇒ điểm P nằm AH, G nằm AH => P, H, G thẳng hàng Xét AB ≠ AC Gọi M giao điểm DE tiếp tuyến với đường trịn (O) A Theo giả thiết ta có , EA EB DA DC = = ⇒ OD ⊥ AC , OE ⊥ AB ⇒ tứ giác ADOE nội tiếp AM tiếp tuyến đường tròn (O) ⇒ ∠ADE = ∠ACB = ∠MAB => hai tam giác MAE MDA đồng dạng (g.g) ⇒ MA ME = ⇒ MA2 = ME.MD MD MA => MA tiếp tuyến với đường tròn ngoại tiếp tứ giác ADOE Từ M kẻ tiếp tuyến MN với đường tròn (O) ( N ≠ A ) ⇒ MA2 = MN = ME.MD ⇒ MN ME = MD MN => hai tam giác MDN MNE đồng dạng (c.g.c) ⇒ ∠MNE = ∠MDN ⇒ MN tiếp tuyến đường tròn qua (DEN) ⇒ N ≡ P Liên hệ tài liệu word toán SĐT (zalo): 039.373.2038 120 Website: tailieumontoan.com H hình chiếu A BC => AH ⊥ DE ⇒ tam giác AMH có MD vừa trung tuyến đồng thời đường cao ⇒ MA = MH ⇒ MA = MH = MP Nối P với H cắt đường tròn (O) Q Xét tam giác HAQ: 180° − ∠HAQ = ∠AHQ + ∠AQH = (180° − ∠AHP ) + ∠MAP = ( ∠AMP + ∠AOP ) = ∠AMO + ∠AOM = 90° ⇒ 180° − ∠HAQ= 90° ⇒ ∠HAQ= 90° ⇒ AQ ⊥ AH ⇒ AQ // BC Gọi I trung điểm BC Đường tròn qua I, D, E theo tính chất đường trịn Euler => H thuộc đường tròn qua I, D, E Đường thẳng HQ cắt AI G Kẻ QK ⊥ BC ⇒ AQCB hình thang cân, AH ⊥ BC ⇒ IH = IK ⇒ AQ = HK = HI , AQ // BC Theo định lí Thales, ta có AG AQ = = ⇒ G trọng tâm tam giác ABC GI HI => H, G, Q thẳng hàng => P, H, G thẳng hàng Bài 172 Cho tam giác nhọn ABC ( AB > AC ) , đường phân giác AD, Gọi E, F thứ tự hình chiếu D AC, AB Đường thẳng BE, CF cắt H Đường tròn ngoại tiếp tam giác AFH cắt BE I Chứng minh ằng BI, IE, BF độ dài ba cạnh tam giác vuông (TST China 2003) LỜI GIẢI Trước hết chứng minh AH vuông góc với BC Giả sử AH cắt BC K Theo định lí Ceva ta có BK CE AF =1 KC EA FB (1) Theo giả thiết ∠BAD = ∠DAC , DF vng góc AB, DE vng góc AC => hai tam giác AED AFD ⇒ AE = AF Từ (1) suy BK CE BK CD cos C = 1⇒ = KC FB KC BD cos B Mặt khác AD phân giác góc A ⇒ CD AC BK AC cos B = ⇒ = BD AB KC AB cos C Liên hệ tài liệu word toán SĐT (zalo): 039.373.2038 121 Website: tailieumontoan.com Từ A kẻ AG ⊥ BC Khi cos B BG AC BK BG BK BG = ⇒ = ⇒ = cos C AB.CG KC CG BC BC ⇒ BK = BG ⇒ K ≡ G => AH vng góc với BC Theo giả thiết DE vng góc với AC, DF vng góc AB, ∠BAD = ∠CAD ⇒ hai tam giác ADE ADF ⇒ DE = DF Theo chứng minh => AH vng góc với BC Gọi K giao điểm AH với BC ⇒ ∠AFK = ∠AKD = 90° => tứ giác AFDK nội tiếp ⇒ ∠FAK = ∠FDB (1) Theo giả thiết đường tròn ngoại tiếp ΔAFH cắt BE I => tứ giác AFIH nội tiếp ⇒ ∠FAH = ∠FIB (2) Từ (1) (2) ⇒ ∠FDB = ∠FIB ⇒ tứ giác BFID nội tiếp => DI vuông góc với BE Theo định lí Pythagos ta có: DI = DB − IB = DE − IE Mà DB = BF + DF nên BF + DF − IB = DE − IE Kết hợp DF = DE ⇒ BF + IE = IB Theo định lí đảo Pythagos, ta có: BF, BI, IE ba cạnh tam giác vuông Bài 173 Cho tam giác nhọn ABC, đường cao BD CE Vẽ hai đường tròn qua A, E tiếp xúc với BC tương ứng P Q Chứng minh giao điểm PD QE thuộc đường tròn ngoại tiếp tam giác ADE (Germany 2009) LỜI GIẢI Ta có BP = BE.BA QP= = BQ BE.BA ⇒ BP ⇒ ∠EQB = ∠EAQ (góc tạo tiếp tuyến) => hai tam giác BQE, BAQ đồng dạng (c.c) ⇒ ∠QEB = ∠AQB (1) Gọi H trực tâm ΔABC F giao AH BC => AF vng góc với BC Ta có CB.CF = CB ( CB − BF ) = CB − CB.BF (2) Tứ giác ACFE nội tiếp ⇒ BF BC = BE.BA = BQ Kết hợp với (2) ta được: CB.CF = CB − CB.BF = CB − BQ = ( CB + BQ )( CB − BQ ) = CQ.CP (3) Tứ giác ADFB nội tiếp ⇒ CD.CA = CF CB Liên hệ tài liệu word toán SĐT (zalo): 039.373.2038 122 Website: tailieumontoan.com Từ (3) suy CP.CQ = CD.CA => tứ giác ADPQ nội tiếp ⇒ ∠PDC = ∠AQB = ∠QEB (theo (1)) Gọi K giao điểm QE PD => tứ giác ADKE nội tiếp => K thuộc đường tròn ngoại tiếp tam giác ADE Bài 174 Cho đường trịn (O) điểm M ngồi đường trịn Kẻ tiếp tuyến MA, MB (O) (A, B tiếp điểm) Qua M kẻ cát tuyến MCD ( MC < MD ) Gọi E điểm đối xứng C qua O Đường thẳng EA cắt BC S SD cắt (O) K MO cắt AE N 1) Chứng minh tam giác OAC tam giác MAS đồng dạng 2) Chứng minh tam giác BKC cân 3) Chứng minh AD vng góc với DN (Tuyển sinh Chuyên Tin HN 2016) LỜI GIẢI 1) * Cách CE đường kính (O) ⇒ ∠CAE = ∠CAS = 90° MA tiếp tuyến (O) ⇒ ∠MAO = 90°, ∠AOM = ∠AOB = ∠AEB = ∠ACS => hai tam giác ACS AOM đồng dạng (g.g) ⇒ AC AS 90° = , ∠CAE = ∠CAS = AO AM ⇒ ∠CAO = ∠SAM ⇒ hai tam giác OAC MAS đồng dạng (c.g.c) * Cách E điểm đối xứng C qua O => CE đường kính đường tròn (O) ⇒ ∠CAE= 90° Mà MA, MB tiếp tuyến (O) => OA ⊥ MA, AB ⊥ MO => ∠MAB + ∠AMO= 90° , ∠CSA + ∠SCA= 90° Tứ giác AEBC nằm đường tròn (O) ⇒ ∠SCA = ∠AEB , MA tiếp tuyến ⇒ ∠MAB = ∠AEB (cùng chắn cung AB ) Từ ta suy ∠AMO = ∠ASC = ∠NSB Mặt khác MA, MB tiếp tuyến ∠AMO = ∠BMO = ∠BMN ⇒ ∠NSB = ∠BMN ⇒ tứ giác SMBN nội tiếp Liên hệ tài liệu word toán SĐT (zalo): 039.373.2038 123 Website: tailieumontoan.com Kết hợp ∠MNA = ∠MNB , ta có MS = MB = MA ⇒ tam giác SMA cân M Tam giác OAC cân O, AC ), ∠CAE = ∠CAS = ∠MAC = ∠AEC (cùng chắn cung 90° ⇒ ∠CAO = ∠SAM ⇒ hai tam giác OAC MAS đồng dạng (g.g) 2) Tam giác OAC cân O => tam giác MAS cân M ⇒ MS = MA = MB ⇒ ∠MSB = ∠MBS = ∠MBC = ∠BDC = ∠BDM => tứ giác MSDB nội tiếp ⇒ ∠BDM = ∠SDM Mặt khác ∠CBK = ∠CDK = ∠MDS = ∠MBS = ∠MBC ⇒ CB = CK ⇒ tam giác BCK cân C 3) Theo chứng minh trên, tứ giác BMSN nội tiếp, từ điểm M, S, D, N, B nằm đường tròn ⇒ ∠SND = ∠SBD = ∠CED ∠DCE = ∠DAE , ∠DCE + ∠ECD = 90° ⇒ ∠DAN + ∠DNA= 90° ⇒ AD vng góc với DN Bài 175 Cho tam giác nhọn ABC ( AB < AC ) nội tiếp đường tròn (O) Hai đường cao BB’, CC’ cắt H, M trung BC Tia HM cắt đường (O) P 1) Chứng minh tam giác BPC’ CPB’ đồng dạng 2) Đường phân giác góc BPC’ CPB’ cắt cạnh AB, AC E F GỌi O’ tâm đường tròn ngoại tiếp tam giác AEF Tia HM cắt AO K Chứng minh tứ giác PEKF nội tiếp (Tuyển sinh Chuyên toán HN 2016) LỜI GIẢI 1) Tia HM cắt đường tròn (O) P Q (Q thuộc cung nhỏ BC) Nối O với M => OM vng góc với BC Tam giác ABC nhọn ⇒ OM = AH ⇒ OM đường trung bình tam giác AHQ => A, O, Q thẳng hàng => AQ đường kính đường trịn (O) ⇒ ∠APQ= 90° => A, P, C, H, B nằm đường trịn đường kính AH ⇒ ∠PCB ′ = ∠PCA = ∠PBA = ∠PBC ′ , PC ′A = ∠PB′A ∠PC ′B = ∠PB ′C => hai tam giác BPC’ CPB’ đồng dạng (g.g) Liên hệ tài liệu word toán SĐT (zalo): 039.373.2038 124 Website: tailieumontoan.com 2) Theo chứng minh hai tam giác BPC’ CPB’ đồng dạng Cũng theo giả thiết PE PF đường phân giác góc BPC’ CPB’ => hai tam giác PEC’ PFB’ đồng dạng ⇒ ∠PEC ′ = ∠PFB ′ ⇒ ∠PEA = ∠PFA => tứ giác APEF nội tiếp => AO’ trung trực EF Giả thiết AO’ cắt HM K ⇒ ∠APK= 90° , AO ′ = O ′P ⇒ O ′P = O ′A => tứ giác PEKF nội tiếp Bài 176 Cho ba điểm A, M, B đường thẳng theo thứ tự C D phía bờ AB cho tam giác ACM MDB đều, AD BC cắt P 1) Chứng minh tứ giác ACPM MPDC tứ giác nội tiếp 2) Chứng minh CP.CB + DP.DA = AB 3) Đường thẳng nối hai tâm đường tròn tứ giác nội tiếp ACPM MPDB cắt AD CB E F Chứng minh tứ giác CEFD hình thang (Tuyên sinh chuyên ĐHSP 2016) LỜI GIẢI 1) Ta có ∠AMC = ∠BMD = 60° ⇒ ∠CMD= 180° − ∠CMA − ∠DMB= 60° ⇒ ∠AMD = ∠CMB = 120° Hai tam giác AMD CMB có = AM CM = , DM MB => hai tam giác (c.g.c) ⇒ ∠MAD = ∠MCB ⇒ tứ giác CAMP nội tiếp ⇒ ∠CPM = 120° = ∠DPM ⇒ tứ giác MPDB nội tiếp 2) Ta có ∠APM = ∠ACM = 60° => hai tam giác PMD MAD đồng dạng (g.g) ⇒ PD MD = ⇒ MD = PD AD ⇒ MD = MD AD DP.DA = MB Tương tự AM = CP.CB ⇒ CP.CB + DP.DA = AB 3) EF nằm đường nối tâm hai đường tròn (CAMP) (DPMB) => EF trung trực PM => hai tam giác EPM FPM Mặt khác, ∠PCM = ∠PAM = ∠PMD ⇒ hai tam giác CPM MPD đồng dạng (g.g) Liên hệ tài liệu word toán SĐT (zalo): 039.373.2038 125 Website: tailieumontoan.com ⇒ CP PM CP PE = ⇒ = MP PD PF PD Theo định lí đảo Thales, ta CD // DF Bài 177 Cho đường tròn tâm (O) đường kính BC A điểm di chuyển đường trịn (O) (A khác B C) Kẻ AH vng góc với BC H M điểm đối xứng điểm A qua điểm B 1) Chứng minh điểm M ln nằm đường trịn cố định 2) Đường thẳng MH cắt (O) E F (E nằm M F) Gọi I trung điểm HC, đường thẳng AI cắt (O) G Chứng minh tổng AF + FG + GE + EA2 khơng phụ thuộc vào vị trí điểm A 3) Gọi P hình chiếu vng góc H lên AB Tìm vị trí điểm A cho bán kính đường trịn ngoại tiếp tam giác BCP đạt giá trị lớn (Tuyển sinh lớp 10 HD 2016) LỜI GIẢI 1) Trên tia đối tia BC lấy điểm D cho BD = BO ⇒ ∠ABO = ∠MBD (đối đỉnh), = BA BM ⇒ hai tam giác ABO MBD (c.g.c) ⇒ DM = OA Mà BD = BO ⇒ D cố định BC đường kính ⇒ ∠BAC= 90° ⇒ OA= BC BC ⇒ M nằm đường tròn tâm D bán kính 2 2) Ta có hai tam giác ABH CAH đồng dạng Gọi N trung điểm AH => BN đường trung bình tam giác ABH => MN song song MH Theo giả thiết, IH = IC theo tính chất hai tam giác đồng dạng (các góc tạo đường tương ứng nhau) ta suy ∠AMF = ∠ABN = ∠CAI = ∠CAG, ∠BAC = 90° ⇒ AG vng góc với MF AO cắt đường tròn (O) K => KG vng góc AG => KG song song với MF => tứ giác EFGK nội tiếp đường tròn (O) => tứ giác EFGK hình thang cân ⇒ GF = KE , KF = GB Các tam giác AFK AEK tam giác vuông 2 ⇒ AF + FK = AK , AE + EK = AK ⇒ AF + FG + GE + EA2 = AF + EK + FK + FG = 2= AK 2 BC 3) Từ H hạ HQ vuông góc với AC => tứ giác APHQ hình chữ nhật => PQ qua trung điểm AH Vì ∠APQ = ∠NAP = ∠ACB ⇒ tứ giác BPQC nội tiếp Đường thẳng vng góc kẻ từ N với PQ từ O với BC cắt J => J tâm đường tròn ngoại tiếp tam giác PBC Tứ giác ANJO hình bình hành Liên hệ tài liệu word toán SĐT (zalo): 039.373.2038 126 Website: tailieumontoan.com ⇒ OA = JN = JC ⇒ JC = OC + JO = BC AH + 4 Vậy JC lớn AH lớn AH = BC ⇔ A điểm cung BC Bài 178 Cho tam giác nhọn ABC Đường trịn tâm O đường kính BC cắt cạnh AC, AB D E Gọi H giao điểm BD CE AH cắt BC F a) Chứng minh AF vng góc với BC ∠AFD = ∠ACE b) Gọi M trung điểm AH Chứng minh MD vng góc với OD, điểm M, D, O, F, E thuộc đường tròn c) Gọi K giao điểm AH DE Chứng minh MD = MK MF K trực tâm tam giác MBC 1 d) Chứng minh = + FK FA FH (Tuyển sinh lớp 10 TPHCM 2016) LỜI GIẢI a) Đường trịn tâm O đường kính BC cắt cạnh AC, AB D E => BD, AC CE vng góc AB => BD CE đường cao ΔABC BD CE cắt H => H trực tâm tam giác ABC => AH vng góc BC => AF vng góc BC Tứ giác DHFC có ∠HDC = 90° ∠HFC = => D F thuộc đường trịn đường kính HC ⇒ ∠AFD = ∠HFD = ∠DCH = ∠ACE b) Tam giác ADH vuông D ⇒ MA = MH ⇒ MA = MH = MD ⇒ ∠MDH = ∠MHD Mặt khác, tứ giác DHFC nội tiếp đường tròn tâm O ⇒ ∠MHD = ∠FCD ⇒ ∠MDH = ∠FCD ⇒ MD tiếp tuyến đường tròn DHFC với tiếp điểm D => OD vng góc với MD => M, D, O, F nằm đường trịn đường kính MO c) Từ chứng minh suy MD tiếp tuyến đường tròn qua (DHFC) ⇒ ∠KFD = ∠MDK ⇒ hai tam giác MKD MDF đồng dạng (g.g) Liên hệ tài liệu word toán SĐT (zalo): 039.373.2038 127 Website: tailieumontoan.com ⇒ MK MD = ⇒ MD = MK MF MD MF MD cắt (O) N, OD vng góc với MD ⇒ ∠MDN = ∠DCN => hai tam giác MDN MCD đồng dạng (g.g) ⇒ MD= MN MC ⇒ MK MF= MN MC ⇒ MN MF = MK MC => hai tam giác MNK MFC đồng dạng (c.g.c) ⇒ ∠MNK = ∠MFC = 90° Mặt khác, ∠BNC= 90° ⇒ B, K, N thẳng hàng => K trực tâm tam giác MBC d) Hai tam giác BHF ACF đồng dạng (g.g) ⇒ BF AF = ⇒ BF CF = AF HF HF CF Tương tự, BF CF = MF KF ⇒ 1 MF MF FH + FA FH + FA ( FH + MH ) + = = = = = = FA FH FA.FH BF CF BF CF BF CF BF CF KF Bài 179 Cho tứ giác ABCD nội tiếp đường tròn AC BD cắt E Tia AD BC cắt F Dựng hình bình hành AEBG 1) Chứng minh đẳng thức FD.FG = FB.FE 2) Gọi H điểm đối xứng E qua FD Chứng minh tứ giác AHFG nội tiếp (Tuyển sinh Chuyên Toán TPHCM 2016) LỜI GIẢI 1) Trên BD lấy điểm K cho FK = FD => tam giác FDK cân F ⇒ ∠FKD = ∠FDK = ∠PDB Tứ giác ABCD nội tiếp ⇒ FBK = ∠DBC = ∠DAC => hai tam giác ADE BKF đồng dạng (g.g) ⇒ AE BF = DE FD Tứ giác AEBG hình bình hành ⇒ AE = BG ⇒ BG BF = DE FD Tứ giác AEBG hình bình hành => BE // AG ⇒ ∠BAC = ∠BAE = ∠ABG ⇒ ∠ADB = ∠ADC − ∠BDC = ∠ABF − ∠ABG = ∠GBF Liên hệ tài liệu word toán SĐT (zalo): 039.373.2038 128 Website: tailieumontoan.com => hai tam giác FDE FBG đồng dạng (c.g.c) ⇒ FD FE = ⇒ FD.FG = FB.FE FB FG 2) Từ tam giác FDE FBG đồng dạng (c.g.c) ⇒ ∠DFE = ∠BFG ⇒ ∠GFA = ∠EFC Tương tự, ∠FAG = ∠FCA ⇒ hai tam giác FAG FCE đồng dạng (g.g) ⇒ ∠AGF = ∠FEC = 180° − ∠FEA = 180° − ∠FHA ⇒ tứ giác AHFG nội tiếp Bài 180 Cho tam giác vuông ABC vuông A Đường trịn tâm O đường kính AB cắt BC D cắt OC I H hình chiếu A OC AH cắt BC M 1) Chứng minh tứ giác ACDH nội tiếp ∠CHD = ∠ABC 2) Chứng minh tam giác OHB tam giác OBC đồng dạng, HM phân giác góc ∠BHD 3) Gọi K trung điểm BD Chứng minh MD.BC = MB.CD MB.MD = MK MC 4) AM cắt OK E, OI cắt đường tròn (O) J Chứng minh OC EJ cắt đường tròn tâm O (Tuyển sinh lớp 10 TPHCM 2017) LỜI GIẢI 1) Theo giả thiết AB đường kính đường trịn (O) ⇒ ∠ADB= 90° H hình chiếu A OC ⇒ ∠AHC= 90° => tứ giác AHDC nội tiếp ⇒ ∠CHD = ∠CAD Mặt khác, tam giác ABC vuông A ⇒ ∠CAD = ∠ABC ⇒ ∠CHD = ∠ABC 2) Tam giác OAC vng A, AH vng góc OC ⇒ OB = OA2 = OH OC ⇒ OB OC = ⇒ hai tam giác OBH OBC đồng dạng (c.g.c) OH OB ⇒ ∠OHB = ∠OBC = ∠ABC Từ kết ⇒ ∠CHD = ∠OHB ⇒ BHM= 90° − ∠OHB= 90° − ∠CHD= ∠DHM => HM phân giác góc ∠BHD 3) * Cách HM phân giác góc ∠BHD ⇒ MB HB = MD HD AH vng góc với HC => HC phân giác ngồi góc ∠BHD ⇒ CB HB MB CB = ⇒ = ⇒ MB.CD = CB.MD CD HD MD CD Gọi G trung điểm MC Tam giác MHC vuông H Liên hệ tài liệu word toán SĐT (zalo): 039.373.2038 129 Website: tailieumontoan.com ⇒ ∠GHM = ∠GMH ⇒ ∠DHG = ∠GHM − ∠DHM = ∠HMG − ∠BHM = ∠HBD => HG tiếp tuyến đường tròn ngoại tiếp tam giác BHD ⇒ GM =GH =GD.GB =( GK − KD )( GK + KB ) = ( GK − KD )( GK + KD ) = GK − KD ⇒ GK − GM = KD ⇒ ( GK + GM )( GK − GM ) = KD ⇒ CK KM = KD ⇒ ( CM + MK ) MK = KD ⇒ CM MK = KD − KM = MD.MB * Cách Gọi N giao điểm AM đường trịn (O) AN vng góc với OC ON = OA ⇒ ∠ONC= 90° => điểm A, C, N, K, O thuộc đường trịn đường kính OC Theo hệ thức đường trịn ta có MK MC = MA.MN MB.MD = MA.MN ⇒ MK MC = MB.MD 4) Gọi P giao điểm EJ đường tròn (O) Theo hệ thức đường trịn, ta có: MI MJ = MA.MN ⇒ MI MJ = MK MC => hai tam giác MKJ MCI đồng dạng ⇒ ∠MCI = ∠MJK Mặt khác, ∠MCI = ∠MCH = ∠MEK ⇒ ∠MJK = ∠MEK => tứ giác KJEM nội tiếp ⇒ ∠EJM= 90° ⇒ ∠IJP= 90° => P thuộc đường tròn (O) Bài 181 Cho tam giác ABC ( AB < AC ) E, F trung điểm AC AB Đường trung trực EF cắt BC D P điểm góc ∠FAE ngồi tam giác AEF thỏa mãn ∠PEC = ∠DEF ∠PFB = ∠DFE PA cắt đường tròn ngoại tiếp tam giác PEF Q (Q khác P) 1) Chứng minh góc ∠EQF = ∠BAC + ∠EDF 2) Tiếp tuyến P với đường tròn ngoại tiếp tam giác PEF cắt AC, AB M N Chứng minh B, N, C, M nằm đường trò, gọi đường tròn (K) 3) Chứng minh đường tròn (K) tiếp xúc với đường tròn ngoại tiếp tam giác AEF (Tuyển sinh Chuyên KHTN 2017) LỜI GIẢI Liên hệ tài liệu word toán SĐT (zalo): 039.373.2038 130 Website: tailieumontoan.com 1) Theo giả thiết, P điểm góc ∠FAE tam giác AEF thỏa mãn ∠PEC = ∠DFE ⇒ DE PE đối xứng qua ∠DEF ∠PFB = đường phân giác góc ∠FEC DE PF đối xứng qua đường phân giác ∠EFB => P giao điểm hai đường đối xứng Tứ giác FQEP nội tiếp => ∠FQE = 180° − ∠FPE = ∠EFP + ∠FEP = ∠DFB + ∠DEC = ( ∠DFA + ∠ADF ) + ( ∠EAD + ∠ADE ) = ∠BAC + ∠FDE 2) Theo giả thiết PM tiếp tuyến đường tròn ngoại tiếp tam giác PEF ⇒ ∠EPM = ∠EFP = ∠DFB Trung trực EF cắt BC D ⇒ DE = DF ⇒ tam giác DEF cân D ⇒ ∠DEF = ∠DFE ⇒ ∠MEP = ∠DEF = ∠DFE = ∠FDB (do EF song song với BC) ⇒ ∠EMP = 180° − ∠EPM − ∠MEP = 180° − ∠DFB − ∠FDB = ∠FBD ⇒ ∠NBC = ∠NMC => B, N, C, M nằm đường tròn 3) Gọi X, Y, Z điểm điểm đối xứng P qua EF, AC, AB ⇒ AY =∠ AZ , PEC = ∠DEF ⇒ ∠DEY = ∠DEX , EX = EY ⇒ DX = DY Tương tự, DX = DZ Từ ta có DX = DY = DZ Mà AY = AP = AZ ⇒ ∠DAZ = ∠DAY Do tính đối xứng ⇒ ∠PAB = ∠DAC Gọi I giao điểm AP với đường trịn (AEF) Ta có ∠FIA = ∠FEA = ∠ACB = ∠ANP => FIPN tứ giác nội tiếp ⇒ ∠AIE = ∠AFE = ∠ABC = ∠AMP ⇒ IPME tứ giác nội tiếp => Hai tam giác AFI ADC đồng dạng (g.g) Liên hệ tài liệu word toán SĐT (zalo): 039.373.2038 131 Website: tailieumontoan.com Mặt khác ∠FQP = ∠FEP = ∠DEC ⇒ hai tam giác FQI DEC đồng dạng (g.g) Từ đó, kết hợp EA = EC ⇒ QA = QI => FQ song song với BI, ∠IME = ∠IPE = ∠QFE = ∠IBC ⇒ tứ giác BIMC nội tiếp ⇒ ∠FIB = ∠FPB = ∠FEP = ∠FEI + ∠IEP = ∠FEI + ∠IMP Kẻ tiếp tuyến Ix với đường tròn (K) ⇒ ∠xIB = ∠ICB ⇒ ∠xIF = ∠IEF => Ix tiếp xúc với đường tròn (AEF) Bài 182 Cho tứ giác ABCD nội tiếp đường tròn (O) AB CD cắt E, AB CD cắt F, AC BD cắt G Gọi M giao điểm thứ hai đường tròn ngoại tiếp hai tam giác ADE DCF Đường phân giác góc ∠AMB cắt AB I, đường phân giác góc ∠CMD cắt CD J Chứng minh I, J, G thẳng hàng (LQĐ 2014) LỜI GIẢI Theo tính chất tứ giác toàn phần => M giao điểm đường tròn ngoại tiếp tam giác ADE, DCF, ABF, BCE ⇒ ∠MDA = ∠MDF = ∠MCF = ∠MCB , ∠MAD = 180° − ∠MEC = ∠MBC => tam giác MAD MBC đồng dạng (g.g) ⇒ MA AD MD (1) = = MB BC MC MI phân giác góc ∠AMB ⇒ MA IA = (2) MB IB MJ phân giác góc ∠CMD ⇒ MD JD = (3) MC JC Tam giác GAD tam giác GBC đồng dạng (g.g) ⇒ GA AD GD (4) = = GB BC GC Từ (1), (2), (3) (4) ⇒ GA IA JD GD = = = GB IB JC GC => GI phân giác góc ∠AGB GJ phân giác góc ∠CGD Mà góc ∠AGB ∠CGD đối đỉnh => I, J, G thẳng hàng Bài 183 Cho tam giác nhọn ABC ( AB < AC ) nội tiếp đường tròn (O) AE đường cao H trực tâm tam giác ABC, D trung điểm BC, AD cắt đường tròn (O) F 1) Chứng minh BC = DA.DF 2) Tia DH cắt đường tròn (O) G Chứng minh A, G, E, D thuộc đường tròn Liên hệ tài liệu word toán SĐT (zalo): 039.373.2038 132 Website: tailieumontoan.com 3) EF cắt đường tròn (O) K Chứng minh BC tiếp xúc với đường tròn ngoại tiếp tam giác GKE (Tuyển sinh vào Chuyên tin HN 2017) LỜI GIẢI 1) Tam giác ADB CDF đồng dạng (g.g) ⇒ AD BD = CD DF BC ⇒ AD.DF = BD.DC = BD = ⇒ BC = DA.DF 2) Nối AO cắt đường tròn (O) M ⇒ ∠ABM = ∠ACM = 90° H trực tâm tam giác ABC => BH vuông góc với AC => BH song song với CM Tương tự, MB song song CH => tứ giác BHCM hình bình hành => MH qua trung điểm BC => M, H, G thẳng hàng ⇒ ∠AGD = ∠AED = 90° => A, G, E, D thuộc đường tròn 3) ∠EGD= ∠EAD= 90° − ∠ADE= 90° − ( ∠DEF + DFE ) ⇒ ∠DEF= 90° − ( ∠EGD + ∠DFE ) ⇒ ∠KEB = ∠DEF = 90° − ( EGD + DEF ) (1) Mặt khác, tứ giác AGKF nội tiếp đường tròn (O) ⇒ ∠DFE = ∠AFK = 180° − ∠AGK = 180° − ∠AGD − ∠DGE − ∠EGK = 90° − DGE − ∠EGK (2) Thay (2) vào (1) ta ∠KEB = ∠DEF = 90° − ( EGD + DEF ) = 90° − ( ∠EGD + 90° − ∠DGE − ∠EGK ) = ∠EGK => BE tiếp xúc với đường tròn ngoại tiếp tam giác GKE Bài 184 Cho tam giác nhọn ABC ( AB < AC ) nội tiếp đường tròn (O) I tâm đường tròn nội tiếp tam giác ABC D hình chiết I BC G giao điểm AD với đường tròn (O) F điểm cung lớn BC FG cắt ID H 1) Chứng minh tức giác IBHC nội tiếp 2) Gọi J giao điểm AI với đường tròn ngoại tiếp tam giác BIC Chứng minh BH = CJ 3) Gọi N giao điểm FH với đường tròn ngoại tiếp tam giác BIC Chứng minh NJ qua trung điểm BC (Tuyển sinh Chuyên toán HN 2017) LỜI GIẢI Liên hệ tài liệu word toán SĐT (zalo): 039.373.2038 133 Website: tailieumontoan.com 1) Gọi E giao điểm AI đường trịn (O) Theo tính chất đường trịn nội tiếp ta có EB = EC ⇒ FE vng góc với BC Mà ID vng góc BC => ID song song với FE ⇒ ∠IHG = ∠GFE = ∠GAE => tứ giác IAHG nội tiếp ⇒ DA.DG = DI DH = DB.DC => B, I, C, H nằm đường tròn hay tứ giác IBHC nội tiếp 2) Cũng theo tính chất đường trịn nội tiếp ⇒ EB = EC = EI ⇒ E tâm đường tròn ngoại tiếp tam giác BIC => E tâm đường tròn ngoại tiếp tứ giác IBHC AI cắt đường tròn ngoại tiêp tam giác BIC J ⇒ EJ = EI ⇒ J thuộc đường tròn ngoại tiếp tứ giác IBHC => tứ giác BCJH hình thang cân ⇒ BH = CJ 3) EC vng góc với AC => AC tiếp tuyến đường tròn ngoại tiếp tam giác BIC Tương tự BF tiếp tuyến đường tròn ngoại tiếp tam giác BIC => HF đường đối trung tam giác BHC => HK trung tuyến tam giác HBC Mà HJ song song với BC => N, K, J thẳng hàng => NJ qua trung điểm BC Bài 185 Cho hai đường tròn (O1) (O2) cắt A B Trên tia AB lấy điểm C Kẻ tiếp tuyến CD CE với đường tròn (O1) (E nằm (O2)), AD AE cắt đường tròn (O1) M N DE cắt MN I AB cắt O1O2 H DE J O1O2 cắt DE F Chứng minh rằng: 1) EF JD = FD.JE 2) MB.EB.DI = IB AN BD 3) O2I vng góc với MN LỜI GIẢI 1) CD, CE tiếp tuyến với đường trịn (O1) => O1D vng góc với CD O1E vng góc với CE Đường trịn (O1) (O2) cắt A B => O1O2 vuông góc với AB hay ∠CHO1 =° 90 => năm điểm C, D, O1, H, E nằm đường tròn đường kính CO1 ⇒ ∠CHD = ∠CO1 D = ∠CDE = ∠CHE => CH phân giác góc ∠DHE Theo tính đường phân giác, ta có: HF phân giác ngồi góc ∠DHE ⇒ HD FD JD = = HE FE JE Liên hệ tài liệu word toán SĐT (zalo): 039.373.2038 134 Website: tailieumontoan.com ⇒ EF JD = FD.JE 2) Trong đường trịn (O2), ta có: ∠BMI = ∠BMN = ∠BAN = ∠BAE Trong đường tròn (O1) ta có: ∠BDI = ∠BDE = ∠BAE Từ ta có ∠BMI = ∠BDI => tứ giác DBIM nội tiếp ⇒ ∠MBI = ∠IDM = ∠IDA = ∠ABE => tam giác MBI tam giác ABE đồng dạng (g.g) ⇒ MB BI = AB BE Ta có ∠BAE = ∠BDE , ∠BNA = ∠BMA = ∠BMD = ∠BID ⇒ tam giác ABN tam giác DBI đồng dạng (g.g) ⇒ AB AN = DB DI Nhân hai đẳng thức với ta được: MB AB BI AN = ⇒ MB.BE.DI = IB AN BD AB DB BE DI 3) Tứ giác DBIM nội tiếp ⇒ ∠BIN = ∠BDM = ∠BDA, ∠BEN = ∠BDA ⇒ ∠BEN = ∠BIN ⇒ tứ giác BEIN nội tiếp ⇒ ∠NBI = ∠NEI = ∠DEA = ∠DBA => tam giác IBN tam giác DBA đồng dạng (g.g) ⇒ IN DA = IB DB CD tiếp tuyến (O1) => tam giác CAD tam giác CDB đồng dạng (g.g) CD =CE ⇒ DA CD CE = = DB CB CB CE tiếp tuyến (O1) => tam giác CBE tam giác CEA đồng dạng ⇒ CE CA = CB CE Tam giác MBI ABE đồng dạng ⇒ EA IM IN IM = ⇒ = ⇒ IN = IM ⇒ O2I vng góc với MN EB IB IB IB Bài 186 Cho tứ giác ABCD nội tiếp đường tròn AC BD cắt P, AD BC cắt Q thỏa mãn PQ vuông góc với AC E trung điểm AB Chứng minh PE vng góc với BC (Pakistan 2017) LỜI GIẢI Liên hệ tài liệu word toán SĐT (zalo): 039.373.2038 135 Website: tailieumontoan.com Gọi K giao điểm đường tròn ngoại tiếp tam giác AQC đường tròn ngoại tiếp tam giác QBD Tứ giác ABCD nội tiếp đường tròn => giao điểm (ABCD) Và đường tròn (AQC) A C, giao điểm (ABCD) đường tròn (QBD) B D => Q, P, K thẳng hàng ⇒ ∠DQK = ∠DBK Ta có ∠ADB = ∠ACB ⇒ ∠QDB = ∠ACQ ⇒ hai tam giác QAC QBD đồng dạng (g.g) => ∠QAC = ∠QBD Theo giả thiết QP vng góc với AC ⇒ 90° = ∠AQP + ∠QAP = ∠PBK + ∠QBD ⇒ ∠KBD= 90° , hay KB vng góc với BC Gọi H trực tâm tam giác QAC => AH vng góc với QC, QP vng góc với AC => H thuộc BK Theo tính chất trực tâm tam giác ta có PH = PK Theo giả thiết EA = EB ⇒ PE song song với KB => PE vng góc với QC Bài 187 Cho tam giác nhọn ABC Tiếp tuyến B C với đường tròn ngoại tiếp tam giác ABC cắt P Gọi D điểm đối xứng B qua AC, E điểm đối xứng C qua AB, O tâm đường tròn ngoại tiếp tam giác PDE Chứng minh AO vuông góc với BC LỜI GIẢI * Cách Theo giả thiết, tiếp tuyến B C cắt P ⇒ PB = PC D điểm đối xứng B qua AC, E điểm đối xứng C qua AB ⇒ CD = CB ⇒ ∠BCD = 2∠ACB BE = BC ⇒ ∠CBE = 2∠ABC Ta có ∠PBC = ∠PCB = ∠BAC ⇒ ∠PCD = ∠PCB + ∠BCD = ∠BAC + 2∠ACB ⇒ ∠PBE = 360° − ∠PBC − ∠CBE = 360° − ∠BAC − 2∠ABC = 2(∠ABC + ∠BCA + ∠CAB) − ∠BAC − 2∠ABC = ∠BAC + 2∠ACB ⇒ ∠PBE = ∠PCD ⇒ tam giác BPC CPD (c.g.c) ⇒ PE = PD ⇒ tam giác PDE cân P ⇒ ∠PDE = ∠PED ⇒ ∠PBC = ∠PED Gọi K giao điểm PB đường tròn ngoại tiếp tam giác PDE ⇒ ∠BKD = ∠PED = ∠PBC ⇒ BC song song với KD ⇒ ∠BAC = ∠BKD Theo giả thiết, CB = CD AC vng góc với BD ⇒ AB = AD Liên hệ tài liệu word toán SĐT (zalo): 039.373.2038 136 Website: tailieumontoan.com ⇒ ∠BAC = ∠CAD ⇒ ∠BAD = 2∠BAC = 2∠BKD ⇒ A tâm đường tròn ngoại tiếp tam giác DKP O tâm đường tròn ngoại tiếp tam giác PDK ⇒ O A nằm trung trực DK ⇒ AO vng góc DK Mà DK song song BC ⇒ AO vng góc với BC *Cách Gọi J tâm đường tròn ngoại tiếp tam giác PBC Tương tự cách 1, ta chứng minh tam giác PCD tam giác PBE ⇒ ∠CPD = ∠BPE , PD = PE ⇒ Tam giác PDE cân P PB tiếp tuyến với đường tròn ngoại tiếp tam giác ABC ⇒ tam giác PBC cân P ∠BPC = ∠BPD + ∠DPC = ∠BPD + ∠BCP = ∠DPE ⇒ tam giác PBC tam giác PDE đồng dạng BE = BC AB vng góc với CE ⇒ AE = AC ∠EAC = 2∠BAC Tam giác JBP cân ⇒ ∠BJP = 2∠BAC ⇒ tam giác AEC tam giác JBP đồng dạng (g.g) O tâm đường tròn ngoại tiếp tam giác PDE ⇒ OP = OE ∠POE = 2∠BAC ⇒ tam giác OPE đồng dạng tam giác JBP ⇒ ba tam giác AEC, OEP, JBP đồng dạng ⇒ ∠AEC = ∠OEP ⇒ ∠AEO = ∠CEP AE AC = OE EP ⇒ tam giác AEO tam giác CEP đồng dạng (c.g.c) ⇒ ∠AOE = ∠CPE , ∠OEP = ∠JBP = ∠JPC ⇒ AO song song JP Mà JP vng góc BC ⇒ AO vng góc BC *Cách Trước hết ta chứng minh toán tổng quát: Cho tam giác ABC, P điểm đường tròn ngoại tiếp tam giác ABC Kẻ cát tuyến PB PC cắt đường tròn ngoại tiếp ABC N M Gọi D E điểm đối xứng B C qua AM, AN K tâm đường tròn ngoại tiếp tam giác PDE Chứng minh AK vng góc với MN Thật vậy: tam giác PMB tam giác PNC có ∠PMB = ∠CMB = ∠CNB = ∠CNP ⇒ hai tam giác đồng dạng (g.g) Liên hệ tài liệu word toán SĐT (zalo): 039.373.2038 137 Website: tailieumontoan.com ⇒ PM PN PM PN (*) = ⇒ = BM CN MD NE Theo giả thiết ta có B D đối xứng với qua AM ⇒ ∠AMD = ∠AMB ∠BMC = ∠BAC ⇒ ∠BMD = 2∠AMB ⇒ ∠PMD = ∠PMC + ∠BMD = ∠BAC + 2∠BCA C E đối xứng với qua AN ⇒ ∠CNE = 2∠CNA = 2∠CBA ⇒ ∠PNE = 360° − ∠PNC − ∠CNE = 2(∠ABC + ∠BCA + ∠CAB ) − ∠BAC − 2∠ABC =∠BAC + 2∠BCA ⇒ ∠PMD = ∠PNE Kết hợp (*) suy tam giác PMD PNE đồng dạng (c.g.c) ⇒ ∠DPM = ∠EPN ⇒ ∠DPE = ∠DPN + ∠NPE = ∠DPN + ∠MPD = ∠MPN Mà PD PM = PE PN ⇒ tam giác DPE đồng dạng với tam giác MPN (c.g.c) ⇒ ∠PNM = ∠PED (**) Gọi F giao điểm PB với đường tròn ngoại tiếp tam giác PDE ⇒ ∠BAD = 2∠BAM = 2∠BNM = 2∠PFD ⇒ A tâm đường tròn ngoại tiếp tam giác BDF ⇒ AK vng góc với DF Mặt khác ∠PED = ∠PFD Kết hợp (**) suy ra: DF song song MN ⇒ AK vng góc với MN Trường hợp PB, PC tiếp tuyến đường tròn ngoại tiếp tam giác ABC ⇒ M ≡ C N ≡ B ⇒ AK vng góc với BC Bài 188 Cho đường trịn (O) điểm C ngồi đường trịn (O) Từ C kẻ tiếp tuyến CA, CB với đường tròn (O) M điểm đối xứng A qua B Đường tròn ngoại tiếp tam giác CBM cắt đường tròn (O) D (D khác B) CD cắt đường tròn (O) E (E khác D) Chứng minh EM tiếp xúc đường tròn ngoại tiếp tam giác CBM LỜI GIẢI Liên hệ tài liệu word toán SĐT (zalo): 039.373.2038 138 Website: tailieumontoan.com *Cách Theo giả thiết, C, D, B, M thuộc đường tròn A, D, B, E thuộc đường tròn ⇒ ∠MCE = ∠MCD = ∠ABD = ∠AED ⇒ AE song song với CM CA tiếp tuyến đường tròn (O) ⇒ ∠MAC = ∠BEA ⇒ tam giác CMA tam giác BAE đồng dạng (g.g) ⇒ CM CA MA = = BA BE AE Theo giả thiết AB = BM ⇒ MA AE = MC MB ⇒ tam giác AME tam giác BMC đồng dạng (c.g.c) ⇒ ∠AME = ∠BCM ⇒ ∠BME = ∠BCM ⇒ EM tiếp tuyến đường tròn ngoại tiếp tam giác CBM Cách Giả thiết C, D, B, M thuộc đường tròn A, D, B, E thuộc đường tròn ⇒ ∠BMC = ∠BDE = ∠BAE ⇒ AE song song với MC Gọi N giao điểm BC AE ⇒ ∠ANC = ∠NCM Mặt khác tam giác AME tam giác BMC đồng dạng (c.g.c) ⇒ ∠AME = ∠BCM ⇒ ∠BME = ∠BNE ⇒ B, E, N, M thuộc đường tròn ⇒ ∠EMB = ∠BCM ⇒ EM tiếp tuyến đường tròn ngoại tiếp tam giác CBM *Cách Giả thiết C, D, B, M thuộc đường tròn A, D, B, E thuộc đường tròn ⇒ ∠BMC = ∠BDE = ∠BAE ⇒ AE song song với MC Ta có ∠CMB = 180° − ∠CDB = ∠EAB Từ A kẻ đường thẳng song song với EM cắt CM N ⇒ tứ giác AEMN hình bình hành ⇒ ∠BNC = 180° − ∠BNM = 180° − ∠AEB = ∠MBC ⇒ tam giác BNC tam giác MBC đồng dạng (g.g) ⇒ CN CB = ⇒ CB = CM CN = CA2 CB CM ⇒ CA CN = CM CA ⇒ tam giác ANC tam giác MAC đồng dạng (c.g.c) ⇒ ∠CAN = ∠CMA = ∠CBN ⇒ tứ giác ACNB nội tiếp Liên hệ tài liệu word toán SĐT (zalo): 039.373.2038 139 Website: tailieumontoan.com ⇒ ∠EMB = ∠BAN = ∠BCM ⇒ EM tiếp tuyến đường tròn ngoại tiếp tam giác CBM Bài 189 Cho tứ giác ABCD, AC = BD AC cắt BD P Gọi (O1 ) đường tròn ngoại tiếp tam giác ABP, (O2 ) đường tròn ngoại tiếp tam giác CDP BC cắt (O1 ), (O2 ) theo thứ tự E F Gọi M điểm cung PE (O1 ) (không chứa A), N điểm cung PF (O2 ) (khơng chứa A) Chứng minh MN song song với O1O2 LỜI GIẢI *Cách Gọi Q giao điểm đường tròn (O1 ) với đường tròn (O2 ) (Q khác P) ⇒ ∠QCP = ∠QDP, ∠QAP = ∠QBP Theo giả thiết, AC = DB ⇒ tam giác QAC tam giác QBD (g.c.g) ⇒ QD = QC QB = QA ⇒ tam giác QDC tam giác QAB cân Mặt khác, ∠QDC = ∠QPC = ∠ABQ ⇒ ∠ABQ = ∠CQD ∠DPA ⇒ ∠BPQ = ∠CPQ ⇒ PQ phân giác góc ∠BPC = Theo giả thiết, M điểm cung PE đường trịn (O1 ) ⇒ BM tia phân giác góc ∠PBC Tương tự CN phân giác góc ∠PBC Gọi I giao điểm BM CN ⇒ I thuộc PQ I tâm đường tròn nội tiếp tam giác PBC Mặt khác theo hệ thức đường tròn, ta có IB= IM IP = IQ IC.IN ⇒ bốn điểm B, N, M, C thuộc đường tròn ⇒ MN song song với đường phân giác ngồi góc ∠BPC ⇒ MN vng góc PQ ⇒ MN song song O1O2 *Cách Gọi Q giao điểm đường tròn (O1 ) với đường tròn (O2 ) (Q khác P) Liên hệ tài liệu word toán SĐT (zalo): 039.373.2038 140 Website: tailieumontoan.com Đường trịn (O1 ) có bán kính R1 , đường trịn (O2 ) có bán kính R2 I giao điểm O1O2 PQ Từ giả thiết AC = BD , chứng minh tương tự cách ta tam giác QCD QAB đồng dạng ⇒ BQ R1 = CQ R2 Mặt khác ∠O1O2 J = ∠O1O2 N = ∠PO2O1 − ∠PO2 N = ∠PCQ − ∠PCF = ∠BCQ Tương tự ∠O2O1 J = ∠CBQ ⇒ tam giác O1 JO2 tam giác BQC đồng dạng (g.g) ⇒ JO1 QB R1 O1M = = = JO2 QC R2 O2 N Theo định lý Thales ta có MN song song O1O2 Bài 190 Cho tam giác ABC, đường cao AD, BE CF EF cắt BC A1 Gọi I, J, K trung điểm BC, CA, AB Gọi A2 hình chiếu I AA1 , tương tự ta có B1 , B2 C1 , C2 Chứng minh IA2 , JB2 , KC2 đồng qui LỜI GIẢI Gọi H trực tâm tam giác ABC Ta có ∠ADI = ∠AA2 I = 90° ⇒ tứ giác AA2 DI nội tiếp Theo hệ thức đường tròn ta có A1 A2 A1 A = A1 D A1 I (1) Tứ giác BFEC nội tiếp ⇒ A1 F A1 E = A1 B A1C (2) Theo giả thiết D, E, F chân đường cao tam giác ABC I trung điểm cạnh BC, theo đường tròn Eurle (9 điểm) ⇒ tứ giác FEID tứ giác nội tiếp ⇒ A1 F A1 E = A1 D A1 I (3) Từ (1), (2) (3) ⇒ A1 F A1 E = A1 A2 A1 A Theo dấu hiệu tứ giác nội tiếp bốn điểm A, A2 , F , E nằm đường tròn Mặt khác, BE, CF đường cao tam giác ABC nên bốn điểm A, F, H, E thuộc đường tròn ⇒ năm điểm A2 , A, F, H, E thuộc đường tròn ∠AEH = ∠AFH = 90° ⇒ ∠AA2 H = 90° hay HA2 vng góc với AA1 Theo giả thiết IA2 vng góc AA1 ⇒ I , H , A2 thẳng hàng ⇒ IA2 qua H Tương tự, JB2 , KC2 qua H ⇒ IA2 , JB2 , KC2 đồng quy Bài 191 Cho tam giác ABC Đường phân giác ∠BAC cắt cạnh BC D cắt đường tròn ngoại tiếp tam giác ABC E M trung điểm BC Đường tròn ngoại tiếp tam giác ADM cắt cạnh AC, AB P Q N trung điểm PQ, H hình chiếu E ND Chứng minh EM tiếp tuyến đường tròn ngoại tiếp tam giác MNH LỜI GIẢI Liên hệ tài liệu word toán SĐT (zalo): 039.373.2038 141 Website: tailieumontoan.com Ta có BQ.BA = BD.BM ⇒ BQ = Tương tự CP = BD.BM BA CM CD CA AD đường phân giác ∠BAC ⇒ Hay BD AB = DC AC BD DC = BA AC M trung điểm BC ⇒ MB = MC Suy BQ = CP Gọi I, J trung điểm BP CQ , MJ NI ⇒ MI song song CP MI = CP= = , MI NJ Theo chứng minh trên: BQ = CP ⇒ tứ giác NJMI hình thoi ⇒ MN phân giác góc ∠INJ ⇒ MN song song với AE MB = MC , ∠BAE = ∠EAC ⇒ EB = EC ⇒ tam giác EBC cân E ⇒ EM vng góc với BC Giả thiết: EH vng góc ND ⇒ tứ giác DMEH tứ giác nội tiếp ⇒ ∠HME = ∠HDE = ∠HNM ⇒ EM tiếp tuyến đường tròn ngoại tiếp tam giác MHN Bài 192 Cho hai đường tròn (O1 ) (O2 ) cắt A B, AB đường kính đường trịn (O1 ) Qua B kẻ tiếp tuyến với đường tròn (O1 ) cắt đường tròn (O2 ) C CA cắt đường tròn (O1 ) D BD cắt đường tròn (O2 ) E M điểm CE, MD cắt đường tròn (O1 ) N AN cắt BC P Chứng minh CM CB = ME BP (Taiwan 2016) LỜI GIẢI *Cách Theo giả thiết, BC tiếp tuyến đường tròn (O1 ) ⇒ ∠ABC= 90° ⇒ AC đường kính đường trịn (O2 ) ⇒ điểm O2 AC Ta có ∠ADN = ∠MDC (đối đỉnh), ∠AND = ∠ABD = ∠ACE ⇒ ∠DMC = 180° − ∠MDC − ∠DCM = 180° − ∠ADN − ∠AND = ∠DAN ⇒ ∠CMD = ∠CAP ⇒ tam giác MDC tam giác APC đồng dạng (g.g) ⇒ CM CD = (1) CA CP Tam giác DCE tam giác BCA hai tam giác vuông đồng dạng ⇒ CA CB =.(2) CE CD Nhân hai đẳng thức (1) (2) ta Liên hệ tài liệu word toán SĐT (zalo): 039.373.2038 142 Website: tailieumontoan.com CM CB CM CB = ⇒ = CE CP CE − CM CP − CB ⇒ CM CB = ME PB *Cách Tứ giác ANBD ABCE nội tiếp ⇒ ∠AND = ∠ABD = ∠ABE = ∠ACE Mà ∠ADN = ∠MDC ⇒ tam giác DAN tam giác DMC đồng dạng (g.g) ⇒ Tương tự CM CD = (1) NA ND NB ND (2) = EM ED Nhân hai đẳng thức (1) với (2) ta CM NB CD ND CD CM CD.NA = = ⇒ = NA EM ND ED ED EM ED.NB Tam giác DCE tam giác DBA đồng dạng (g.g) ⇒ CD BD = DE AD Tam giác ABC vng BD vng góc AC ⇒ BD BC CD BC = ⇒ = AD BA DE BA Tam giác ABP vuông BN vuông góc AP ⇒ NA AB CM CD.NA CB AB CB = ⇒ = = = NB BP EM ED.NB AB BP BP *Cách Theo bổ đề diện tích CM SCDM DC sin ∠CDM BD sin ∠ADN BD AN ; = = = = ME S MDE DE sin ∠MDC DA sin ∠BDN AD.BN CB SCAB CA.BD = = BP S PAB PA.BN Tam giác BAC vuông A ⇒ AD AC = AB = AN AP ⇒ CA AN CA.BD AN BD CM CB = ⇒ = ⇒ = AP AD PA.BN AD.BN ME BP Bài 193 Cho tam giác ABC, M trung điểm BC Đường trịn đường kính AM cắt AB, AC P Q D điểm đường tròn ngoại tiếp tam giác ABC cho tứ giác APDQ hình bình hành Tính AD AM (IOM Shorlist 2015) LỜI GIẢI Liên hệ tài liệu word toán SĐT (zalo): 039.373.2038 143 Website: tailieumontoan.com Theo giả thiết APDQ hình bình hành ⇒ ∠BAD = ∠ADQ D đường tròn ngoại tiếp tam giác ABC ⇒ ∠BAD = ∠BCD ⇒ ∠BCD = ∠ADQ Mà ∠QAD = ∠CAD = ∠CBD ⇒ tam giác BDC tam giác AQD đồng dạng (g.g) ⇒ BD DC BC = = AQ QD AD Gọi I, J trung điểm cạnh AC AB Tứ giác APMQ nội tiếp ⇒ ∠MPQ = ∠MAQ Mà ∠CIM = ∠CAB ⇒ ∠AIM = 180° − ∠CIM = 180° − ∠CAB = ∠PMQ Mà ∠IAM = ∠MPQ ⇒ tam giác AIM tam giác PMQ đồng dạng (g.g) ⇒ AM MI IA AM AB AC = = ⇒ = = PQ QM MP PQ QM MP ⇒ AM BC BD AC DC AB = = AD.PQ AQ.PM QD.MQ Biến đổi tỉ lệ thức áp dụng định lý Ptolemy tứ giác ABDC APMQ ta có: BD AC + DC AB AD.BC AM BC AD.BC = ⇒ = AQ.PM + QD.MQ AM PQ AD.PQ AM PQ ⇒ AM = AD ⇒ AD = AM Liên hệ tài liệu word toán SĐT (zalo): 039.373.2038 144 Website: tailieumontoan.com IV BÀI TỐN DỰNG HÌNH Dựng hình góp vai trị quan trọng giải tốn hình học Nhiều tốn khơng nhắc tới “dựng hình” song bắt buộc trước giải địi hỏi phải biết vẽ hình theo giả thiết, từ giúp tìm hướng giải tốn Dựng hình giúp phát triển tư nhiều kỹ sáng tạo, cẩn thận, xác… Lí thuyết dựng hình hình học Euclid đưa khái niệm sở thừa nhận, hệ thống tiên đề cho phép ta thực phép dựng Trong phần này, ta xét tốn dựng hình dụng cụ thước (thước thẳng) compa mà quen gọi toán dựng hình Những phép dựng hình bản: - Dựng đoạn thẳng đoạn thẳng cho trước; - Dựng góc góc cho trước; - Dựng đường trung trực đoạn thẳng, dựng trung điểm đoạn thẳng; - Dựng đường phân giác góc cho trước; - Dựng đường thẳng vng góc (song song) qua điểm với đường thẳng cho trước; - Chia đoạn thẳng làm nhiều phần nhau; - Dựng đường trịn biết ba điểm cho trước (khơng thẳng hàng); - Chia đơi (hoặc nhân đơi) góc; - Dựng tiếp tuyến qua điểm cho trước tới đường tròn… Để giải tốn dựng hình thơng thường ta phải thực qua bốn bước sau: 1) Phân tích: Giả sử hình cần dựng dựng được, thỏa mãn điều kiện toán kết nối chúng với từ để tới cách dựng Thực chất đưa toán cho toán mà ta biết; 2) Cách dựng: Thực bước dựng theo thứ tự phân tích thỏa mãn điều kiện toán; 3) Chứng minh: Chứng minh tính đắn bước dựng; 4) Biện luận: Căn vào cách dựng để tìm điều kiện tồn hình dựng Trong bốn bước, bước phân tích cốt lõi tốn dựng hình Khi phân tích đúng, tốn dựng hình xong Chú ý điều kiện toán dựng hình có yếu tố độ dài đoạn thẳng Lời giải toán dựng hình thước compa số hữu hạn phép dựng Từ thời kì Hy Lạp cổ đại, người ta đưa ba tốn dựng hình cổ điển sau chứng minh dựng thước compa: 1) Gấp đơi hình lập phương Bài tốn: Cho khối lập phương dựng khối lập phương tích gấp hai lần thể tích khối lập phương cho Ở cho khối lập phương nghĩa cho cạnh hay cho đoạn thẳng biết a dựng đoạn thẳng b thỏa mãn b3 2.a3 Điều có nghĩa dựng b a đoạn b dựng dựng (vì đa thức x bất khả qui trường số hữu tỉ); Liên hệ tài liệu word toán SĐT (zalo): 039.373.2038 Website: tailieumontoan.com 2) Chia ba góc Bài tốn: Cho trước góc, dùng thước compa chia góc thành ba góc nhau; 3) Phép cầu phương đường trịn Bài tốn: Cho hình trịn, thước compa dựng hình vng có diện tích diện tích hình trịn (Hình trịn có bán kính r diện tích r , cạnh hình vng a a2 r hay a r hồn tồn khơng dựng được) Dựng đa giác số lẻ cạnh Năm 1796, nhà toán học Carl Friedrich Gauss (1777 – 1855) người Đức mệnh danh “ông Vua tốn học” tìm cách vẽ đa giác có 17 cạnh thước compa Năm năm sau ơng khai triển lí thuyết gọi “Chu kỳ Gauss” Lí thuyết giúp ơng tìm điều kiện đủ để đa giác dựng thước compa Điều kiện cần đủ đa giác n cạnh dựng thước compa n tích lũy thừa với số nguyên tố Fermat Số nguyên tố Fermat số có dạng F 2k , k số nguyên dương Đến người ta biết có số nguyên tố Fermat là: F1 21 tam giác đều, F2 22 ngũ giác đều, F3 24 17 đa giác 17 cạnh, F4 28 257 đa giác 257 cạnh, F5 216 56537 đa giác 56537 cạnh Để ghi nhớ công ơn thực di chúc ông, người ta khắc đa giác 17 cạnh nội tiếp đường trịn mộ ơng Đối với dựng hình, chúng tơi xin nêu phân tích cách dựng CÁC BÀI TỐN Bài Chia đoạn thẳng AB cho trước thành ba phần LỜI GIẢI Phân tích: Giả sử chia đoạn thẳng AB ba phần điểm chia C D, nghĩa AC CD CB Qua A kẻ tia Ax Trên lấy điểm N Từ D C kẻ đường thẳng song song với BN cắt Ax Tại M N Theo định lí Thales, ta có AE EM MN Từ giả thiết AC CD CB ta suy AC CD BD AE EM MN , từ suy cách dựng Cách dựng: Qua A dựng tia Ax tùy ý Trên lấy ba điểm liên tiếp AE EM MN Từ E M kẻ đường thẳng song song với BN cắt AB C D Liên hệ tài liệu word toán SĐT (zalo): 039.373.2038 Website: tailieumontoan.com AC CD CB Bài tốn ln có nghiệm hình Chú ý: Chia đoạn thẳng n (n lẻ) đoạn thẳng làm tương tự Bài Cho đoạn thẳng AB Xác định điểm M AB cho AM MB LỜI GIẢI Phân tích: Từ giả thiết AM MB AM MB 2 AM AM AM MB AB Như chia đoạn AB phần nhau, lấy điểm M thỏa mãn AM MB Cách dựng: Kẻ tia Ax tùy ý Trên lấy điểm C D thỏa mãn AC 2cm, AD 5cm Từ C kẻ đường thẳng song song với DB cắt AB M AM MB Bài Dựng tam giác ABC, biết trung điểm cạnh AB, AC đường thẳng d chứa đường phân giác góc B LỜI GIẢI Phân tích: Giả sử tam giác ABC dựng Gọi E, I trung điểm AB, AC F điểm đối xứng E qua d Khi F nằm cạnh BC Cách dựng: Dựng điểm F đối xứng E qua d Từ F dựng đường thẳng song song với EI, đường thẳng cắt d B Dựng đỉnh A đường kéo dài BE cho EA EB Nối A với I cắt đường thẳng BF C Chứng minh: EA EB , EI song song với BC IA IB DBC D trung trực EF ABD Biện luận: Liên hệ tài liệu word toán SĐT (zalo): 039.373.2038 Website: tailieumontoan.com Khi E thuộc d d song song với EI tốn vơ nghiệm hình Ngược lại, tốn có nghiệm hình Bài Dựng tam giác ABC, biết cạnh BC đường cao, phân giác trung tuyến xuất phát từ đỉnh A chia góc BAC thành bốn góc LỜI GIẢI Phân tích: Giả sử dựng tam giác ABC có đường cao AH, phân giác AD trung tuyến AM Đường phân giác AD cắt đường tròn ngoại tiếp tam giác ABC E BAE EAC EB EC Mà MB MC M nằm trung trực BC; AH cao xuất phát từ A AH song song với ME HAE AEM (so le trong) Theo giả thiết HAD DAM DAM MEA tam giác MAE cân M MA ME M nằm trung trực AE M tâm đường tròn ngoại tiếp tam giác ABC BC đường kính BAC 90 MAC 90 2230 ACB 2230 Từ suy cách dựng tam giác ABC Bài Dựng tam giác, biết BC a, CA b , trung tuyến ma kẻ từ đỉnh A LỜI GIẢI Phân tích: Giả sử tam giác ABC dựng BC a, CA b, ma Gọi M trung điểm BC Tam giác AMC dựng (biết ba cạnh) Xác định đỉnh B Cách dựng: Dựng tam giác AMC với AC b, AM ma , MC a Trên MC kéo dài lấy điểm B thỏa mãn MB MC tam giác ABC dựng Liên hệ tài liệu word toán SĐT (zalo): 039.373.2038 Website: tailieumontoan.com Bài Dựng tam giác, biết góc BAC nhọn, đường cao hb trung tuyến ma LỜI GIẢI Phân tích: Giả sử tam giác ABC dựng được: biết góc BAC , đường cao BH hb Tam giác vng ABH hồn tồn xác định (biết góc nhọn cạnh góc vng) Gọi M, N trung điểm BC, AB MN song song với AC, BH vng góc với AC MN vng góc với BH MN qua trung điểm BH hay MN trung trực BH trung trực BH hoàn toàn xác định Mặt khác AM ma M giao điểm trung trực BH đường trịn tâm A, bán kính ma (M, H phía với AB) Xác định M suy xác định điểm C, tam giác ABC dựng Bài Dựng tam giác, biết góc ABC , ACB a b LỜI GIẢI Phân tích: Giả sử tam giác ABC dựng Kéo dài BC phía C lấy điểm D cho CD b hay CD CA tam giác ACD cân C ADC ACB tam giác ABD hồn tồn xác định (biết BD, góc ABC , ADB ) C giao điểm cạnh BD với trung trực AD Từ tam giác ABC hồn tồn xác định Bài Dựng tam giác, biết ba đường trung tuyến ma , mb , mc LỜI GIẢI Phân tích: Giả sử tam giác ABC dựng biết ba đường trung tuyến ma , mb , mc Gọi ba trung tuyến AD, BE, CI Ba trung tuyến đồng qui G Theo tính chất đường trung tuyến, ta có DG 1 2 AD ma , BG BE mb , CG mc 3 3 Gọi K trung điểm CG Liên hệ tài liệu word toán SĐT (zalo): 039.373.2038 Website: tailieumontoan.com GK song song với BE DK 1 1 BG mb , GK CG mc 3 tam giác GDK hoàn toàn xác định biết ba cạnh Cách dựng: Dựng tam giác DGK Kéo dài DG cho DA ma Xác định C cách kéo dài KC KG Xác định điểm B thỏa mãn DB DC Bài Dựng tam giác, biết độ dài đường cao hai trung tuyến mb , mc LỜI GIẢI Phân tích: Giả sử tam giác ABC dựng AH , BD mb , CE mc Xét điểm P, Q cho B, C trung điểm CP, BQ AP 2mb , AQ 2mc tam giác APQ hoàn toàn xác định Cách dựng: Dựng đường cao AH Dựng đường thẳng a qua H vuông góc với AH Từ đỉnh A dựng cạnh AP 2mb , AQ 2mc đường thẳng a Chia PQ ba đoạn xác định B C Từ P kẻ đường thẳng tùy ý lấy PM MN NK Từ M, N kẻ đường thẳng song song với KQ, xác định B C Bài 10 Dựng tam giác biết chân ba đường cao Liên hệ tài liệu word toán SĐT (zalo): 039.373.2038 Website: tailieumontoan.com LỜI GIẢI Phân tích: Giả sử tam giác ABC dựng biết chân ba đường cao nằm ba cạnh D, E, F Tứ giác BFHD nội tiếp FBH FDH Tứ giác DHEC nội tiếp HDF HCF Tứ giác BFEC nội tiếp FBE FCE FDA ADE DH phân giác góc FDE Tương tự EH phân giác góc DEF FH phân giác góc EFD H tâm đường tròn nội tiếp tam giác DEF Tam giác DEF hoàn toàn xác định H xác định tam giác ABC xác định Cách dựng: Dựng H tâm đường tròn nội tiếp tam giác DEF Từ đỉnh D, E, F kẻ đường thẳng vng góc với HD, HE, HF Giao điểm đường tam giác ABC cần dựng Bài 11 Dựng hình vng nội tiếp tam giác cho trước (một cạnh hình vuông nằm cạnh tam giác, hai đỉnh hình vng nằm hai cạnh cịn lại) LỜI GIẢI Phân tích: Giả sử hình vng ABCD dựng nội tiếp tam giác PQR cho trước Lấy điểm PQ, dựng MN song song với AB; NE, MF song song với BC Ta có NE QN MN NE MN BC QB AB Hình vng MNEF có cạnh EF nằm cạnh QR, MN song song với AB, BC song song với NE Q, N, B thẳng hàng Cách dựng: Dựng hình vng MNEF với cạnh tùy ý cho EF nằm cạnh QR M nằm cạnh PQ QN cắt PR B từ xác định hình vng ABCD nội tiếp tam giác PQR Với cách dựng ta có ba hình vng nội tiếp tam giác cho trước Cách dựng có tên đồng dạng Bài 12 Dựng ngũ giác đều, biết cạnh LỜI GIẢI Phân tích: Liên hệ tài liệu word tốn SĐT (zalo): 039.373.2038 Website: tailieumontoan.com Giả sử ngũ giác ABCDE dựng thỏa mãn cạnh a, góc 180 Đường chéo AD cắt BE I AEB EAD 36 IA IE , IAB AIB AB BI Tam giác AEB tam giác IAE đồng dạng (g.g) AB IE AB BE AB BE EA BE AB AB BE AB.BE Chia hai vế cho AB ta 1 BE BE BE BE 1 AB AB AB AB Giải phương trình bậc hai với ẩn BE 1 BE BE a AB 2 AB Cách dựng: Trước hết ta dựng BE 1 a , với a cạnh ngũ giác Cách khác dựng góc 36 Bài 13 Cho tam giác ABC Dựng M tam giác thỏa mãn MAB MBC MCA LỜI GIẢI * Cách 1: Giả sử điểm M dựng thỏa mãn: MAB MBC M nằm đường tròn O1 thỏa mãn: qua A B đồng thời tiếp xúc với BC B ( O1 giao điểm trung trực AB đường vng góc với BC kẻ từ B) MAB MCA M nằm đường tròn O2 qua A C đồng thời tiếp xúc với AB A ( O2 giao điểm trung trực AC đường vng góc với AB kẻ từ A) M giao điểm hai đường tròn O1 O2 Liên hệ tài liệu word toán SĐT (zalo): 039.373.2038 Website: tailieumontoan.com * Cách 2: Dựng đường tròn (O) qua A C, tiếp xúc với AB A Qua A kẻ đường thẳng song song với BC cắt đường tròn (O) D Nối F với B cắt đường tròn (O) M MAB MBC MCA (Bạn đọc tự chứng minh) Trong tam giác ABC nhọn tồn điểm M gọi điểm Broca thỏa mãn MAB MBC MCA , góc có tên gọi góc Broca Chú ý: Đây đề thi Vô địch Hunggari 1985, thêm giả thiết tam giác ABC vuông A) Bài 14 Cho ba đường thẳng a, b, c song song với Dựng tam giác có ba đỉnh thuộc ba đường thẳng LỜI GIẢI Phân tích: Giả sử ABC tam giác có đỉnh nằm ba đường thẳng song song a, b, c Kẻ CH vng góc với đường thẳng b Dựng tam giác HCE E cách đường thẳng b c E nằm đường thẳng song song với a, đường thẳng cắt BC F FB FC AF vng góc với BC FAC 30 FAC CEF tứ giác AFCE nội tiếp AE vuông góc với EC Cách dựng: - Dựng đường thẳng HC vng góc với đường thẳng c - Dựng tam giác HCE - Dựng đường thẳng vng góc với CE cắt đường thẳng a A CA cạnh tam giác - Dựng góc CAx 30 Từ C kẻ CF vng góc với Ax, cắt đường thẳng b B tam giác ABC dựng Bài 15 Cho tam giác nhọn ABC, dựng phía ngồi nửa đường trịn đường kính cạnh BC, CA, AB Tìm nửa đường trịn điểm D, E, K tương ứng thỏa mãn AE AK , BK BD, CD CE (Vô địch Hungari 1939) LỜI GIẢI Liên hệ tài liệu word toán SĐT (zalo): 039.373.2038 Website: tailieumontoan.com Phân tích: Giả sử tìm điểm D, E, K thỏa mãn AE AK , BK BD, CD CE Từ điểm D, E, K hạ đường vng góc DH BC , EN AC , KM AB K thuộc nửa đường trịn đường kính AB tam giác KAB vng K AK AM AB Tương tự: AE AN AC Mà AE AK nên: AM AB AN AC AM AN AC AB Mà MAN CAB tam giác NAM tam giác CAB đồng dạng (c.g.c) AMN ACB tứ giác MNCB tứ giác nội tiếp BMC BNC Tương tự: AMC AHC , ANB AHB Mặt khác ta có: BMC AMC AHB AHC BNC ANB 180 BMC AMC AHB AHC BNC ANB 90 A, H, D thẳng hàng AH đường cao tam giác ABC Tương tự BN CM đường cao tam giác ABC Cách dựng: Kẻ đường cao tam giác cắt nửa đường tròn điểm D, E, F thỏa mãn AE AK , BK BD, CD CE Bài 16 Cho đường tròn (O, R) điểm M đường tròn Dựng dây cung qua M có độ dài a cho trước LỜI GIẢI Phân tích: Giả sử dây AB dựng thỏa mãn AB a M thuộc đoạn AB Gọi I trung điểm AB OI vng góc với AB OI OA2 AI R a2 Liên hệ tài liệu word toán SĐT (zalo): 039.373.2038 10 Website: tailieumontoan.com I nằm đường tròn tâm O, bán kính R a2 MIO 90 I nằm đường trịn đường kính MO I giao điểm hai đường tròn dây AB dựng Cách dựng: Dựng đường tròn O, R a Dựng đường tròn đường kính MO Giao điểm hai đường xác định điểm I dây AB xác định Biện luận: Trước hết R a2 R a Nếu R a2 OM : tốn có hai nghiệm Nếu R a2 OM : tốn có nghiệm Nếu R a2 OM : toán vô nghiệm Bài 17 Dựng tam giác, biết góc, cạnh đối diện với góc bán kính đường trịn nội tiếp LỜI GIẢI Phân tích: Giả sử tam giác ABC dựng thỏa mãn: BC a, BAC , bán kính đường trịn nội tiếp r Gọi I tâm đường tròn nội tiếp tam giác ABC Theo tính chất đường trịn nội tiếp ta có BIC 90 I nằm cung chứa góc 90 , chắn đoạn BC a Đồng thời I nằm đường thẳng song song với cạnh BC cách BC khoảng r xác định điểm I đường tròn tiếp I , r đỉnh A giao điểm hai tiếp tuyến kẻ từ B C với đường tròn I , r Cách dựng: Liên hệ tài liệu word toán SĐT (zalo): 039.373.2038 11 Website: tailieumontoan.com Dựng đoạn BC a Dựng cung chứa góc 90 chắn đoạn BC Dựng đường thẳng song song với BC cách BC khoảng r Giao điểm đường thẳng với cung chứa góc tâm I Dựng đường tròn I , r Từ B C dựng tiếp tuyến với đường tròn I , r Hai tiếp tuyến cắt A Bài 18 Dựng tam giác, biết hai bán kính đường trịn nội tiếp, ngoại tiếp góc LỜI GIẢI Phân tích: Giả sử góc BAC biết, độ dài bán kính đường trịn nội, ngoại tiếp tam giác r R Giả sử tam giác ABC dựng nội tiếp đường tròn O, R BOC 2BAC , OB OC R tam giác OBC xác định xác định cạnh BC Gọi I tâm đường tròn nội tiếp tam giác ABC BIC 90 BAC ID vng góc cạnh BC ID r tam giác IBC xác định Từ tam giác ABC xác định Bài 19 Cho đường tròn (O) điểm M ngồi đường trịn, qua M dựng tiếp tuyến với đường trịn (O) LỜI GIẢI Phân tích: Giả sử MA tiếp tuyến đường tròn (O) A tiếp điểm MA vng góc với OA, hay MAO 90 điểm A nằm đường tròn đường kính MO Cách dựng: Dựng đường trịn đường kính MO cắt (O) A B MA, MB tiếp tuyến đường trịn kẻ từ M Bài tốn ln có hai nghiệm Bài 20 Cho hai đường trịn O1 , R1 O2 , R2 cắt hai điểm, với R1 R2 Dựng tiếp tuyến chung hai đường tròn LỜI GIẢI Liên hệ tài liệu word toán SĐT (zalo): 039.373.2038 12 Website: tailieumontoan.com Phân tích: Giả sử xy tiếp tuyến chung hai đường tròn O1 , R1 O2 , R2 Gọi A B tiếp điểm O1 A, O2 B vng góc với tiếp tuyến xy Từ O2 kẻ đường thẳng vng góc với O1 A cắt O1 A C ABO2C hình chữ nhật hay O1CO2 90 O1C O1 A O2 B R1 R2 Do C giao điểm đường trịn đường kính O1O2 đường trịn O1 , R1 R2 , O1C cắt đường tròn O1 , R1 A Qua A kẻ đường thẳng vng góc với O1C cắt O2 , R2 B AB tiếp tuyến chung hai đường tròn O1 , R1 O2 , R2 Lấy đối xứng xy qua O1O2 ta có tiếp tuyến thứ hai Chứng minh: Bạn đọc tự chứng minh Biện luận: Bài tốn ln có hai nghiệm Bài 21 Cho hai đường tròn O1; r1 O2 ; r2 không giao nhau, không chứa nhau, với r1 r2 Dựng tiếp tuyến chung hai đường tròn LỜI GIẢI Tiếp tuyến chung dựng Bài 20 Dựng tiếp tuyến chung trong: Gọi C D hai tiếp điểm M giao điểm CD O1O2 Tam giác MCO1 tam giác MDO2 đồng dạng (g.g) MO1 O1C r1 MO2 O2 D r2 Liên hệ tài liệu word toán SĐT (zalo): 039.373.2038 13 Website: tailieumontoan.com điểm M hoàn toàn xác định O1CM 90 C giao điểm đường trịn đường kính O1 M với đường trịn O1; r1 Nối CM cắt O2 D Từ suy ta có hai tiếp tuyến chung Kết luận: Bài tốn có tiếp tuyến chung hai tiếp tuyến chung Bài 22 Cho đường tròn O ba điểm D, E, F đường tròn Dựng tam giác ABC cho D, E, F điểm cung BC, CA, AB đường trịn O LỜI GIẢI Phân tích: Giả sử tam giác ABC dựng thỏa mãn D, E, F điểm cung BC, CA, AB DB DC , EC EA, FA FB O tâm đường tròn OA OB OC OE, OF trung trực AC AB OE vng góc với AC OF vng góc với AB Gọi giao điểm OE với AC H, OF với AB K Tứ giác AHOK nội tiếp HAK 180EOF Mà EOF 2EDF HAK 180 2EDF BAC 180 2EDF Tương tự, ABC 180 2DEF , BCA 180 2EFD E điểm cung AC ABE CBE EAC ABC 90DEF Liên hệ tài liệu word toán SĐT (zalo): 039.373.2038 14 Website: tailieumontoan.com Tương tự FAB BCA 90EFD EAF EAC CAB BAF 360 2EDF DEF EFD 180EDF A giao điểm đường tròn O cung chứa góc nhìn đoạn EF góc 180EDF Từ tam giác ABC hồn tồn xác định Bài 23 Cho đường tròn O , điểm A nằm đường trịn điểm M nằm đường trịn Tìm hai điểm B C đường tròn O cho: 1) M trọng tâm tam giác ABC 2) M trực tâm tam giác ABC LỜI GIẢI 1) Phân tích: Giả sử dựng tam giác ABC nhận M trọng tâm AM MD D trung điểm BC OD BC xác định B C Cách dựng: Kéo dài AM lấy điểm D cho AM MD Dựng đường thẳng qua D vng góc với OD Giao điểm đường thẳng với đường tròn O hai đỉnh B, C Chứng minh: Bạn đọc tự chứng minh Biện luận: Bài tốn có nghiệm hình với điều kiện AD R 2) Phân tích: Giả sử tam giác ABC dựng nhận M trực tâm Kéo dài AM cắt đường tròn O E Gọi I giao điểm AM với BC I IM IE AM đường cao tam giác AM BC đường thẳng qua I vng góc với AM giao với đường trịn O hai đỉnh B C Bài 24 Cho ba đường tròn rời Dựng đường trịn tiếp xúc trong, tiếp xúc ngồi ba đường trịn LỜI GIẢI Liên hệ tài liệu word tốn SĐT (zalo): 039.373.2038 15 Website: tailieumontoan.com Phân tích: Giả sử O1 , r , O2 , r , O3 , r ba đường trịn có bán kính rời Đường tròn nội tiếp tâm O, bán kính m nội tiếp (ngoại tiếp) ba đường tròn cho OO1 m r OO1 m r tâm O cách ba điểm O1 , O2 , O3 O tâm đường tròn nội tiếp tam giác O1O2O3 OO1 cắt đường tròn O1 , r A B xác định bán kính đường trịn nội (ngoại tiếp) với ba đường tròn O1 , r , O2 , r , O3 , r Cách dựng: Xác định điểm O tâm đường tròn ngoại tiếp tam giác O1O2O3 Xác định giao điểm OO1 với đường tròn O1 , r Bài 25 Cho ba điểm A, B, C không thẳng hàng Dựng điểm D theo thứ tự ABCD vừa tứ giác nội tiếp tứ giác ngoại tiếp LỜI GIẢI Phân tích: Giả sử tứ giác ABCD thỏa mãn vừa tứ giác nội tiếp vừa tứ giác ngoại tiếp Tứ giác ABCD nội tiếp đường tròn suy D thuộc đường tròn ngoại tiếp tam giác ABC Tứ giác ABCD ngoại tiếp, suy ra: AB CD BC AD AD CD AB BC d Theo giả thiết, điểm D thuộc cung AC không chứa B Giả sử AB BC AD CD Trên AD đặt AE d tam giác DEC cân E Đặt ABC Liên hệ tài liệu word toán SĐT (zalo): 039.373.2038 16 Website: tailieumontoan.com CDE 180 , DEC DCE AEC 180 Từ tam giác AEC hoàn toàn xác định (c.g.c) Cách dựng: Dựng đường tròn qua ba điểm A, B, C Dựng cung chứa góc 180 ABC đoạn AC khác phía với B Dựng đường trịn A, d Đường trịn cắt cung chứa góc E Tia AE cắt đường tròn ngoại tiếp tam giác ABC D tứ giác ABCD dựng Bài 26 Cho đường thẳng d độ dài hai đoạn thẳng r1 , r2 r1 r2 Dựng hai đường trịn bán kính r1 , r2 tiếp xúc ngồi nhau, đồng thời tiếp xúc với đường thẳng d LỜI GIẢI Phân tích: Giả sử hai đường trịn O1 , r1 O2 , r2 tiếp xúc A tiếp xúc với đường thẳng d B C O1O2 r1 r2 , O1B d , O2C d Hạ O2 D O1B O2 D BC , O1D r1 r2 Theo định lí Pythagoras ta có DO2 O1O2 O1D DO2 4r1r2 DO2 r1r2 Cách dựng: - Dựng MN r1 , NP r2 Dựng nửa đường trịn đường kính MP - Dựng NQ MP NQ r1r2 - Dựng DO2 r1r2 xác định B C Bài 27 Hai đường tròn O1 , r1 , O2 , r2 tiếp xúc A tiếp xúc với đường thẳng d B C dựng đường tròn tiếp xúc với tam giác cong ABC LỜI GIẢI Liên hệ tài liệu word toán SĐT (zalo): 039.373.2038 17 Website: tailieumontoan.com Phân tích: Gọi O, r đường tròn thỏa mãn tiếp xúc với O2 , r2 O1 , r1 , tiếp xúc với đường thẳng d E Giả sử r1 r2 O1O2 r1 r2 , O1D r1 r2 Qua O kẻ đường thẳng song song với BC cắt O1B M, cắt O2C N OO1 r1 r , OO2 r2 r , MN O2 D OM ON Theo định lí Pythagoras ta có 2 2 OM OO12 O1 M r1 r r1 r 4r1r OM r1r ON OO2 O2 N r2 r r2 r 4r2r ON r2r 2 DO2 O1O2 O1D r1 r2 r1 r2 4r1r2 O2 D r1r2 r1r r2r r1r2 r r1r2 r1 r2 Xác định r xác định độ dài O1O O2O Xác định vị trí O Đến nhiều người cho “dễ”, thực không dễ chút phải qua nhiều tốn dựng Ở có đến ba cách dựng đoạn r theo cách phân tích sau: 1) r 2) r r1r2 r1 r2 r1r2 r1 r1 r (áp dụng định lí Thales) r2 r r r r 2 1 1 (dựng nghịch đảo hai đoạn thẳng) r r1 r2 r2 r1r2 3) r1 r2 r1r2 r r1r2 (dựng đường tròn qua ba điểm) Bài 28 Cho trước đoạn thẳng a Dựng đoạn thẳng x a LỜI GIẢI Liên hệ tài liệu word toán SĐT (zalo): 039.373.2038 18 Website: tailieumontoan.com Phân tích: Viết đoạn thẳng cần dựng dạng: xa a2 a.a x a.a dựng đoạn trung bình nhân với nhiều cách dựng Trước hết dựng a : - Dựng liên tiếp đường thẳng hai đoạn AC a CD a - Dựng đường tròn đường kính AD a a - Dựng đường thẳng vng góc với AD C cắt đường trịn E CE đoạn thẳng cần dựng AB AC a, BC CD a Bài 29 Cho trước đoạn thẳng a, b, c Dựng đoạn thẳng x 1 a b c LỜI GIẢI Phân tích: Trước hết dựng đoạn thẳng d thỏa mãn 1 Với d a b này, ta có nhiều cách dựng Xin giới thiệu cách đơn giản Trước hết chứng minh: Tam giác ABC, M điểm tùy ý BC, từ B C kẻ đường thẳng song song với AM cắt AC, AB P, Q Chứng minh 1 AM BP CQ Theo định lí Thales: Tương tự AM CM BP BC AM BM CQ BC Cộng hai đẳng thức lại ta 1 AM BP CQ Cách dựng: Liên hệ tài liệu word toán SĐT (zalo): 039.373.2038 19 Website: tailieumontoan.com Vậy toán cần dựng 1 , CE c song song với AM, EM giao AC O, từ O kẻ đường x d c thẳng song song với AM OD x Với cách dựng 1 1 x a1 a2 a3 an Bài 30 Dựng đoạn thẳng x ab , với a, b, c ba đoạn thẳng cho trước c LỜI GIẢI Phân tích: x ab a c Dựng đoạn tỉ lệ thứ tư định lí Thales c x b Cách khác: x ab c.x a.b (áp dụng hệ thức lượng đường tròn) c Cách dựng: Cách 1: OA c, OB b, OC a , BD song song với AC OD x Cách 2: Dựng AC a, CB b, DC c Qua ba điểm A, D, B dựng đường tròn Kéo dài DC cắt đường tròn E AC.CB DC.CE CE x Liên hệ tài liệu word toán SĐT (zalo): 039.373.2038 20